Samenvatting bij de 2e druk van Neuroanatomy through Clinical Cases van Blumenfeld


Wat zijn de basisprincipes van neuroanatomie? - Chapter 2

 

Hoe is het zenuwstelsel georganiseerd?

Het zenuwstelsel bestaat uit het centraal zenuwstelsel (CZS) en het perifeer zenuwstelsel (PZS). Het centraal zenuwstelsel wordt gevormd door de hersenen en het ruggenmerg en het perifere zenuwstelsel behelst de rest.

Het CZS is embryologisch ontstaan uit een plaat van ectodermale cellen, die vouwt en de neurale buis vormt. Zwellingen in de kop van de neurale buis vormen de hersenen. De rest vormt het ruggenmerg. Holtes in de neurale buis die gevuld zijn met vocht, vormen de ventrikels die gevuld zijn met cerebrospinale vloeistof (CSF).

De hersenen in ontwikkeling hebben drie onderdelen:

  1. Het prosencephalon (voorbrein): dit is het grootste deel en wordt verder onderverdeeld in het:

    1. Telencephalon: de telencephalon zijn de twee hemisferen. Deze bevatten weer structuren als de cortex, de witte massa en de basale ganglia, die later besproken zullen worden.

    2. Diencephalon: het diencephalon bestaat uit de thalamus en de hypothalamus en de bijbehorende structuren.

  2. Het mesencephalon (middenbrein) is een relatief klein stukje dat het voor- en achterbrein met elkaar verbindt.

  3. Het rhombencephalon (achterbrein) bestaat uit de pons, het cerebellum en de medulla. De pons en het cerebellum samen zijn het metencephalon en de medulla is het myelencephalon.

Het mesencephalon, de pons en de medulla vormen de connectie tussen de hersenen en het ruggenmerg. Dit wordt ook wel de hersenstam genoemd, omdat de hersenen als een soort bloemkool op deze structuren ligt. De hersenstam is het oudste deel van onze hersenen en is belangrijk voor vele overlevingsfuncties van het lichaam, zoals de hartfrequentie, ademhaling, etc.

Dus als ezelsbruggetje: telencephalon (hemisferen), diencephalon (thalamus + hypothalamus), mesencephalon, metencephalon (pons + cerebellum) en myelencephalon (medulla). (tel die mes met my).

Van boven naar beneden zijn er de volgende structuren: prosencephalon à midbrain-diencephalon junction à midbrain à pontomesencephalic junction à pons à pontomedullary junction à medulla à cervicomedullary junction

Cerebrospinale vloeistof (CSF) wordt gevormd door ependymcellen van de plexus choroideus in de laterale ventrikels. De cerebrospinale vloeistof stroomt van de laterale ventrikels naar de derde ventrikel en verlaat via de foramina van het vierde ventrikel de ventrikels. De hersenen worden omgeven door drie hersenvliezen. Van binnen naar buiten zijn dit de pia mater, de arachnoidea (spinragvlies) en de dura mater. Wanneer de cerebrospinale vloeistof de ventrikels verlaat, stroomt het naar de ruimte tussen de pia mater en de arachnoidea (de subarachnoidale ruimte), waar het uiteindelijk wordt opgenomen in het veneuze systeem dat hierop uitkomt. Dit gebeurt door ankervenen die uitmonden in de veneuze sinussen.

Bij dieren is het zenuwstelsel in één lineaire richting georiënteerd, maar bij mensen is er een knik van 90 graden ter hoogte van het middenbrein en de diencephale overgang.

Bij dieren geldt dat ventraal naar de aarde is (inferior). Dorsaal is naar de hemel (superior). Rostraal is naar de snuit (anterior/craniaal). Caudaal is naar de staart (posterior).

Bij mensen geldt dat voor structuren boven het middenbrein de volgende anatomische benamingen geldig zijn: anterior = rostraal, posterior = caudaal, superior = dorsaal en inferior = ventraal. Voor het middenbrein en daaronder geldt: anterior = ventraal, posterior = dorsaal, superior = rostraal en inferior = caudaal.

Wat zijn de definities van secties?

Horizontaal is parallel aan de vloer, loodrecht op de lange as (superior-inferior) van het lichaam). Het wordt ook wel axiaal of transversaal genoemd.

Coronaal is loodrecht op de anterior-posterior as, parallel aan de lange as van het lichaam. Hiervoor wordt ook wel de benaming frontaal gebruikt.

Sagittaal is loodrecht op de links-rechts as

  • Midsagittaal is door de middenlijn

  • Parasagittaal is links of rechts naast de middenlijn

Oblique is tussen horizontaal, coronaal en sagittaal in.

Hoe is de cellulaire organisatie van het zenuwstelsel opgesteld?

Het zenuwstelsel bestaat uit twee typen cellen, namelijk neuronen (zenuwcellen) en gliacellen (ondersteunende cellen).

Neuronen

De neuronen zorgen voor de verspreiding van signalen door het zenuwstelsel. Een neuron bestaat uit een cellichaam met een nucleus en cytosol. Het heeft uitlopers genaamd dendrieten (kort, ontvangen input) en axonen (lang, verzorgen de output). Axonen splitsen vaak distaal in meerdere axon collateralen. Er zijn verschillende soorten neuronen. Multipolaire neuronen hebben meerdere dendrieten en axonen. Vaak hebben deze cellen maar één lang axon dat verschillende zijtakken afgeeft. Bipolaire neuronen hebben één axon en één dendriet. Deze zijn meestal sensorisch, zoals de bipolaire cellen in de retina. Sommige neuronen zijn pseudo-unipolair, wat betekent dat de uitlopers eerst samenkomen en vervolgens splitsen in twee axonen.

Unipolaire neuronen, waarbij zowel de dendrieten en axonen ontspringen uit één aftakking, bestaan voornamelijk in ongewervelden.

Communicatie tussen neuronen gebeurt bij een synaps. Dit is de plek waar een axon een uitmondt op een ander neuron (ofwel een dendriet, cellichaam of axon). Bij een chemische synaps worden uit het axon neurotransmitters vrijgelaten uit de synaptische blaasjes. Deze neurotransmitters worden via exocytose vanuit het axon in de synaptische spleet uitgestoten. De neurotransmitters binden vervolgens aan neurotransmitterreceptoren op het postsynaptische membraan van de dendriet. Het postsynaptische neuron wordt hierdoor ofwel gestimuleerd, ofwel geinhibeerd.

Bij een elektrische synaps wordt er geen gebruik gemaakt van neurotransmitters, maar wordt de elektrische stroom doorgegeven via gap-junctions. Voor de rest werkt het wel hetzelfde als een chemische synaps.

Een actiepotentiaal ontstaat als het membraan van een neuron voldoende wordt gestimuleerd. Het zorgt voor de release van neurotransmitters uit synaptische blaasjes uit het presynaptische membraan.

Glia cellen

Glia cellen zijn de ‘steuncellen’ van het centrale zenuwstelsel en er zijn 4 typen glia cellen: ependymcellen, microgliacellen, astrocyten en oligodendrocyten. De oligodendrocyten vormen een lipide myelineschede om de axonen in het centrale zenuwstelsel voor een snellere geleiding van de actiepotentiaal. In het perifere zenuwstelsel heten deze glia cellen Schwann cellen. Tussen de myelinescheden zijn stukjes axon ongeïsoleerd: dit heet de knoop van Ranvier. Hier zitten voltage gereguleerde ion kanalen. Saltatoire geleiding is het snelle proces waarbij geleiding plaatsvindt van knoop tot knoop. Ependymcellen zitten in de plexus choroideus en deze cellen produceren cerebrospinale vloeistof (CSF). Astrocyten vormen met hun uitlopers de bloed-hersenbarriere en microgliacellen zijn de afweercellen van het zenuwstelsel.

Wat zijn de functies van neurotransmitters?

De functies van neurotransmitters zijn snelle communicatie en neuromodulatie. De snelle communicatie tussen neuronen wordt gedaan door excitatoire postsynaptische potentialen (EPSP’s) en inhibitoire postsynaptische potentialen (IPSP’s). Beide worden door postsynaptische neuronen vrijgelaten door integratie van meerdere inputs. Deze integratie gebeurt in de axon heuvel. Neuromodulatie is de langzame verandering van intracellulaire mechanismen.

Neurotransmitters zijn talrijk, kunnen kleine en grote moleculen zijn en verschillen in functie. Dit verschil in functie is niet alleen afhankelijk van de soort neurotransmitter, maar ook van de receptoren. Er worden vaak meerdere neurotransmitters tegelijk vrijgelaten bij één actiepotentiaal.Neurotransmitters van het centraal zenuwstelsel zijn:

  • Excitatoir: glutamaat,

  • Inhibitoir: GABA.

Neurotransmitter voor perifeer, somatische zenuwstelsel: acetylcholine (neuromusculair).

Neurotransmitter autonoom zenuwstelsel: acetylcholine en norepinephrine, oftewel noradrenaline.

Witte en grijze stof

Het centraal zenuwstelsel bestaat zowel in de hersenen als in het ruggenmerg uit witte en grijze stof. Witte stof zijn gemyeliniseerde neuronen, via welke de signalen worden vervoerd. Grijze stof zijn de cellichamen. Deze liggen in de cerebrale cortex, de thalamus, de basale ganglia en craniale zenuwkernen.

In de hemisferen ligt de grijze stof aan de buitenkant en witte stof aan de binnenzijde. In de hersenstam verschilt dit. In het ruggenmerg ligt de witte stof juist aan de buitenzijde en de grijze stof aan de binnenzijde.

Er zijn veel verschillende namen voor routes van witte stof, zoals fasciculus, lemniscus en bundel. Commisuren zijn de witte stof bundels die structuren van de ene hemisfeer met structuren van de andere hemisfeer verbinden. Axonen van het perifere zenuwstelsel vormen bundels die de perifere zenuwen worden genoemd. Clusters van cellichamen in het perifere zenuwstelsel worden ganglia genoemd.

Over het algemeen worden zenuwen die informatie zenden naar de structuren toe afferente vezels genoemd. Zenuwen die informatie van de integrerende structuren naar de doelcellen transporteren worden efferente vezels genoemd. De perifere zenuwen zenden dus sensorische informatie via afferente zenuwen naar het centraal zenuwstelsel en brengen motorische signalen via efferente zenuwen van het centraal zenuwstelsel naar de periferie toe.

Hoe werken ruggenmerg en het perifere zenuwstelsel?

Het menselijke zenuwstelsel vormt verschillende segmenten, net als bij dieren. De segmenten van de hemisferen en de hersenstam zijn deels gefuseerd. Twaalf craniale zenuwen verlaten deze hersenstructuren. De spinale zenuwen ontspringen van de verschillende segmenten van het ruggenmerg. In het zenuwstelsel lopen de sensorische zenuwwortels meer dorsaal. De motorische zenuwwortels ontspringen meer ventraal. Deze worden vernoemd naar het segment waar ze ontspringen, dus cervicaal, thoracaal, lumbaal of sacraal. Tijdens de ontwikkeling van het lichaam groeit de benige ruggengraat sneller dan het ruggenmerg. Daarom reikt het ruggenmerg tot ongeveer L1 of L2. Onder dit niveau liggen de zenuwen los en wordt er gesproken van de cauda equina (paardenstaart).

Voor het aansturen van de armen en benen zijn veel meer signalen nodig dan voor bijvoorbeeld de borstkas. Daarom komen de zenuwen van de armen en benen na het uittreden uit het wervelkanaal weer bij elkaar en vormen daar een complex netwerk van zenuwen. Deze worden respectievelijk de brachiale en lumbosacrale plexus genoemd. Ook wordt hier in het ruggenmerg meer grijze stof gevonden dan in andere delen van het ruggenmerg. Deze verdikte ruggenmerg segmenten worden respectievelijk de cervicale en lumbosacrale verdikking genoemd.

Wat is het autonome zenuwstelsel?

Het autonome zenuwstelsel reguleert de automatische processen, zoals de hartfrequentie, peristaltiek, etc. Dit zijn zaken waar de mens geen actieve invloed met de wil op kan uitoefenen. Het autonoom zenuwstelsel wordt verdeeld in twee componenten: de (ortho)sympathische component en de parasympathische component. De (ortho)sympathische component van het autonoom zenuwstelsel heet ook wel de thoracolumbale component. De zenuwen van dit deel van het zenuwstelsel ontspringen uit de segmenten Th1 tot en met L2 van het ruggenmerg. Het zorgt voor de flight-or-fight reactie van het lichaam, zoals toegenomen hartfrequentie, toegenomen bloeddruk, bronchodilatatie en pupilverwijding. De belangrijkste neurotransmitter die werkt op de doelcellen is norepinephrine, oftewel noradrenaline.

Het parasympathische deel van het autonoom zenuwstelsel heet ook wel het craniosacrale deel. De zenuwen van dit deel van het zenuwstelsel ontspringen uit de segmenten S2 tot S4 van het ruggenmerg en daarnaast wordt het gevormd door de hersenzenuwen III (oculomotorius), VII (facialis), IX (glossopharyngeus) en X (vagus). Het is betrokken bij de rest-and-digest reacties, zoals meer gastrische secretie en peristaltiek, vertragen van de hartfrequentie en pupilversmalling. De belangrijkste neurotransmitter is acetylcholine. Het sympathische en parasympathische systeem worden beïnvloed door hoger gelegen centra als de hypothalamus, het limbische systeem en sensorische informatie uit de periferie.

Wat is de cerebrale cortex?

De hersen zijn niet glad, maar bevatten verschillende bergen en dalen, ook wel respectievelijk de gyri en de daartussen gelegen sulci (invouwingen) genoemd. De twee hemisferen zijn van elkaar gescheiden door de interhemisferische fissuren. Ze zijn wel met elkaar verbonden door het corpus callosum. Het corpus callosum bestaat uit het rostrum, het genu, het lichaam en het splenium. De hersenen worden verdeeld in vier hersenkwabben. De vier hersenkwabben zijn de frontale, parietale, temporale en occipitale kwab. De frontale kwab ligt anterieur, daarachter ligt de parietale kwab. De temporale kwab ligt lateraal en inferior en de occipitaal kwab ligt posterior inferior.

De frontale kwab loopt tot de centrale sulcus (van Rolando). Deze sulcus vormt ook de grens tussen de frontale kwab en de parietale kwab. De grens tussen de temporale kwab en de frontale kwab wordt gevormd door de Sylvian (of laterale) fissuur.

De parietale kwab ligt achter de centrale sulcus en voor de parieto-occipitale sulcus. Deze laatste vormt de grens met de occipitale kwab.

De insulaire cortex is een aparte structuur die in het midden van de hemisferen ligt. Anterior ligt het frontale operculum en posterior ligt het parietale operculum eroverheen. Dit zijn delen van de frontale en parietale kwabben.

Sulci en gyri

De centrale sulcus ligt, van lateraal gezien, superior iets verder naar posterior en inferior iets verder naar anterior. Voor de centrale sulcus loopt de precentrale gyrus. Erachter loopt de postcentrale gyrus. Nog verder anterior van de centrale sulcus is het oppervlak verdeelt door de superiore en inferiore frontale sulci. Deze verdelen het oppervlak in de superiore, mediale en inferiore frontale gyri.

Achter de centrale sulcus ligt de postcentrale gyrus, en hier begint de parietale kwab. De intraparietale sulcus scheidt de parietale kwab in superior en inferior. De inferiore parietale kwab bestaat uit de supramarginale en de angulaire gyrus.

De cingulaire gyrus loop parallel aan het corpus callosum. Het bestaat uit anterior de paraterminale gyrus en posterior de isthmus. Vlak voor de cingulaire sulcus loopt de centrale sulcus. Hieronder loopt de precentrale lobulus.Onder de calcarine fissuur ligt de lingula, erboven ligt de cuneus. Beide zijn delen van de occipitaal kwab. Voor de cuneus ligt de precuneus (parietaal).

Aan het inferiore oppervlak ligt anterior de orbitale frontale gyri. Mediaal ligt de olfactoire sulcus, die de orbitale frontale gyrus en de gyrus rectus scheidt.

Meer naar posterior ligt in de temporaalkwab de inferiore temporale sulcus. Deze scheidt de inferiore temporale gyrus (lateraal) van de occipitotemporale gyrus (mediaal). Naast de occipitotemporale gyrus ligt de collaterale sulcus, waarnaast (mediaal) de parahippocampale gyrus ligt.

Wat zijn de motorische en sensorische cortex?

De primaire motorische cortex wordt gevormd door de precentrale gyrus en de primaire somatosensorische cortex wordt gevormd door de postcentrale gyrus. De primaire visuele cortex ligt in de occipitaal kwab bij de calcarine fissuur. De primaire auditieve cortex ligt in de transversale gyri van Heschl bij de Sylvian fissuur in de temporaal kwab.

De verschillende cortexen hebben een topografische verdeling van de binnenkomende en uitgaande signalen. Dat wil zeggen dat bepaalde informatie naar een bepaalde plek gaat. Bij de somatosensorische en motorische cortex is dit voor te stellen als een homunculus, een mannetje dat over de cortex ligt en aangeeft waar welke informatie wordt verwerkt.

Het zenuwstelsel verwerkt informatie gekruisd. Dit betekent dus dat de somatosensorische en motorische cortex het lichaam innerveren aan de andere helft dan dat het in de hersenen ligt. Dit wil dus zeggen dat informatie uit het rechter been in de linker hemisfeer wordt verwerkt. In het oog is dit lastiger. Hier wordt de informatie die links valt in beide ogen, naar de rechter helft getransporteerd. Informatie uit bijvoorbeeld het rechter oog wordt dus gedeeltelijk in de linker hemisfeer en gedeeltelijk in de rechter hemisfeer verwerkt.

De cortex bestaat voor het grootste deel uit neocortex. Dit bestaat uit zes cellagen:

  • Laag 1 is de moleculaire laag, die bestaat uit dendrieten en axonen van dieper gelegen neuronen.

  • Laag 2 en 3 zijn de kleine en medium pyramidale laag, die bestaan uit neuronen met axonen in de cortex.

  • Laag 4 is de granulaire laag, die input van de thalamus ontvangt.

  • Laag 5 is de grote pyramidale laag, bestaand uit neuronen met axonen naar structuren buiten de cortex, maar niet naar de thalamus.

  • Laag 6 is de polymorfe laag, die bestaat uit neuronen met axonen naar de thalamus.

De grootte van de laag is afhankelijk van de functie van de cortex. Aan de hand van de verschillende samenstellingen van deze lagen kunnen de hersenen in 52 gebieden verdeeld worden, die overeenkomen met de verschillende functies van de hersenen. Dit wordt de verdeling van Brodmann genoemd.

De belangrijkste motorische vezelbaan is de corticospinale tractus, ook wel de pyramidale tractus genoemd. Deze begint in de cortex en loopt via de hersenstam naar het ruggenmerg. Ter hoogte van de overgang van de medulla naar het ruggenmerg, kruist deze baan de mediaanlijn. Dit heet de pyramidale deccusatie.

In het ruggenmerg synapteren de axonen (van de upper motor neurons of UMNs) op de lower motor neurons (LMNs) in de voorhoorn van de grijze stof van het ruggenmerg. Vanuit deze lower motor neurons ontspringen nieuwe axonen, die het ruggenmerg verlaten via de ventrale zenuwwortels.

Laesies van de corticospinale tractus boven de kruising, veroorzaken contralaterale uitval ten opzichte van de laesie. Laesies van de corticospinale tractus onder de kruising, veroorzaken ipsilaterale uitval ten opzichte van de laesie.

Cerebellum en basale ganglia

Het cerebellum en de basale ganglia zorgen voor soepele bewegingen door middel van feedback systemen. De basale ganglia ontvangen input van de motorische cortex. Het cerebellum ontvangt input van de motorische cortex, de hersenstam en het ruggenmerg. Beide sturen output naar de thalamus en via daar naar de motorische cortex. Zo beïnvloeden ze de corticospinale tractus.

Laesies van het cerebellum veroorzaken ataxie, afwijkingen in coördinatie en balans. Laesies in de basale ganglia veroorzaken ofwel hypokinetische (bijv. ziekte van Parkinson) ofwel hyperkinetische bewegingsafwijkingen (bijv. ziekte van Huntington).

Somatosensorische systemen

Somatosensorische informatie bestaat uit gevoel waarvan je je bewust wordt. Er zijn twee vezelbanen in het ruggenmerg hiervoor. De posterior column pathway (gnostische sensibiliteit) voor proprioceptie, vibratie en fijne tast. De anterolateral pathway (vitale sensibiliteit) voor pijn, temperatuur en grove tastzin.

De cellichamen van de neuronen liggen in de spinale dorsale wortel ganglia. De axonen lopen naar de periferie en naar het ruggenmerg.

Gnostische sensibiliteit

De gnostische vezelbanen komen binnen in het ruggenmerg in de dorsale hoorn en gaan naar de dorsale witte stof. Ze blijven ipsilateraal tot in de medulla. Hier synapteren de axonen en de volgende axonen kruisen naar de contralaterale kant. Deze synapteren weer in de thalamus en vervolgens gaan de axonen naar de primaire somatosensorische cortex.

Vitale sensibiliteit

De vezelbanen van de vitale sensibiliteit komen binnen in het ruggenmerg in de dorsale hoorn en synapteren meteen in de grijze stof. De volgende axonen kruisen meteen naar de contralaterale kant in de anterolaterale witte stof. Ook deze synapteren in de thalamus en gaan vervolgens naar de primaire somatosensorische cortex.

Thalamus

De thalamus is het integratiecentrum van de hersenen en ligt midden in de hersenen. Het bestaat uit een aantal nuclei. Alle sensorische informatie (reuk uitgezonderd) gaat via de thalamus naar de cortex. Elk zintuig heeft zijn eigen kern. Ook niet-sensorische informatie gaat langs de thalamus (bv. van het cerebellum en de basale ganglia).

Wat belangrijk is, is dat de informatie die van de thalamus naar de cortex toegaat, ook weer teruggestuurd wordt naar de thalamus.Het diencephalon bestond behalve uit de thalamus ook uit de hypothalamus en de epithalamus. De hypothalamus is voor de controle van autonome, neurroendocriene en limbische informatie. De epithalamus bestaat uit de pijnappelklier (epifyse) die melatonine produceert, habenula en het pretectum.

Monosynaptische strekreflex

Reflexen geven belangrijke informatie over verschillende zenuwbanen en de hersenen.

Een reflex begint bij de spierspoeltjes. Deze sturen informatie naar de dorsale hoorn in de grijze stof in het ruggenmerg. Deze synapteert en er wordt meteen een motorisch axon geactiveerd in de voorhoorn die via de ventrale zenuwwortels een efferent signaal terugstuurt naar de spier. Deze spier trekt dan samen. Er zijn dus geen hersenstructuren bij betrokken.

Er zijn ook descenderende banen vanuit de hersenen die toch invloed uitoefenen op de reflex. Als deze banen of betrokken hersenstructuren niet goed werken, worden de reflexen hyper- of hypoactief.

Wat is de hersenstam?

De hersenstam bestaat uit het middenbrein, de pons en de medulla. In de hersenstam zitten kernen voor de craniale zenuwen, andere kernen die belangrijk zijn voor motorische functies en witte stof banen. Alle informatie van en naar de cortex en de periferie verloopt namelijk via de hersenstam. In de hersenstam zit de reticulaire formatie met vele belangrijke kernen. De kernen caudaal hebben te maken met motorische en autonome functies. De kernen rostraal in de pons en het middenbrein hebben te maken met het bewustzijn, door thalamische en corticale activiteit te moduleren.

Craniale zenuwen

Er zijn twaalf craniale zenuwen (CN), waarvan enkele slechts uit sensorische of motorische vezels bestaan en enkele uit zowel motorische als sensorische vezels:

  1. N. Olfactorius (N. I) is voor reuk

  2. N. Opticus (N.II) is voor zicht

  3. N. Oculomotorius (N.III) is voor de buitenste oogspieren, behalve die door de hersenzenuwen IV en VI worden geinnerveerd

  4. N. Trochlearis (N.IV) innerveert de m. obliquus superior van het oog

  5. N. Trigeminus (N.V) is voor algemene sensibiliteit in het gezicht, de mond en de neussinussen; voor de kauwspieren en de m. tensor tympani

  6. N. Abducens (N.VI) is voor de m. rectus lateralis van het oog

  7. N. Facialis (N.VII) is voor de spieren van het gezicht (expressie); de m. stapedius; het anterieure 2/3e deel van de smaak voor de tong; huilen, en de sublingulaire en submandibulaire speekselklieren

  8. N. Vestibulocochlearis (N.VIII) is voor het gehoor en het evenwicht

  9. N. Glossopharyngeus (N.IX) is voor de m. stylopharyngeus; het posterieure 1/3 deel van de smaak voor de tong; chemo- en baroreceptoren van de sinus caroticus en de parotide speekselklier

  10. N. Vagus (N.X) is voor de slikspieren; de stembanden; parasympathische vezels naar het hart, de longen en het maag-darm kanaal; en chemo- en baroreceptoren van de aortaboog

  11. N. Accesorius(N.XI) is voor de m. sternocleidomastoideus en m. trapezius

  12. N. Hypoglossus(N.XII) is voor de intrinsieke spieren van de tong

Limbische systeem

Het limbische systeem heeft te maken met emoties, geheugen en autonome en neuroendocriene controle. Het bestaat uit de hippocampus, de amygdala, delen van de frontale kwabben en kernen in de (hypo)thalamus, basale ganglia en hersenstam.

De fornix is een van de structuren die de limbische componenten met elkaar verbindt. De fornix zit tussen de hippocampus en de hypothalamus.Laesies in het limbische systeem kunnen problemen geven met het vormen van nieuwe herinneringen, psychiatrische stoornissen en epilepsie.

Associatieve cortex

De associatieve cortex is eigenlijk alle andere ruimte in de cerebrale hemisferen dan de primaire motor cortex en primaire sensorische cortexen. Het verbindt informatie uit de verschillende cortexen.In unimodale associatie is er een verband tussen één sensorische of motorische modaliteit en deze ligt vlakbij de primaire cortex. In heteromodale associatie worden verbanden gelegd tussen meerdere modaliteiten.

Taal wordt ontvangen in de primaire auditieve cortex. In gebied van Wernicke wordt dit omgezet in taalbegrip. Laesies hierin veroorzaken onbegrip in taal, en heten receptieve afasie. In het gebied van Broca wordt de uitvoering van taal gedaan, dus het bewegen van mond, lippen, etc. Laesies hierin veroorzaken een taalgebrek en heten expressieve afasie. Beide structuren liggen in dezelfde hemisfeer, meestal links (90% van de gevallen).

Als er een laesie ontstaat in de inferiore parietale kwab ontstaat het Gerstmann’s syndroom. Hierbij is er een scala aan functiestoornissen, zoals moeite met rekenen, links en recht door elkaar halen en moeite met geschreven taal. Laesies in de hersenen kunnen altijd een stoornis veroorzaken in de motoriek, zoals planning en uitvoeren. Dit heet apraxie.

De parietale kwabben zorgen voor ruimtelijke oriëntatie. Als er hier een laesie optreedt, zorgt dat voor een verdraaide ruimtelijke perceptie en het negeren van de contralaterale zijde. Aan de non-dominante hemisfeer is er hemineglect. De patiënt heeft geen interesse of bewustzijn van die kant, van het eigen lichaam of de omgeving. Ook kan er sprake zijn van extinctie. Als een stimulus aan beide zijden wordt gepresenteerd, wordt de contralaterale kant van de laesie genegeerd.

Meestal geeft een laesie in de dominante hemisfeer minder klachten, omdat deze meer gespecialiseerd is in spraak.

Bij laesies in de frontale kwab kunnen persoonlijkheidsstoornissen optreden, die als asociaal opgevat worden. Ook kunnen de patiënten complexe handelingen met meerdere stappen in een bepaalde volgorde niet uitvoeren. Ze blijven dan hangen in één van de stappen.

Laesies in de visuele associatie cortex kunnen verschillende symptomen hebben.Prosopagnosia is het onvermogen om gezichten te herkennen. Achromatopsia is het onvermogen om kleuren te herkennen. Palinopsia is het steeds terugkeren van een eerder gezien beeld.

Hoe werkt de bloedvoorziening van hersenen en ruggenmerg?

De aa. carotis interna en aa. vertebralis voorzien de hersenen van bloed. De vv. jugularis interna draineren de hersenen.

De aa. vertebralis komen bij elkaar en vormen de a. basilaris. De a. basilaris vormt de twee aa. cerebri posterior (PCA), die het achterste deel van de cirkel van Willis vormen. De aa. cerebri posterior zitten door middel van de a. communicans posterior (PComm) vast aan de aa. carotis interna. Uit de beide aa. carotis interna ontspringen de aa. cerebri media (MCA) en aa. cerebri anterior (ACA). De aa. cerebri anterior zitten aan elkaar vast door de a. communicans anterior (AComm).

De aa. cerebri posterior, aa. cerebri media en aa. cerebri anterior voorzien de cerebrale hemisferen van bloed.

Het cerebellum en de hersenstam worden door de a. cerebellaris superior, anterior en posterior voorzien van bloed. Deze ontspringen uit de aa. vertebralis en a. basilaris.

Het ruggenmerg wordt van bloed voorzien door de a. spinalis anterior (ventraal) en de twee aa. spinalis posterior (dorsaal). In de cervicale regio ontspringen deze uit de a. vertebralis, in de thoracolumbale regio uit de aorta.

Hoe leert neurologische onderzoek meer over neuroanatomie? - Chapter 3

 

Hoe werkt neurologische onderzoek?

Onderzoekstechnieken en tests

Het neurologische onderzoek test de verschillende functies van iemands lichaam.

Voor het neurologische onderzoek wordt er gekeken naar 6 verschillende onderdelen:

  1. Mentale status

Hierbij wordt er gekeken naar het niveau van alertheid door de patiënt een woord te laten spellen en weer achterstevoren te laten spellen. Normaal gesproken kan iemand woorden met 6 of meer letters in normale volgorde spellen en woorden van 4 of meer letter achterwaarts spellen. Ook is dit natuurlijk afhankelijk van de leeftijd en het educatieniveau van de patiënt. Hierbij wordt er gekeken of er een bilaterale laesie aanwezig is van de thalamus of de cerebrale hemisferen.

Er wordt ook gekeken naar de oriëntatie. Hierbij wordt er gevraagd naar iemands naam, locatie en de datum van de dag. Hierbij wordt het lange termijn geheugen getest. Wanneer een patiënt beperkt antwoord kan geven op deze vragen, dan is het van belang dat de onderzoeker het exacte antwoord van de patiënt noteert.

Het geheugen van de patiënt wordt getest. Dit wordt gedaan door de patiënt een kort verhaaltje te vertellen en aan de patiënt te vragen of deze het verhaal zou willen herhalen na ongeveer 3-5 minuten. Zorg dat de patiënt tussentijds een beetje afgeleid is. Op deze wijze wordt het korte termijn geheugen van de patiënt getest. Om het lange termijn geheugen te testen moet er gevraagd worden naar persoonlijke ervaringen van langer terug.

De taal wordt getest. Bij spontane spraak wordt gelet op hoe vloeiend de patiënt spreekt, de lengte van zinnen en de hoeveelheid spontane spraak. Modulatie van toon en parafrase fouten, net als neologismen (zelf bedachte woorden) moeten worden opgevangen. Daarnaast moet gelet worden op het begrip van de patiënt. Snapt de patiënt simpele vragen en daarnaast moet ook gekeken worden of begrip heeft van grammaticale structuren.

Het rekenen, recht-links verwarring, vinger agnosie en agrafie worden getest. Indien al deze componenten zijn verstoord, wordt er gesproken van het Gerstmann’s syndroom. Het rekenen wordt getest door de patiënt simpele rekensommen op te laten lossen. De patiënt wordt gevraagd links en rechts aan te geven. De patiënt wordt gevraagd zijn vingers te benoemen. De patiënt wordt gevraagd zijn/haar naam op te schrijven en een zin, die wordt gegeven door de onderzoeker.

Er wordt gekeken of er sprake is van apraxie. Er is sprake van apraxie indien iemand niet in staat is een motorische opdracht uit te voeren. Dit kan getest worden door de patiënt te vragen een alledaagse bezigheid voor te wenden, zoals het kammen van zijn/haar haar.

“Neglect” is wanneer een patiënt een kant van het lichaam geheel negeert, omdat de patiënt zich niet meer bewust is van dit deel. Dit kan getest worden door te kijken hoe de patiënt andere opdrachten uitvoert, bijvoorbeeld wanneer de patiënt tijdens het schrijven één kant van de pagina onbenut laat. Er moet ook gekeken worden naar anosognosia. Dit is wanneer de patiënt ook ontkent dat er met een bepaalde kant van het lichaam iets mis is. Zo kunnen patiënten geheel niet op de hoogte zijn dat de helft van hun lichaam is verlamd. Dit wordt getest door de patiënt te vragen wat er mis is met die helft van hun lichaam.

Constructieve taken worden getest door de patiënt complexe figuren te laten tekenen.

Taakbeheer en frontale signalen worden getest. Bij frontaal kwab dysfuncties kan het zijn dat de patiënt moeite heeft over te gaan van de ene opdracht op de andere opdracht bij zich herhalende patronen. Dit wordt getest door de patiënt een herhaald patroon uit te laten tekenen of in een patroon met de handen op de tafel te laten tikken. De frontale signalen worden getest aan de hand van verschillende reflexen. Dit wordt later besproken.

Logica en abstractie worden getest door de patiënt een simpel raadsel op te laten lossen. Er wordt getest op hallucinaties en delusies door de patiënt te vragen of hij/zij stemmen hoort of voelt dat iemand hem volgt of hem in te gaten houdt, etc.

Tot slot wordt de stemming van de patiënt getest door te letten op tekenen van depressie, manie, etc.

  1. Hersenzenuwen

Wanneer hoofdpijn, lethargie, instabiliteit en duizeligheid in combinatie voorkomen is er reden om hersenstam problematiek te overwegen. Hierbij is het van belang de hersenzenuwen te testen. Dit kan gedaan worden aan de hand van de functie van de hersenzenuwen.

Motorisch onderzoek

Hierbij zijn er verschillende stappen die gevolgd moeten worden. Zo moet je eerst observeren, dan inspecteren, dan palperen, dan de spiertonus testen, dan de functies testen en uiteindelijk de kracht testen van individuele spiergroepen.

  1. Reflexen

Hierbij moet er gekeken worden naar de “diepe peesreflexen”en de plantaire reactie. Bij de diepe peesreflexen kan er gedacht worden aan de triceps-, biceps- en achillespeesreflex. Voor de plantaire respons moet men met behulp van een voorwerp langs de onderkant van de voet gaan. Een normale reactie vertoont een flexie van de voet (en de grote teen), terwijl er bij een positieve test van Babinski juist extensie wordt vertoond.

  1. Coördinatie en looppatroon

Voor de coördinatie wordt er gekeken of iemand zijn vinger naar zijn neus kan halen en vervolgens weer naar de vinger van de arts kan brengen zonder een scheef patroon weer te geven.

  1. Sensorische examinatie

Hierbij kijkt men naar de pijnsensatie, de temperatuursensatie, de vibratietast, en de propriocepsis

Neurologisch onderzoek als flexibel instrument

Het is bijna nooit mogelijk om alle onderdelen uit te voeren voor een neurologisch onderzoek. Wanneer een patiënt één van de proeven niet kan uitvoeren, bijvoorbeeld wanneer de patiënt geen goede aandacht kan leveren, zal het ook moeilijk zijn om aparte spiergroepen aan te spannen. Wanneer iemand beschadigingen heeft in het taalgebied kun je de patiënt vragen of hij de vragen kan beantwoorden met ja of nee. Voor mensen die doof zijn moet je de verschillende opdrachten opschrijven, zodat deze het ook zullen begrijpen.

Hoe werkt coma-onderzoek?

Het onderzoek is korter dan bij een patiënt die wakker is. Hierdoor kan het onderzoek snel uitgevoerd worden

  1. Mentale status

Coma is het verschijnsel waarbij de patiënt niet reageert op bepaalde activiteiten en waarbij de ogen gesloten zijn. Er zijn verschillende niveaus voor bewustzijn tussen een coma en de volledig alerte staat. Soms is het moeilijk om het bewustzijn te testen, doordat de reticulaire formatie van de hersenstam is aangetast. Soms kan het zo zijn dat er verschijnselen zijn voor een coma maar dat de patiënt niet in een coma ligt. Dit is aanwezig bij akinetisch mutisme. Hierbij is de patiënt ziekelijk besluiteloos. Hierbij zijn de ogen open en er kunnen nog bepaalde bewegingen gemaakt worden. Bij het locked-in-syndroom is het bewustzijn en de sensatie normaal, maar de patiënt is niet in staat om te bewegen door een laesie aan de motorische banen van de hersenstem. Soms kan er nog geknipperd worden met de oogleden bij het locked-in-syndroom.

  1. Hersenzenuwen

Belangrijk om te onderzoeken zijn de volgende hersenzenuwen:

  • CN II: hierbij moet er gekeken of beide ogen geen papiloedeem vertonen. Wanneer het visuele veld is uitgeschakeld moet je gebruik maken van de knipogen-op-gevaar test. Bij deze test knipoogt een normaal persoon wanneer de arts met zijn vinger richting het oog gaat.

  • CNIII: Hierbij wordt er gekeken naar de pupilreflexen. Asymmetrische niet-reagerende ogen kunnen duiden op aantastingen van de middenhersenen.

  • CN V, VII: hierbij wordt er gekeken naar de gezichtssymmetrie en of er asymmetrische spontane veranderingen ontstaan in het gezicht. Ook wordt er gekeken naar de cornea reflex, die in hoofdstuk 12 verder wordt besproken.

  • CN IX, X: hierbij wordt de kotsreflex getest door de posteriore farynxwand aan te raken.

  1. Sensorisch en motorisch onderzoek

Er moet gekeken worden naar spontane bewegingen van alle extremiteiten. De spiertonus moet getest worden en elke ledemaat moet getest worden op het verlies van sensibiliteit. Pijnvolle stimuli moeten alleen gebruikt worden wanneer het echt nodig is.

  1. Reflexen

Hierbij moet er gekeken worden naar de “diepe peesreflexen” en de plantaire respons. Wanneer er reflexen in het postuur verschijnen, kan dit op schade wijze van de bovenste motorneuronbanen. De reflexen zijn afhankelijk van de hersenstam en het spinale circuit en zijn vaak te zien in ernstige laesies als een coma. In de onderste extremiteit kan er soms een flexie reflex ontstaan. Deze wordt de triple flexion genoemd, omdat er flexie van de knie en de dij aanwezig zijn en dorsiflexie van de enkel.

  1. Coördinatie en looppatroon

Deze zijn vaak niet te testen tijdens een neurologisch onderzoek.

Wat is hersendood zijn?

Hersendood is een irreversibel gebrek aan hersenfunctie. Om er zeker van te zijn of er geen hersenstamfunctie meer is, moet men gebruik maken van de calorische test en de apneu test. Bij de apneu test wordt er gekeken of er een gebrek aan spontane respiratie optreedt zonder een ventilator. Mensen met postering reflexen, dus reflexen van het postuur, worden niet als hersendood gezien, terwijl mensen met alleen een peesreflex en een triple flexion wel hersendood kunnen zijn.

Conversiestoornis, malingering en gerelateerde aandoeningen

Conversion disorder (conversiestoornis): psychiatrische ziekte kan zorgen voor verlies van sensorische en motorische gebieden zonder de aanwezigheid van een laesie in het zenuwstelsel.

Somatization disorder (somatisatie): patiënten hebben verschillen somatische klachten die over een tijd veranderen. Bij een andere klasse van verstoringen hebben patiënten vaak controle over hun bewustzijn en andere symptomen, maar wordt het gebruikt voor andere doeleinden. Een ‘factitious disorder’ is een verstoring bij de patiënt die ervoor zorgt dat een patiënt neppe klachten gaat weergeven, omdat hij of zij het leuk vindt op in de patiëntrol te zitten. Om erachter te komen of de patiënt deze verstoringen bevatten of juist wel een echte laesie in het centrale zenuwstelsel, kan met de volgende technieken gebruiken:

  1. Handdropping test in pseudocoma: wanneer een patiënt in een echte coma zit en zijn hand ligt boven zijn gezicht, moet de hand het gezicht aanraken op zijn weg naar beneden.

  2. Saccadic eye movements in pseudocoma: Ssaccades zijn snelle bewegingen van ogen. Deze komen in een coma niet voor, maar wel wanneer een patiënt locked in is of een slaapparalyse bevat.

  3. Hoover test: bij unilaterale zwakte in het been moet de contralaterale m. gastrocnemius worden gepalpeerd terwijl de patiënt zijn aangetaste been probeert op te tillen van het bed.

  4. Unconscious movements: dit kan optreden wanneer de patiënten in hun slaap een ledemaat bewegen terwijl ze verplaatst worden naar bijvoorbeeld een ander bed.

 

Hoe werkt neurologische onderzoek?

 

Onderzoekstechnieken en tests

Het neurologische onderzoek test de verschillende functies van iemands lichaam.

Voor het neurologische onderzoek wordt er gekeken naar 6 verschillende onderdelen:

  1. Mentale status

Hierbij wordt er gekeken naar het niveau van alertheid door de patiënt een woord te laten spellen en weer achterstevoren te laten spellen. Normaal gesproken kan iemand woorden met 6 of meer letters in normale volgorde spellen en woorden van 4 of meer letter achterwaarts spellen. Ook is dit natuurlijk afhankelijk van de leeftijd en het educatieniveau van de patiënt. Hierbij wordt er gekeken of er een bilaterale laesie aanwezig is van de thalamus of de cerebrale hemisferen.

Er wordt ook gekeken naar de oriëntatie. Hierbij wordt er gevraagd naar iemands naam, locatie en de datum van de dag. Hierbij wordt het lange termijn geheugen getest. Wanneer een patiënt beperkt antwoord kan geven op deze vragen, dan is het van belang dat de onderzoeker het exacte antwoord van de patiënt noteert.

Het geheugen van de patiënt wordt getest. Dit wordt gedaan door de patiënt een kort verhaaltje te vertellen en aan de patiënt te vragen of deze het verhaal zou willen herhalen na ongeveer 3-5 minuten. Zorg dat de patiënt tussentijds een beetje afgeleid is. Op deze wijze wordt het korte termijn geheugen van de patiënt getest. Om het lange termijn geheugen te testen moet er gevraagd worden naar persoonlijke ervaringen van langer terug.

De taal wordt getest. Bij spontane spraak wordt gelet op hoe vloeiend de patiënt spreekt, de lengte van zinnen en de hoeveelheid spontane spraak. Modulatie van toon en parafrase fouten, net als neologismen (zelf bedachte woorden) moeten worden opgevangen. Daarnaast moet gelet worden op het begrip van de patiënt. Snapt de patiënt simpele vragen en daarnaast moet ook gekeken worden of begrip heeft van grammaticale structuren.

Het rekenen, recht-links verwarring, vinger agnosie en agrafie worden getest. Indien al deze componenten zijn verstoord, wordt er gesproken van het Gerstmann’s syndroom. Het rekenen wordt getest door de patiënt simpele rekensommen op te laten lossen. De patiënt wordt gevraagd links en rechts aan te geven. De patiënt wordt gevraagd zijn vingers te benoemen. De patiënt wordt gevraagd zijn/haar naam op te schrijven en een zin, die wordt gegeven door de onderzoeker.

Er wordt gekeken of er sprake is van apraxie. Er is sprake van apraxie indien iemand niet in staat is een motorische opdracht uit te voeren. Dit kan getest worden door de patiënt te vragen een alledaagse bezigheid voor te wenden, zoals het kammen van zijn/haar haar.

“Neglect” is wanneer een patiënt een kant van het lichaam geheel negeert, omdat de patiënt zich niet meer bewust is van dit deel. Dit kan getest worden door te kijken hoe de patiënt andere opdrachten uitvoert, bijvoorbeeld wanneer de patiënt tijdens het schrijven één kant van de pagina onbenut laat. Er moet ook gekeken worden naar anosognosia. Dit is wanneer de patiënt ook ontkent dat er met een bepaalde kant van het lichaam iets mis is. Zo kunnen patiënten geheel niet op de hoogte zijn dat de helft van hun lichaam is verlamd. Dit wordt getest door de patiënt te vragen wat er mis is met die helft van hun lichaam.

Constructieve taken worden getest door de patiënt complexe figuren te laten tekenen.

Taakbeheer en frontale signalen worden getest. Bij frontaal kwab dysfuncties kan het zijn dat de patiënt moeite heeft over te gaan van de ene opdracht op de andere opdracht bij zich herhalende patronen. Dit wordt getest door de patiënt een herhaald patroon uit te laten tekenen of in een patroon met de handen op de tafel te laten tikken. De frontale signalen worden getest aan de hand van verschillende reflexen. Dit wordt later besproken.

Logica en abstractie worden getest door de patiënt een simpel raadsel op te laten lossen. Er wordt getest op hallucinaties en delusies door de patiënt te vragen of hij/zij stemmen hoort of voelt dat iemand hem volgt of hem in te gaten houdt, etc.

Tot slot wordt de stemming van de patiënt getest door te letten op tekenen van depressie, manie, etc.

  1. Hersenzenuwen

Wanneer hoofdpijn, lethargie, instabiliteit en duizeligheid in combinatie voorkomen is er reden om hersenstam problematiek te overwegen. Hierbij is het van belang de hersenzenuwen te testen. Dit kan gedaan worden aan de hand van de functie van de hersenzenuwen.

 

Hoe zijn de hersenen en hun omgeving gevormd? - Chapter 5

 

 

Hoe is de schedel opgebouwd?

De hersenen zijn omgeven door meerdere structuren ter bescherming. Van buiten naar binnen zijn dit de scalp, het cranium (schedel) en de meningen (hersenvliezen).

De scalp (hoofdhuid) bestaat uit vijf lagen op de schedel. Je kunt deze onthouden met het ezelsbruggetje SCALP.

  • Skin

  • Connective tissue

  • Aponeurotica galea

  • Loose areolar connective tissue

  • Pericranium

Onder de scalp ligt het cranium. In het cranium zitten vele verschillende foramina voor verschillende structuren. Het belangrijkste is het foramen magnum. Deze ligt in de basis van de schedel. Het is waar de medulla overgaat in het ruggenmerg (cervicomedullary junction).

Het cranium is verder in het binnenste oppervlak verdeeld in compartimenten. Dit zijn de anteriore, mediale en posteriore fossae.

In de anteriore fossa liggen de frontale kwabben. Het wordt gescheiden van de mediale fossa door de lesser wing of the os sphenoidale.

In de mediale fossa liggen de temporale kwabben. Het wordt gescheiden van de posteriore fossa door de petrous ridge of the os temporale en door het tentorium cerebelli. Het tentorium cerebelli scheidt de posteriore fossa van de rest. Het splitst het achterste deel in een supratentoriaal en een infratentoriaal deel. Supratentoriaal ligt de occipitaalkwab en infratentoriaal het cerebellum.

De posteriore fossa bevat het cerebellum en de hersenstam.

Onder het cranium liggen de meningen (hersenvliezen). Van binnen naar buiten zijn dit de pia mater, arachnoid mater en dura mater (PAD).

De dura bestaat uit twee harde fibreuze lagen. De buitenste periosteale laag die aan het cranium vastzit en de binnenste meningeale laag. Soms zitten deze lagen niet aan elkaar vast, zoals bij het falx cerebri, de invouwing tussen de twee hemisferen, en bij het tentorium cerebelli, dat over het cerebellum en onder de occipitaal kwab ligt. Door een opening in het tentorium cerebelli, de tentorial incisura, loopt het middenbrein.

Het arachnoid ligt aan de meningeale laag van de dura. Tussen het arachnoid en de pia mater ligt cerebrospinale vloeistof.

De pia ligt op de hersenen en loopt mee met alle contouren. Ook vormt het een perivasculaire ruimte om de bloedvaten die de hersenen in gaan, en gaat vervolgens over in het bloedvat.

De epidurale ruimte is een potentiële ruimte die tussen het cranium en de dura kan ontstaan door een bloeding van de a. meningea media die tussen het cranium en de dura mater loopt. In fysiologische toestand is de epidurale ruimte afwezig.

De subdurale ruimte is een potentiële ruimte tussen de dura en het arachnoid. Ook deze kan ontstaan door een bloeding. Dit is altijd een veneuze bloeding, omdat hier alleen maar venen lopen die naar de veneuze sinussen gaan. Deze veneuze sinussen komen uit in de vv. jugularis interna. Ook deze subdurale ruimte is normaliter niet aanwezig.

De subarachnoïdale ruimte is een fysiologische ruimte gevuld met cerebrospinale vloeistof tussen het arachnoid en de pia mater. In deze ruimte lopen de cerebrale arteriën. Een subarachnoïdale bloeding is dus ook vaak arterieel.

De meningen lopen mee met het ruggenmerg naar beneden. Er zit dan wel een extra laagje epiduraal vet tussen de dura en de wervelkolom.

Wat zijn ventrikels en de cerebrospinale vloeistof?

De ventrikels ontstaan als holtes tijdens de embryonale ontwikkeling. Er zijn er vier: de laterale (eerste en tweede) ventrikels en de derde en vierde ventrikel. De ventrikels produceren cerebrospinale vloeistof. Dit gebeurt in de plexus choroideus door het epitheel van de ventrikels, de ependymcellen.

De twee laterale ventrikels liggen elk in een hemisfeer. Ze hebben een C-vormige corpus met een frontale, een temporale en een occipitale hoorn. Het atrium verbindt de corpus, de posterior en de inferior hoorn. De laterale ventrikels zitten vast aan het derde ventrikel door het interventriculaire foramen van Monro.

Het derde ventrikel ligt in het diencephalon. De wanden bestaan uit de thalamus en de hypothalamus. Het derde ventrikel zit vast aan het vierde ventrikel door de aquaductus cerebralis ook wel het aquaductus van Sylvius genoemd.

Het vierde ventrikel ligt tussen de pons en medulla als vloer en het cerebellum als dak. De cerebrospinale vloeistof verlaat het ventrikelsysteem via de vierde ventrikel. In de vierde ventrikel zitten drie foramina: de Laterale foramina van Luschka en het Midline foramen van Magendie. De cerebrospinale vloeistof loopt door deze foramina in de subarachnoïdale ruimte langs de hersenen of het ruggenmerg. De cerebrospinale vloeistof wordt vervolgens door de granulationes arachnoideae in de veneuze sinussen en vervolgens in de bloedstroom opgenomen. Een mens heeft op elk moment 150 mL cerebrospinale vloeistof in zijn lichaam.

Op bepaalde plaatsen zijn er grote meren met cerebrospinale vloeistof, de cisternen.

De perimesencephalische cisternen bestaan uit een drietal cisternen: de ambient cisterne lateraal ten opzichte van het middenbrein; de quadrigeminal cisterne posterior ten opzichte van het middenbrein en de interpenduncular cisterne tussen de cerebrale pedunculi, ventraal van het middenbrein. De prepontine cisterne ligt ventraal van de pons. De cisterna magna ligt onder het cerebellum bij het foramen magnum. De lumbale cisterne ligt in het lumbale deel van de wervelkolom. Hier worden lumbaalpuncties afgenomen.

Wat is de bloed-hersen barrière?

De capillairen in de hersenen zijn niet gefenestreerd, maar zijn verbonden middels tight junctions. Dit betekent dat als een stof de hersenen in wil, het door actief transport door het endotheel getransporteerd moet worden. Dit gespecialiseerde endotheel vormt samen met de uitlopers van astrocyten de bloed-hersen barrière. Deze barrière beschermt de hersenen, ook tegen de fluctuaties van de chemische samenstelling van het bloed. Er is ook een bloed-CSF barrière tussen de capillairen van de plexus choroideus en de CSF. Via de arachnoid villus cells kan het CSF wel vrij de venen in stromen.

Er zijn enkele plaatsen in de hersenen waar de bloed-hersen barrière niet aanwezig is. Dit is onder andere in de ‘median eminence’ en de neurohypofyse (beide in de hypofyse).

De area postrema, ook wel de chemotactic trigger zone, is een plaats in de medulla langs het vierde ventrikel. Deze zone detecteert toxines, waardoor je gaat overgeven.

Vasogenic edema ontstaat doordat er te veel extracellulaire vloeistof in de hersenen is. Cytotoxic edema ontstaat doordat er te veel intracellulaire vloeistof in de hersenen is.

KCC 5.2 Intracraniale massale lesies

Intracranial mass lesions zijn structuren die niet thuishoren in het cranium (tumor, bloeding). Hierdoor ontstaan compressie van delen van de hersenen, een verhoogde intracraniale druk en hernia (verschuiving van structuren naar een ander compartiment). De gevolgen zijn neurologische beschadiging en/of uitval.

Een mass effect is een verstoring van de normale hersenlokalisatie. Als het klein is, kan het voorkomen dat er geen symptomen ontstaan. Grote mass effects veroorzaken echter neurologische uitval. Bloedvaten kunnen ook beschadigd raken en voor hoofdpijn, ischemie of een bloeding zorgen.

Oedeem in de hersenen kan ontstaan door teveel vloeistof in de extracellulaire ruimte. Een waterhoofd (hydrocephalus) kan ontstaan doordat de cerebrospinale vloeistof niet weg kan. Er kunnen convulsies ontstaan door verstoorde actiepotentialen.

Midline shift ontstaat als de massa zo groot wordt dat de middenlijn van de hersenen verschuift, waardoor de hersenstam gerekt wordt. Dit leidt tot een verminderd bewustzijn en coma, met uiteindelijk de dood tot gevolg.

KCC 5.4 Herniaire syndromen

Een hernia van de hersenen ontstaat wanneer een hersenstructuur in een ander compartiment wordt gedrukt. Bij elke soort verdrukking ontstaan specifieke verschijnselen.

Transtentoriale herniatie

Een transtentorial herniation ontstaat als de mediale temporaal kwab inferior door de tentoriale opening wordt gedrukt. Dit kan leiden tot:

  • Blown pupil: een gedilateerde, niet reagerende pupil door compressie van CN III. Later ook verminderde oogbewegingen. Vaak is dit ipsilateraal.

  • Hemiplegia: verlamming van een helft van het lichaam, door verdrukking van de tractus corticospinalis. Dit is vaak contralateraal, tenzij de verdrukking helemaal naar de andere kant is, zodat de hersenstam tegen de andere zijde van het tentorium cerebelli wordt verdrukt.

  • Coma: dit ontstaat door verplaatsing van de reticulaire formatie.

Centrale herniatie

Central herniation is verdrukking van de hersenstam naar inferior. Dit ontstaat door een verhoogde intracraniale druk. De CN VI wordt gerekt en er ontstaat een lateral rectus palsy (parese van de m. rectus lateralis) in het oog. Hoge drukken kunnen ook leiden tot een transtentoriale herniatie. Extreem hoge drukken kunnen de medulla door het foramen magnum drukken.

Als er tonsillar herniation van de cerebellaire lobben ontstaat, wordt de medulla verdrukt. De gevolgen hiervan zijn ademhalingsstilstand, instabiele bloeddruk en uiteindelijk de dood.

Subfalcine herniatie

Bij een subfalcine herniation schuiven hersenstructuren van de links naar rechts of andersom onder het falx cerebri door. Vaak ontstaan er hierbij geen symptomen. Als de aa. cerebri anterior verdrukt worden ontstaat er een infarct.

KCC 5.6 Intracraniale bloeding

Een intracraniële bloeding kan traumatisch en niet-traumatisch ontstaan.

Epiduraal hematoom (EDH) ontstaat in de epidurale ruimte, door een ruptuur van de a. meningea media. Meestal ontstaat dit door een fractuur van het os temporale. Op beeldvormende technieken is een biconvex hematoom te zien. Deze ophoping van bloed in de epidurale ruimte kan uiteindelijk leiden tot verdrukking van hersenweefsel en neurologische symptomen, waardoor er vaak een wekadvies wordt gegeven aan iemand met een hoofdtrauma.

Subduraal hematoom (SDH) ontstaat in de subdurale ruime door een ruptuur van venen. Op beeldvormende technieken is een halvemaanvormig (biconcaaf) hematoom te zien. Dit is vaak chronisch bij ouderen, doordat de hersenen kleiner worden en er meer rek op de venen is. Het kan ook acuut ontstaan door een enorm harde klap, wat vaak ook met een hersenschudding gepaard gaat.

Subarachnoidaal hematoom (SAH) ontstaat als bloed in de subarachnoïdale ruimte komt, wat gezien kan worden op een CT doordat er bloed in de culi zit.

Dit kan niet-traumatisch ontstaan door een ruptuur van een aneurysma. De patiënt voelt dan een plotselinge catastrofische hoofdpijn. Sacculaire aneurysma’s zijn een soort zakjes aan de vaatwand. Een fusiform aneurysma is een gewoon aneurysma. Deze knappen niet vaak, maar kunnen wel een mass effect veroorzaken.

Het kan ook traumatisch ontstaan, doordat bloedvaten door een hersenschudding kapot gaan. Bedenk dat er hierbij ook bloed in de CSF gevonden kan worden.

Traumatische intraparenchymale hematoom (IPH) ontstaat door een hersenschudding, waar de hersenen vlakbij een botstuk zitten. Het hersenparenchym is dan aangedaan.

Niet-traumatische intraparenchymale hematoom (IPH) ontstaat door hypertensie, tumor, na een infarct en nog veel meer. Het vaakst ontstaat het door hypertensie. Een hematoom kan ook in een kwab voorkomen. Bepaalde vasculaire vervormingen kunnen ook intracraniale hematomen veroorzaken. Dit zijn voornamelijk arterioveneuze malformaties en cavernomas.

Extracraniale hematoom kan ontstaan in het binnenoor, in de scalp of tussen het externe periosteum en de schedel.

KCC 5.7 Hydrocephalus

Een hydrocephalus is een waterhoofd. Er is dan te veel cerebrospinale vloeistof in het hoofd, omdat dit niet weg kan, er te veel wordt geproduceerd of omdat er te weinig resorptie is. Hydrocephalus wordt verdeelt in communicating hydrocephalus (buiten het ventrikelsysteem) en noncommunicating hydrocephalus (obstructie in het ventrikelsysteem). De symptomen zijn hetzelfde als bij een verhoogde intracraniale druk. Er zijn verschillende behandelingen, zoals een externe ventriculaire drain, endoscopische neurochirurgie en third ventriculostomy.

Hydrocephalus kan ook ontstaan doordat, voornamelijk bij ouderen, de ventrikels vergroot zijn.

KCC 5.10 Lumbaalpunctie

Bij een lumbaal punctie wordt er in de lumbale cisterne geprikt. Hier kan er een cerebrospinaal monster genomen worden, de druk gemeten worden, of cerebrospinale vloeistof verwijderd worden. Ook kunnen op deze manier medicijnen worden toegediend. Bij de punctie wordt er een holle naald in de wervelkolom geprikt ter hoogte van L4/L5. Hier zit er geen vast ruggenmerg, maar is de cauda equina aanwezig, waardoor er veilig in de subarachnoidale ruimte geprikt kan worden.

De druk wordt eerst gemeten en vervolgens worden er monsters genomen. Als er rode bloedcellen in zitten kan dit komen door een subarachnoïdale bloeding of door een traumatic trap (bloedvaten die aangeprikt waren tijdens de punctie).

Hoe verlopen de tractus corticospinalis (piramidebaan) en andere motorische pathways? - Chapter 6

 

Wat is de motor cortex, sensorische cortex en somatotopische organisatie?

De primaire motor cortex bevindt zich in de precentrale gyrus, terwijl de primaire somatosensorische cortex juist in de postcentrale gyrus zit. Laesies in de geassocieerde sensorische en motorische cortex zullen niet voor erge tekortkomingen zorgen bij de basisbewegingen en de basissensatie. De primaire en somatosensorische cortexen vertonen een somatotopische organisatie. Hiermee wordt er bedoeld dat elk deel van de cortex overeenkomt met een deel van het lichaam. Het wordt weergegeven aan de hand van de homunculus.

Basis anatomie van het ruggenmerg

Op de afbeelding hiernaast kan je zien hoe een deel van het ruggenmerg eruit ziet. De grijze stof bevat een dorsale hoorn voor sensorische processen, een intermediaire zone voor de interneuronen en bepaalde gespecialiseerde kernen en een ventrale hoorn met motorneuronen. De witte stof bestaat een dorsale kolom, een laterale kolom en een ventrale kolom. In het cervicale gebied is de witte stof het dikst. Het sacrale deel bevat de meeste grijze stof. Het ruggenmerg bevat twee verdikkingen:

  1. Cervicale verdikking

  2. Lumbosacrale verdikking

Deze zorgen voor de zenuwplexussen voor de armen en benen.

Bloedvoorziening van het ruggenmerg

De arteriële bloedvoorziening wordt verzorgd door:

  • a. spinalis anterior: takken van de a. vertebralis die 2/3e van het voorste deel van het ruggenmerg verzorgt. Dit is inclusief de voorhoorn en anteriore en laterale witte stof kolommen.

  • a. spinalis posterior: takken van de a. vertebralis of a. cerebelli inferior posterior. Het voorziet de posteriore witte stof kolommen en een deel van de achterhoorn van bloed.

  • Spinale arteriële plexus: deze wordt gevormd door takken van de a. spinalis anterior en posterior.

  • a. radicularis: dit zijn 6 arteriën die op verschillende niveaus uit de aorta ontspringen en bloed toevoeren naar het ruggenmerg. De belangrijkste is de grote radiculaire arterie van Adamkiewicz. Deze ontspringt op het niveau van Th9-Th12 en voorziet het lumbale en sacrale deel van het ruggenmerg van bloed.

De veneuze terugvoer vindt plaats door een drainage naar een plexus van venen die in de epidurale ruimte zit. Dit is de zogenaamde ‘Batson’s plexus’.

Th4-Th8 ligt tussen de vertebrale en lumbale arteriële bloedvoorziening, waardoor het een kwetsbare zone is van relatief lage perfusie.

Hoofdorganisatie van de motorische systemen

In de afbeelding kan je zien dat er een groot netwerk nodig is om bepaalde bewegingen uit te voeren. Zo zorgen het cerebellum en de basale ganglia voor een feedbackloop die via de thalamus informatie verschaft aan de cerebrale cortex. Verschillende descenderende motor systemen ontspringen uit de cortex en de hersenstam, de zogenaamde laterale en mediale motorsystemen. De twee laterale motorsystemen zijn het tractus corticospinalis lateralis en de tractus rubrospinalis. Deze trajecten controleren de bewegingen van de contralaterale extremiteiten. Het corticospinale traject is voornamelijk van belang bij de snelle bewegingen bij individuele gewrichten.

Er bestaan hiernaast 4 mediale systemen:

  1. Anterior corticospinale traject

  2. Vestibulospinale traject

  3. Reticulospinale traject

  4. Tectospinale traject

Deze systemen zijn van belang bij proximaal axiale en gordelspieren die de balans, oriënterende bewegingen van het hoofd en de bewegingen voor de looppas coördineren.

De tractus rubrospinalis is klein en zou een rol spelen bij het overnemen van verschillende functies na schade aan de tractus corticospinalis. Hiernaast zou het ook een rol spelen bij flexie van de bovenste extremiteiten.

Wat is het laterale corticospinale traject?

De helft van de vezels ontspringen in de primaire motorische cortex (Brodmann’s area 4) en de andere helft in de premotorische en supplementaire motorgebieden van de pariëtale kwab. De neuronen van de primaire motor cortex die bij het laterale corticospinale traject horen liggen in laag 5. (afb. 6.9 blz. 233 van belang!).

De piramidecellen van deze laag vormen direct synapsen met de motor neuronen in de ventrale hoorn en met de spinale interneuronen. De grootste piramide cellen worden de Betz cellen genoemd. Axonen van de cerebrale cortex gaan de bovenste delen van de cerebrale witte stof, de corona radiata, binnen en descenderen richting de capsula interna. De thalamus en de nucleus caudatus liggen mediaal ten opzichte van de capsula interna.

De capsula interna bestaat uit 3 delen:

  1. Anterior ledemaat: dit scheidt het hoofd van de nucleus caudatus van de globus pallidus en putamen.

  2. Posterior ledemaat: dit scheidt de thalamus van de globus pallidus en het putamen. Hier bevindt zich ook de tractus corticospinalis (piramidebaan).

  3. Genu: Dit zit bij de overgang van het anteriore deel naar het posteriore deel. Het bevindt zich op het niveau van het foramen van Monro.

De somatotopische map bevindt zich ook in de capsula interna. De vezels voor het gezicht zullen dan anterior liggen, terwijl die voor de extremiteiten meer posterior zullen liggen. Vezels die van de cortex naar de hersenstam gaan worden corticolbulbulair genoemd.

Na de capsula interna gaan de vezels verder naar de cerebrale pedunculi die in de middenhersenen liggen. De witte stof is gelokaliseerd in het ventrale deel van de cerebrale pedunculi en wordt de basis pedunculi genoemd. Het middelste 1/3 deel van de basis pedunculi bevat de corticobulbare en corticospinale vezels met de axonen voor het gezicht, de arm en de benen. Hierna loopt de tractus corticospinalis verder via de ventrale pons waarbij er op de medulla de medullaire piramides worden gevormd. De overgang van medulla naar ruggenmerg wordt de ‘cervicomedullaire junction’ genoemd die op het niveau van het foramen magnum ligt.

85% van de vezels kruisen (zorgen voor contralaterale innervatie van de spieren), terwijl de overige 15% recht naar beneden lopen (deze zorgen vooral voor de aansturing van de axiale spiergroepen, dus van de romp). De verschillende trajecten kan je zien op blz. 236/237 in het boek.

Wat is het autonome zenuwstelsel?

Dit systeem heeft de controle over de meer automatische en viscerale functies. In autonome efferenten is er een perifere synaps in het ganglion aanwezig tussen het centrale zenuwstelsel en het doelorgaan. Het bevat twee divisies:

  1. Sympathische/thoracolumbale: Het ontspringt uit T1 – L2 en is van belang voor de fight-or-flight response.

  2. Parasympatische/craniosacrale: Deze ontspringt uit S2-S4 en de hersenzenuwen en is van belang voor de rest-or-digest reactie.

Als derde systeem wordt er vaak over het enterische zenuwstelsel gesproken. Deze bevat de neurale plexus die van belang is voor de peristaltiek en de gastro-intestinale secreties.

KCC 6.1: Bovenste motorneuron versus lagere motorneuron laesies

Symptomen van laesies aan de perifere zenuwen zijn spierzwakte, atrofie, fasciculaties, verlaagde tonus en hyporeflexia. Fasciculaties zijn abnormale spiersamentrekkingen die door spontane activiteit van groepen spiercellen worden veroorzaakt. Symptomen van laesies aan centrale zenuwen zijn spierzwakte en een combinatie van een verhoogde tonus en hyperreflexie. Vaak wordt er gerefereerd naar spasticiteit. Normaliter heeft het centrale zenuwstelsel een enigszins dempende werking op de uit het ruggenmerg tredende, perifere zenuw. Als deze demping wegvalt door een laesie in het ruggenmerg zal er hypperreflexie en spasticiteit van de spiergroepen onder de laesie optreden.

KCC 6.2 Voorwaarden om de zwakte te beschrijven

Term voor Ernst

Definitie

Paresis

Zwakte

-plegia

Geen beweging

Paralysis

Geen beweging

Palsy

Term voor zwakte of geen beweging

Term voor locatie

Definitie

Hemi-

Één kant van het lichaam

Para-

Beide benen

Mono-

Één ledemaat

di-

Beide kanten van het lichaam zijn hetzelfde aangetast

Quadri- of tetra-

Alle vier de ledematen

KCC 6.3 Zwaktepatronen en lokalisatie

Zwakte van een gedeelte van het lichaam kan ontstaan door een laesie of disfunctie op elk niveau van het motorische systeem. In het boek worden verschillende voorbeelden gegeven.

 

Wat is de motor cortex, sensorische cortex en somatotopische organisatie?

 

De primaire motor cortex bevindt zich in de precentrale gyrus, terwijl de primaire somatosensorische cortex juist in de postcentrale gyrus zit. Laesies in de geassocieerde sensorische en motorische cortex zullen niet voor erge tekortkomingen zorgen bij de basisbewegingen en de basissensatie. De primaire en somatosensorische cortexen vertonen een somatotopische organisatie. Hiermee wordt er bedoeld dat elk deel van de cortex overeenkomt met een deel van het lichaam. Het wordt weergegeven aan de hand van de homunculus.

 

Hoe verlopen somatosensorische pathways? - Chapter 7

 

 

Wat is somasensorie?

Somatosensorisch staat voor de lichamelijke sensaties als aanraking, pijn, temperatuur, vibratie en proprioceptie. Er bestaan twee pathways:

  1. Posterior column-medial lemniscal pathway: voor proprioceptie, vibratiegevoel, fijne tast (gnostisch)

  2. Anterolateral pathways: voor pijn, temperatuur en grove tast (vitaal)

Sensorische neuronale cellichamen liggen in de dorsale wortel ganglia. Een perifeer gebied dat geïnnerveerd wordt door sensorische vezels van één zenuwwortelniveau wordt een dermatoom genoemd.

Wat is het posterior kolom-mediale lemniscale pathway?

Gemyeliniseerde axonen gaan van het ruggenmerg via de ipsilaterale posteriore kolom naar de posterior kernen in de medulla. De fasciculus gracilis bevat informatie voor de benen en de lagere romp terwijl de meer laterale fasciculus cuneatus informatie bevat voor het rompgedeelte boven Th6. De 1e orde sensorische neuronen in deze fasciculi synapteren op 2e orde neuronen in de nucleus gracilis en nucleus cuneatus. Deze vormen de mediale lemniscus aan de andere kant van de medulla. Hier heeft dus de kruising plaatsgevonden. De axonen van de mediale lemniscus synapteren in de ventrale nucleus posterior lateralis van de thalamus waarna de informatie terecht zal komen in de primaire somatosensorische cortex.

Wat zijn spinothalamuskanaal en andere anterolaterale pathways?

De ongemyeliniseerde axonen zorgen voor informatie over pijn en temperatuur en gaan het ruggenmerg binnen via de dorsale wortels. De eerste synapsen vinden direct plaats in de grijze stof van het ruggenmerg in de dorsale hoorn. Axonen van de 2e orde sensorische neuronen kruisen over naar de anteriore commissuur in het ruggenmerg om in de anterolaterale witte stof richting de hersenen te gaan. Er zijn 2-3 spinale segmenten nodig voor de vezels om de andere kant te bereiken. De anterolaterale pathways bestaat uit drie trajecten:

  1. Spinothalamische traject: is voor de locatie en intensiteit van de stimulus voor pijn en temperatuur.

  2. Spinoreticulaire traject

  3. Spinomesencephalische traject

Belangrijk voor het spinothalamische traject is de VPL, de ventrale posterior laterale nucleus, van de thalamus. Vanaf de VPL reist informatie in de tractus spinothalamicus naar de primaire somatosensorische cortex. Er zijn ook delen van het spinothalamische traject die naar andere kernen lopen. Dit zijn de intralaminaire thalamische kernen en de mediodorsale kernen.

Wat is de somatosensorische cortex?

Van de thalamische VPL en VPM kernen wordt somatosensorische informatie naar de cortex in de postcentrale gyrus gebracht. Het bevat Brodmann’s area 3,1 en 2. Informatie van de primaire cortex wordt naar de secundaire somatosensorische cortex gebracht die in de Sylvian fissure ligt. In het posteriore gedeelte van de pariëtale kwab liggen Brodmann’s areas 5 en 7.

Centrale modulatie van pijn

Gate control theory: sensorische input van grote diameter, geen pijn veroorzakende A-β-vezels verlagen de pijn transmissie door de dorsale hoorn. De periaqueductale grijze stof ontvangt input uit de hypothalamus, amygdala en cortex en inhibeert de pijn transmissie in de dorsale hoorn via de rostral ventral medulla.

De thalamus

De thalamus is een belangrijk processing systeem in het centrum van de hersenen. De kernen in de thalamus ontvangen reciprocale feedback connecties uit de corticale gebieden. Het is een deel van het diencephalon. De thalamus kan onderverdeeld worden in de mediale nucleusgroep, de laterale nucleusgroep, en de anteriore nucleusgroep door een Y-vormige witte stof die de internal medullary lamina wordt genoemd. Er zijn ook kernen in de lamina, de intralaminaire kernen.

De midline thalamic nuclei zijn een dunne collectie kernen die bij het derde ventrikel liggen. Als laatste bestaat er nog de thalamic reticular nucleus die een lateraal deel van de thalamus vormt.

Relay kernen: Deze kernen zijn het meest aanwezig in de thalamus. De relay kernen voor de sensorische en motorische cortex liggen in de laterale thalamus. Alle sensorische modaliteiten bevatten specifieke relay kernen in het laterale deel van de thalamus, behalve die voor de reuk. Visuele informatie verloopt via de de laterale geniculate kern (LGN) en gehoor via de mediale geniculate kern.

Intralaminaire kernen: Deze liggen in de internal medullary lamina. Ze ontvangen verschillende inputs en hebben verbindingen met de cortex. Ze kunnen onderverdeeld worden in twee functionele gebieden: caudale intralaminaire kernen voor het basale ganglia circuit en de rostral intralaminaire kernen. Ze bevatten een belangrijke rol voor het ascenderende reticulaire activatie systeem. Dit is belangrijk voor de alertheidstatus.

Reticulaire kern: Deze vormt een dunne laag dat lateraal van de rest af ligt en mediaal ten opzichte van de capsula interna. Dit is de enige kern in de thalamus die geen verbindingen met de cortex bevat. Het bevat een gehele inhibitoire populatie van GABA neuronen.

KCC 7.1: Paresthesias

Laesies van de somatosensorische pathways kunnen een abnormaal positief sensorische fenomeen veroorzaken dat paresthesias wordt genoemd. Laesies van de thalamus kunnen voor erge contralaterale pijn zorgen, het zogenaamde Dejerine-Roussy syndroom. Laesies van het cervicale ruggenmerg kunnen samengaan met Lhermitte’s sign, dat bijvoorbeeld vaak aanwezig is bij multiple sclerose. Het zorgt voor een soort elektrische prikkeling die door iemands rug en nek loopt bij het buigen van de nek. Andere benamingen voor sensorische abnormaliteiten zijn dysesthesia, allodynie en hyperpathie.

KCC 7.3: Sensorisch verlies: patronen en lokalisatie

Primaire somatosensorische cortex

Sensorisch verlies komt voor in de contralaterale kant van de laesie. Soms kunnen alle modaliteiten worden bespaard, maar is er wel een patroon aanwezig van corticale sensorisch verlies.

Thalamic Ventral Posterior Lateral (VPL) en ventrale posterior mediale (VPM) nuclei of thalamische somatosensorische stralingen

Sensorisch verlies komt aan de contralaterale kant van de laesie voor. Het sensorische verlies kan men opmerken in de hand, het gezicht en in de voet en minder in de truncus of proximale extremiteiten. Wanneer de capsula interna aangetast wordt, kan er hemiparese of hemianopsie ontstaan.

Laterale pons van laterale medulla

De laesie omvat de anterolaterale pathways en de spinale trigeminale kern. Het zorgt voor verlies van pijn- en temperatuur sensatie in het lichaam aan de andere kant van de laesie en verlies van pijn- en temperatuur sensatie in het gezicht aan de ipsilaterale kant.

Mediale medulla

Hierbij is de mediale lemniscus aangetast, waardoor er contralateraal verlies van vibratie en propriocepsis optreedt.

KCC 7.4: Ruggenmergsyndromen

Transversale laesie

Hierbij zijn alle sensorische en motorische pathways of gedeeltelijk of geheel aangetast. Er is vaak een sensorisch niveau onder de laesie, waarbij alle dermatomen geen sensatie meer vertonen.

Hemicord laesies: Brown-Séquard syndroom

Schade aan het laterale corticospinale traject zorgt voor een ipsilaterale upper motorneuron-type zwakte. Storing van de posterior kolom kan zorgen voor een ipsilateraal verlies van vibratie en propriocepsis. Storing van de anterolaterale systemen zorgen voor contralateraal verlies van pijn en temperatuur sensatie. Dit laatste begint vaak onder de laesie omdat de anterolaterale vezels 2-3 segmenten ascenderen voordat ze in de ventrale commisuur kruisen.

Central Cord Syndrome

Bij kleine laesies kan schade aan de spinothalamische vezels die de ventrale commissure kruisen voor bilaterale gebieden van sensorisch verlies van pijn en temperatuur zorgen. Vaak bevat men verlies van deze modaliteiten aan de laterale zijden van beide armen.

Posterior Cord Syndrome

Laesies aan de posterior kolom kunnen voor verlies van vibratie en positie zorgen onder de laesie. Kan veroorzaakt worden door trauma, MS en vitamine B12-deficiëntie.

Anterior Cord Syndrome

Schade aan de anterolaterale pathways zorgt voor verlies van pijn en temperatuur sensatie onder het niveau van de laesie en schade aan de anteriore hoorncellen zorgt voor een zwakte van lagere motorneuronen op het niveau van de laesie.

 

Wat is somasensorie?

 

Somatosensorisch staat voor de lichamelijke sensaties als aanraking, pijn, temperatuur, vibratie en proprioceptie. Er bestaan twee pathways:

  1. Posterior column-medial lemniscal pathway: voor proprioceptie, vibratiegevoel, fijne tast (gnostisch)

  2. Anterolateral pathways: voor pijn, temperatuur en grove tast (vitaal)

Sensorische neuronale cellichamen liggen in de dorsale wortel ganglia. Een perifeer gebied dat geïnnerveerd wordt door sensorische vezels van één zenuwwortelniveau wordt een dermatoom genoemd.

 

Hoe verlopen de spinale zenuwwortels? - Chapter 8

 

 

Hoe werkt de segmentale organisatie van het zenuwstelsel?

Het ruggenmerg eindigt op het niveau van de conus medullaris. Deze ligt op het niveau van L1 of L2. De zenuwwortels lopen echter nog door tot het niveau waar ze naartoe moeten. Het gedeelte van de conus medullaris naar beneden wordt de cauda equina genoemd. De conus medullaris vormt een spits einde dat de filum terminale wordt genoemd. Dit is een streng van connectief weefsel dat in het midden van de cauda equina loopt en vasthecht aan de onderzijde van de benige wervelkolom.

C1 bevat als enige geen sensorische wortels. C5-T1 zorgen voor de zenuwwortels voor de armen, terwijl L1-S3 voor de zenuwwortels zorgen voor de benen. Een korte afstand van het ruggenmerg voordat de zenuwvezels uit het foramen vertebrale gaan worden er een gemengde spinale zenuw gevormd. Hierna worden er plexussen gevormd.

Zenuwwortels in relatie met de wervels, schijven en ligamenten

De wervels bestaan uit verschillende onderdelen als de processus spinosus en processus transversus en worden onderscheiden van andere wervels door een intervertebrale discus die uit de nucleus pulposus en annulus fibrosus bestaat. Daarnaast bevinden zich binnenin de wervels ook nog verschillende lagen als de pia mater, arachnoidea en dura mater. Op de afbeelding hieronder kan je zien dat er ook nog een epidurale ruimte aanwezig is, die in de schedel niet aanwezig was.

De zenuwwortels verlaten het foramen vertebrale via de neurale foramina intervertebralis. Thoracale, lumbale en sacrale zenuwwortels verlaten het foramen onder de corresponderende wervel. De cervicale wortels verlaten het boven de wervel, behalve C8. Dit komt doordat er geen 8ste cervicale wervel aanwezig is. De zenuwwortel verlaat het foramen intervertebrale tussen C7 en T1.

Wat zijn dermatomen en myotomen?

Een dermatoom is een sensorische regio van de huid dat door een zenuwwortel wordt geïnnerveerd. Er zijn verschillende dermatoomkaarten, maar er zijn ook overeenkomsten aanwezig. Sensatie van het gezicht wordt over het algemeen verzorgd door de n. trigeminus. De rest van het gezicht wordt verzorgd door C2. De tepels zitten op niveau T4, terwijl de navel op T10 ligt. Wanneer men C4-T2 gaat testen, moet je erop letten dat hiervoor geen sensatie aanwezig is op de borst of de rug, omdat C5 van belang is voor de schouder, C6 voor het laterale deel van de arm en twee vingers, C7 voor de middelvinger en C8 voor de 4e en 5e vinger.

In het boek wordt er in tabel 8.1 de verschillende zenuwvezels weergegeven.

KCC 8.3: Radiculopathy

Radiculopathy is een sensorische of motorische dysfunctie die veroorzaakt wordt door de pathologie van een zenuwwortel. Het wordt vaak geassocieerd met een brandende, tintelende pijn die uitstraalt naar een ledemaat van die bepaalde dermatoom. Radiculopathy kan zorgen voor verminderde sensatie en atrofie. Een discushernia komt het vaakst voor bij de zenuwwortel van C6, C7, L5 of S1. Wanneer er degeneratie optreedt door de hernia kunnen er osteofyten ontstaan. Dit zijn botachtige delen tussen de wervels. Het kan zorgen voor een vernauwing van het foramen intervertebrale of een spinale stenose.

Test: omhoog liften van rechte benen

Deze test is handig om een compressie van een zenuwwortel in het lumbosacrale deel te ontdekken. Bij deze test wordt het been steeds hoger gebracht. Bij een hoek minder dan 10 en meer dan 60 graden is er vaak geen sprake van een compressie.

Test: omhoog liften van gekruisde benen

Hierbij wordt het asymptomatische been omhoog gehesen. Dit zorgt voor een bepaald effect in het symptomatische been. Deze test is voor 90% specifiek voor een compressie van een lumbosacrale zenuwwortel.

Andere oorzaken van radiculopathy

  1. Diabetes

  2. Trauma

  3. Herpes zoster

  4. Spinale stenose

  5. Epidurale metastases

 

Hoe werkt de segmentale organisatie van het zenuwstelsel?

 

Het ruggenmerg eindigt op het niveau van de conus medullaris. Deze ligt op het niveau van L1 of L2. De zenuwwortels lopen echter nog door tot het niveau waar ze naartoe moeten. Het gedeelte van de conus medullaris naar beneden wordt de cauda equina genoemd. De conus medullaris vormt een spits einde dat de filum terminale wordt genoemd. Dit is een streng van connectief weefsel dat in het midden van de cauda equina loopt en vasthecht aan de onderzijde van de benige wervelkolom.

C1 bevat als enige geen sensorische wortels. C5-T1 zorgen voor de zenuwwortels voor de armen, terwijl L1-S3 voor de zenuwwortels zorgen voor de benen. Een korte afstand van het ruggenmerg voordat de zenuwvezels uit het foramen vertebrale gaan worden er een gemengde spinale zenuw gevormd. Hierna worden er plexussen gevormd.

 

Wat zijn de belangrijkste plexussen en perifere zenuwen? - Chapter 9

 

 

Wat zijn veel voorkomende plexus- en zenuwsyndromen?

KCC 9.1: Bovenste extremiteit zenuwbeschadigingen

  1. Brachial plexus, lower trunk injury (Erb-Duchenne palsy): Beschadigingen aan de bovenste truncus van de plexus brachialis kan zorgen voor verlies van functie van C5 en C6 geinnerveerde spieren. Men houdt de arm in een positie van de “waiter’s tip”. Zwakheid van de deltoideus, biceps en polsextensoren treedt op. Dit komt voor bij pasgeboren baby’s waarbij er aan de arm getrokken moest worden om de baby ter wereld te brengen.

  2. Brachial plexus, lower trunk injury (Klumpke’s palsy): Hierbij vindt er verlies van functie plaats van de C8-T1 geinnerveerde spieren. Het kan resulteren in zwakheid van de hand en de vinger, atrofie van de hypothenarspieren en sensorisch verlies van het ulnaire deel van de onderarm en de hand. Er zijn 2 syndromen te onderscheiden

    1. Thoracic outlet syndrome: Het onderste deel van de plexus brachialis wordt bekneld wanneer het de clavicula en de eerste rib passeert.

    2. Pancoast’s syndrome; hierbij verdrukt een apicale longtumor het onderste deel van de brachiale plexus. Vaak is de n. laryngeus recurrens beschadigd en treedt er heesheid op. Ook het syndroom van Horner (miosis, ptosis en anhidrosis) kan optreden.

  3. Axillaire neuropathie: dit kan optreden wanneer er een dislocatie of een fractuur optreedt van de proximale humerus waardoor de n. axillaris wordt bekneld. Atrofie van de m. deltoideus kan optreden.

  4. Brachial plexitis (brachial neuritis, parsonage-turner syndrome): Dit komt vaak voor bij mannen, waarbij er een brandend gevoel in de schouder of het laterale deel van nek wordt gevoeld.

  5. Radiale neuropathie: Hierbij treedt er zwakheid op van alle extensoren van de arm, hand en de vingers onder het niveau van de schouder. Er is vaak een ‘wrist drop’, oftewel een ‘dropping hand’, aanwezig.

  6. Mediale neuropathie: Het kan ontstaan door fracturen van de humerus of distale radius. Het kan ook vast komen te zitten wanneer het door de m. pronator teres loopt in de onderarm. Er is verminderde kracht voor polsflexie en abductie, oppositie van de duim en flexie van de 2e en 3e vinger. Men krijgt ook wel een predikershand.

  7. Carpale tunnel syndroom: Hierbij is er sprake van een compressie van de n. medianus. Het innerveert na het passeren van de carpale tunnel LOAF (mm. lumbricales I en II, m. opponens pollicis, m. abductor pollicis brevis, m. flexor pollicis brevis). Symptomen van sensorisch verlies worden gevonden in de eerste 3 vingers. Patiënten proberen door hun hand te schudden weer gevoel terug te krijgen in hun hand. Bij dit syndroom is er sprake van radiculopathy van C6 en C7 of compressie van de n. medialis proximalis.

  8. Ulnaire neuropathie: dit wordt vaak veroorzaakt door het beknellen van de n. ulnaris in het cubitale kanaal in de elleboog. Het kan ook beschadigd worden door fracturen van de mediale epicondyle. Er zal zwakte optreden voor flexie en adductie van de pols, adductie en abductie van de vingers.

Wat voor onderste extremiteit zenuwbeschadigingen zijn er?

  1. Femorale neuropathie: De n. femoralis kan beschadigd worden tijdens een operatie of bedrukt raken door een retroperitoneale hematoom. Er zal zwakte optreden voor de flexie van de dij en de extensie van de knie. Het ontstaat door L3-L4 neuropathie. Belangrijk om te onthouden is dat L3-L4 ook voor zwakte bij dij-adductie kan zorgen, maar dit niet van belang is bij deze vorm van neuropathy.

  2. N. ischiadicus neuropathie: dit kan veroorzaakt worden door dislocatie van het posteriore deel van de heup of een intramusculaire injectie. Er zal zwakte voorkomen van de voet en enkelspieren, en verlies van de achillespees reflex. De belangrijke oorzaak is echter een compressie van de lumbosacrale wortels door osteofyten.

  3. Peroneal nerve palsie: Wanneer de n. peroneus (tegenwoordig ook de n. fibularis genoemd) de kop van de fibula passeert, is het kwetsbaar voor schade en compressie. Er zal “foot drop”ontstaan (klapvoet), waardoor er ook zwakte van de dorsiflexie en eversie van de voet zullen optreden

  4. Obturator nerve palsie: Het kan bedrukt worden bij de bevalling of bij een operatie. Het zorgt voor instabiele looppas.

  5. Meralgia paresthetica: Hierbij kan de n. cutaneus femoralis lateralis worden opgesloten wanneer het het ligamentum inguinalis passeert. Het zorgt niet voor motorische uitval, maar alleen voor sensorische uitval.

  6. Morton’s Metatarsalgie: Hierbij worden de zenuwen in de tenen bedrukt door te strakke schoenen.

 

Wat zijn veel voorkomende plexus- en zenuwsyndromen?

 

KCC 9.1: Bovenste extremiteit zenuwbeschadigingen

  1. Brachial plexus, lower trunk injury (Erb-Duchenne palsy): Beschadigingen aan de bovenste truncus van de plexus brachialis kan zorgen voor verlies van functie van C5 en C6 geinnerveerde spieren. Men houdt de arm in een positie van de “waiter’s tip”. Zwakheid van de deltoideus, biceps en polsextensoren treedt op. Dit komt voor bij pasgeboren baby’s waarbij er aan de arm getrokken moest worden om de baby ter wereld te brengen.

  2. Brachial plexus, lower trunk injury (Klumpke’s palsy): Hierbij vindt er verlies van functie plaats van de C8-T1 geinnerveerde spieren. Het kan resulteren in zwakheid van de hand en de vinger, atrofie van de hypothenarspieren en sensorisch verlies van het ulnaire deel van de onderarm en de hand. Er zijn 2 syndromen te onderscheiden

    1. Thoracic outlet syndrome: Het onderste deel van de plexus brachialis wordt bekneld wanneer het de clavicula en de eerste rib passeert.

    2. Pancoast’s syndrome; hierbij verdrukt een apicale longtumor het onderste deel van de brachiale plexus. Vaak is de n. laryngeus recurrens beschadigd en treedt er heesheid op. Ook het syndroom van Horner (miosis, ptosis en anhidrosis) kan optreden.

  3. Axillaire neuropathie: dit kan optreden wanneer er een dislocatie of een fractuur optreedt van de proximale humerus waardoor de n. axillaris wordt bekneld. Atrofie van de m. deltoideus kan optreden.

  4. Brachial plexitis (brachial neuritis, parsonage-turner syndrome): Dit komt vaak voor bij mannen, waarbij er een brandend gevoel in de schouder of het laterale deel van nek wordt gevoeld.

  5. Radiale neuropathie: Hierbij treedt er zwakheid op van alle extensoren van de arm, hand en de vingers onder het niveau van de schouder. Er is vaak een ‘wrist drop’, oftewel een ‘dropping hand’, aanwezig.

  6. Mediale neuropathie: Het kan ontstaan door fracturen van de humerus of distale radius. Het kan ook vast komen te zitten wanneer het door de m. pronator teres loopt in de onderarm. Er is verminderde kracht voor polsflexie en abductie, oppositie van de duim en flexie van de 2e en 3e vinger. Men krijgt ook wel een predikershand.

  7. Carpale tunnel syndroom: Hierbij is er sprake van een compressie van de n. medianus. Het innerveert na het passeren van de carpale tunnel LOAF (mm. lumbricales I en II, m. opponens pollicis, m. abductor pollicis brevis, m. flexor pollicis brevis). Symptomen van sensorisch verlies worden gevonden in de eerste 3 vingers. Patiënten proberen door hun hand te schudden weer gevoel terug te krijgen in hun hand. Bij dit syndroom is er sprake van radiculopathy van C6 en C7 of compressie van de n. medialis proximalis.

  8. Ulnaire neuropathie: dit wordt vaak veroorzaakt door het beknellen van de n. ulnaris in het cubitale kanaal in de elleboog. Het kan ook beschadigd worden door fracturen van de mediale epicondyle. Er zal zwakte optreden voor flexie en adductie van de pols, adductie en abductie van de vingers.

 

Hoe werken de cerebrale hemisferen en bloedvoerziening? - Chapter 10

 

 

Wat is de algemene indeling?

Het gezicht en de handgebieden van de sensorimotore homunculi liggen in de laterale delen, terwijl de beengebieden in de interhemisferische fissuur liggen. In de dominante hemisfeer, vaak de linker, ligt het gebied van Broca in de inferiore frontale gyrus. Het ligt anterior van de articulatoire gebieden van de primaire motorcortex. Het gebied van Wernicke ligt in de superiore temporale gyrus en is van belang bij taalbegrip.

Wat is de circle of Willis?

De arteriële toevoer naar de cerebrale hemisfeer komt van de aa. carotis internae en de aa. vertebralis. De aa. carotis interna zorgen voor de anteriore circulatie en de aa. vertebralis voor de posterior circulatie. Ze komen samen in de anastomerende ring, de cirkel van Willis. Uit de cirkel van Willis komen de aa. cerebri anterior en de aa. cerebri media uit de aa. carotis internae (zowel links- als rechtszijdig). De anteriore cerebrale arteriën anastomoseren via de anteriore communicerende arterie (a. communicans anterior) en posterior via de posteriore communicerende arteriën (aa. communicans posteriores). De aa. cerebri posteriores ontspringen uit de a. basilaris die gevormd wordt door twee vertebrale arteriën.

De a. carotis interna bevat verschillende segmenten:

  • Cervicale segment: in de nek

  • Petrous segment: in temporale bot

  • Cavernous segment: carotis krijgt een s-vorm

  • Carotid siphon; in de cavernous sinus

Wat is de anatomie en vasculaire gebieden van de drie belangrijkste cerebrale slagaders?

De a. cerebralis anterior, -media en -posterior zorgen voor verschillende vertakkingen in de subarachnoidale ruimte die de sulci en andere diepe structuren van bloed voorzien.

Vasculaire gebieden van de oppervlakkige hersenstructuren

De a. cerebralis anterior loopt over het corpus callosum. Het bevat twee takken, de pericallosale en callosomarginale arterie. Het voorziet het anterior mediale deel van de hersenen van bloed.

De a. cerebralis media gaat de Sylvian fissure in. Daar splitst het in een superior gedeelte en een inferior gedeelte. Soms kunnen er zelfs 3 of 4 hoofdtakken ontspringen. De takken vormen loops naar het laterale deel van de cortex. Het superior deel voorziet de cortex boven de sylvian fissure en de inferior dat wat onder de Sylvian fissure ligt.

De a. cerebralis posterior voorziet de inferiore en mediael temporale en occipitale cortex.

Vasculaire gebieden van de diepe cerebrale structuren

De belangrijkste doordringende vaten aan de basis van de hersenen zijn de lenticulostriate arteriën. Deze kleine vaten ontspringen uit de a. cerebralis media voordat deze de Sylvian fissure ingaat. Er zijn nog andere kleinere arteriën die diepe structuren voorzien van bloed. De a. choroidal anterior ontspringt uit de a. carotis interna. Het zorgt voor de bloedvoorziening van putamen, thalamus, posteriore been van de capsula interna en de globus pallidus. De recurrente arterie van Heubner komt uit een deel van de a. cerebralis anterior en voorziet delen van het caput van de nucleus caudatus, anteriore putamen en de capsula interna. Als laatste zijn er nog de thalmoperforator arteriën die uit de a. cerebralis posterior ontspringen. Zij voorzien de thalamus en een deel van het posterior been van de capsula interna van bloed.

Veneuze drainage van het cerebrale hemisfeer

KCC 10.1: Klinische syndromen van de drie cerebrale slagaders

Middle cerebral artery (MCA) - Middelste hersenslagader

Een infarct of een ischemisch moment komen vaker voor in het voorzieningsgebied van deze arterie dan in de aa. cerebri anterior en aa. cerebri posterior, omdat de aa. cerebri media een groter gebied van bloed voorzien. Een infarct kan in 3 gebieden voorkomen:

  1. Superior divisie

  2. Inferior divisie

  3. Deep territory

Infarcten van de hoofdstam van de a. cerebri media (MCA) zijn proximale MCA occlusies die alle 3 de gebieden aantasten. Een voorbeeld van een tekort aan bloedtoevoer is een infarct aan de linkerkant in het superiore deel. Dit kan zorgen voor zwakte van de rechter kant van het gezicht en de arm. Van belang om te weten zijn de verschillende cortexgebieden, zodat je aan de hand van dat de verschillende verschijnselen kan onderscheiden. Andere voorbeelden staan weergegeven in tabel 10.1

Anterior cerebral artery

Hierbij treedt er vaak zwakte op van motorneuronen en corticale type sensorisch verlies (zowel vitaal als gnostisch) die het contralaterale been meer dan de arm of het gezicht aantasten (denk hierbij aan de homunculus en de projectie daarvan op de cortex!). Grote aanvallen kunnen voor contralaterale hemiplegie zorgen.

Posterior cerebral artery

Deze zorgen vaak voor een contralaterale homonieme hemianopsie. Hierbij zijn beide linker- of rechtergezichtsvelden aangedaan.

KCC 10.2: Watershed infarcten

Watershed zones (‘waterscheidingszones’) zijn gebieden die tussen de voorzieningsgebieden van twee cerebrale arteriën liggen. Wanneer de bloeddruk daalt en er geen bloed meer in deze arteriën stroomt, kan er ischemie of een infarct in dit waterscheidingsgebied optreden. Dit kan voor proximale arm- en beenzwakte zorgen.

KCC 10.3: Transiënte ischemische aanval (TIA) en andere transiënte neurologische episoden

Een TIA is een neurologisch verschijnsel waarbij voor ongeveer 10 minuten tijdelijk hersenischemie heeft plaatsgevonden. TIA’s die meer dan een uur duren zijn kleine infarcten. Ze zijn een teken voor een acute coronaire ziekte of een onstabiele angina. Patiënten die een TIA hebben gehad, hebben een grote kans op nog een TIA. TIA’s kunnen ontstaan door een embolus of thrombusformatie waardoor er een occlusie kan optreden. Mensen waarvan bekend is dat ze een TIA hebben doorgemaakt, worden behandeld met een kinderdosis aspirine (80mg).

KCC 10.4: Ischemische beroerte: mechanismen en behandeling

Mechanismen van ischemische beroertes

Necrose ontstaat wanneer er voor een bepaalde tijd minder bloedtoevoer aanwezig is naar een bepaald gebied. Een embolisch infarct (obstructie van een bloedvat door een embolus, waardoor er necrose van het achterliggende weefsel optreedt) wordt veroorzaakt door een occlusie door een deeltje (embolus) dat uit een ander deel van het lichaam afkomstig is. Bij een thrombotisch infarct wordt een bloedprop gevormd op een bepaalde plek in een bloedvat, waardoor er een plaque ontstaat en dus een occlusie. Dissectie van een vertebrale of carotide arterie kan voor thrombus formatie zorgen die weer kan emboliseren in de hersenen.

Er zijn verschillende embolievormen die een infarct (‘stroke’) kunnen veroorzaken. Zo zijn er embolieën gevormd van lucht, bacterie of cholesterol.

Lacunes zijn kleine vaatinfarcten die vaak gezien worden bij chronische hypertensie, waardoor de diepe witte stof aangetast kan worden of de kernen van de cerebrale hemisferen en hersenstam.

Risicofactoren van beroertes

Er moet bij patiënten gevraagd worden naar hypertensie, diabetes mellitus, roken, familiegeschiedenis en eigen geschiedenis met vasculaire ziekten. Bij jonge patiënten moet er gekeken worden naar leukemie, dehydratie, adenocarcinoom, etc.

Behandeling en diagnose van ischemische beroertes & TIA

Acuut

Wanneer er een cerebrovasculair accident (CVA) heeft plaatsgevonden, moet men snel aan de hand van een CT-scan erachter komen of er sprake is van bloeding (20% van de gevallen) of een infarct (80% van de gevallen). Wanneer er sprake is van een infarct kan met de patiënten behandelen met tissue plasminogeen activator (tPA). Bij patiënten die niet behandeld kunnen worden met tPA moet er aspirine worden toegediend om een terugkerend infarct tegen te gaan. Er moet snel gekeken worden naar hypoglycemie en hyperglycemie omdat deze snel voor verslechterende situaties kunnen zorgen.

Medium-lange termijn

Er kunnen vertraagde zwellingen voorkomen of toch nog een bloeding na bijvoorbeeld een licht trauma. Patienten met een infarct van de a. cerebralis media kunnen oedeem krijgen in de 1e 3-4 dagen na een infarct. Men moet revalideren en kan preventieve maatregelen nemen als het innemen van statines en aspirine.

KCC 10.5: Carotide Stenose

Atherosclerotische ziekten kunnen zorgen voor stenosen vlak na de bifurcatie van de gemeenschappelijke carotis arterie (a. carotis communis). Het kan tijdens lichamelijk onderzoek worden gehoord als een apart geluid (souffle) tijdens de diastole. Symptomatische carotisstenose kan leiden tot ipsilaterale voorbijgaande blindheid aan één oog (amaurosis fugax).

 

Wat is de algemene indeling?

 

Het gezicht en de handgebieden van de sensorimotore homunculi liggen in de laterale delen, terwijl de beengebieden in de interhemisferische fissuur liggen. In de dominante hemisfeer, vaak de linker, ligt het gebied van Broca in de inferiore frontale gyrus. Het ligt anterior van de articulatoire gebieden van de primaire motorcortex. Het gebied van Wernicke ligt in de superiore temporale gyrus en is van belang bij taalbegrip.

 

Hoe is de hersenstam opgebouwd (anatomie en hersenzenuwen)? - Chapter 12

 

 

Wat zijn de kenmerken van het oppervlak van de hersenstam

De hersenstam bestaat uit verschillende onderdelen. Zo bevat het de middenhersenen (mesencephalon), de pons en de medulla. Het cerebellum is aan het dorsale oppervlak van de pons en het bovenste deel van de medulla gehecht. Het dak van de middenhersenen, het tectum, wordt gevormd door de colliculi superiores en colliculi inferiores. Aan de voorkant bevat het mesencephalon cerebrale pedunculi. Achter de pons bevindt zich het vierde ventrikel. De dorsolaterale kant van de pons wordt gehecht aan het cerebellum via de superiore, mediale en inferiore cerebellaire pedunculi.

Hierboven kun je zien dat er verschillende hersenzenuwen bestaan. De eerste twee hersenzenuwen ontspringen echter niet uit de hersenstam. Ze kunnen dus direct contact maken met de voorhersenen. De twee nn. olfactorius lopen via de peervormige spuit richting de tractus olfactorius die langs de ventrale kant van de frontale kwabben naar de olfactorische sulci lopen. De optische zenuwen lopen via het chiasma opticum. CN III – XII ontspringen wel uit de hersenstam. Ze ontspringen aan de ventrale kant of de ventrolaterale kant. De enige uitzondering is CN IV. Deze komt aan de dorsale kant van de hersenstam naar buiten.

Schedelopeningen en hersenzenuw uitgangspunten

Er zijn verschillende foramina waaruit de verschillende zenuwen de hersenen naar hun doelorganen kunnen gaan. In de afbeelding hieronder kan je zien welke zenuw door welk kanaal of andere opening loopt.

Hoe is de sensorische en motorische organisatie van de hersenzenuwen opgebouwd?

De motorische hersenzenuwkernen liggen meer ventraal gelegen, terwijl de sensorische hersenzenuwkernen meer dorsaal gelokaliseerd zijn. Er ontstaan in het zenuwstelsel 3 motorische kolommen en 3 sensorische kolommen met hersenzenuwkernen.

Motorische hersenzenuwkernen

  1. Somatische motorkernen: dit zijn de kernen van de n. oculomotorius (CNIII), n. trochlearis (CN IV), n. abducens (CN VI) en n. hypoglossus (CN XII). Deze liggen vlakbij de middenlijn. Ze innerveren de extraoculaire en intrinsieke tongspieren.

  2. Viscerale motorkernen: Deze splitst zich verder in 2 kolommen van kernen:

    1. Branchiale motorkernen: Dit zijn de kernen van de n. trigeminus (CN V), n. facialis (CN VII), ambiguus(CN IX, X) en n. accessorius (CN XI).

    2. Parasympatische kernen: Dit zijn de Edinger-Westphal (CN III), superior salivatory (CN VII) en inferior salivatory (CN IX) kernen en de dorsale motorkernen van de n. vagus (CN X). Het zijn de enige kernen die geen dwarsgestreepte spieren innerveren.

Sensorische hersenzenuwkernen

  1. Viscerale sensorische column: Deze bevat de nucleus solitarius die je in twee delen kan verdelen. Het rostrale deel ontvangt input uit CN VII, CN IX en X. De caudale, of cardiorespiratoire kern, ontvang input uit CN IX en X.

  2. General somatosensorische kernen: Dit worden ook wel de trigeminale kernen genoemd. Deze zijn er voor de aanraking, pijn, temperatuur, positie en vibratiegevoel van het gezicht, sinusen en meningen. Het ontvangt input uit CN V, VII, IX en X.

  3. Speciale somatosensorische kernen: De speciale zintuigen zijn de reuk, visie, gehoor, tast en evenwicht. Deze kernen zijn van de n. cochlearis (CN VIII) en de n.vestibularis (CN VIII).

Wat voor functies en verloop van de hersenzenuwen zijn er?

Er zijn twaalf hersenzenuwen:

  1. N. olfactorius: van belang bij reuk. Via chemoreceptoren op de bipolaire primaire sensorische neuronen in het olfactoire neuroepitheel in de bovenste nasale holtes kunnen verschillende stimuli worden gedetecteerd. Via de olfactoire zenuwen die door de cribriforme plaat van het ethmoid lopen, kunnen er synapsen plaatsvinden met de bulbi olfactoria, de peervormige structuren, en kan de informatie naar de sulcus gaan.

  2. N. opticus: informatie wordt van de retina via deze zenuw naar de corpi geniculatum laterale van de thalamus gebracht en vervolgens naar de extrageniculate pathways. Het deel van de visuele pathway dat voor het chiasma opticum ligt wordt de n. opticus genoemd, terwijl het erna de tractus opticus wordt genoemd.

  1. N. oculomotorius: Van belang bij alle extraoculaire spieren, behalve de m. obliquus superior en de m. rectus lateralis. Ook belangrijk voor de levator palpebrae superior. Het geeft informatie aan de kernen die in het mesencephalon gelegen zijn (colliculi superiores). Vanaf het mesencephalon verloopt de baan via de sinus cavernosus naar de uitgang van de schedel bij de fissura orbitalis superior. Het bevat ook parasympatische kernen voor de m. constrictor pupillae en de m. ciliaris die invloed heeft op de bolling van de lens.

  1. N. trochlearis: Van belang voor de innervatie van de m. obliquus superior en zorgt voor motorische depressie en intorsie van het oog. Intorsie is een binnenwaartse wending van de verticale corneale meridianen. De zenuw verloopt vanaf de nuclei trochlearis aan de dorsale kant van het tectum inferior dezelfde route als CN III. Van de nuclei gaat het dus naar de sinus en de orbital fissure.

  1. N. trigeminus: Deze zenuw bestaat uit 3 verschillende componenten. De sensorische component is van belang bij de sensibele innervatie van het gezicht en de motorische bij het kauwen. De drie delen zijn de ophtalmic(V1), maxillary(V2) en mandibular(V3) division. De zenuw komt ventrolateraal uit de pons en loopt verder naar Meckel’s cave. Hierin ligt ook wel het sensorische ganglion van de zenuw. V1 loopt via het inferiore deel van de cavernous sinus naar de superior orbital fissure. V2 verlaat het skelet via het foramen rotundum en V3 via het foramen ovale. Het sensorische deel is ook van belang bij de pijnsensatie voor de nasale sinusen, de mond en het voorste 2/3e deel van de tong.

Trigeminale somatische sensorische functie

De trigeminale kernen ontvangen sensorische informatie van CN V, VII, IX en X. Het trigeminale kerncomplex loopt van het mesencephalon naar het bovenste deel van het cervicale ruggenmerg en bevat 3 kernen: mesencephalische, chief sensorische en spinale trigeminale kernen. De chief sensorische kern ontvangt informatie over de fijne tast en tanddruk. De spinale trigeminale kern ontvangt grove tast, pijn en temperatuur sensatie. De mesencephalische trigeminale kern en tractus is belangrijk voor de kaakreflex.

Trigeminale branchiale motorfunctie

De trigeminale motor kern bevat een functie bij de branchiale motorfuncties van de trigeminus. Tezamen met V3 gaat het de schedel uit via het foramen ovale.

N. abducens: Deze zenuw is naast CN III en IV ook van belang bij de controle over de extraoculaire spieren. Het is van belang bij de innervatie van de m. rectus lateralis en zorgt voor abductie van het oog. Het loopt vanaf de abducens kern die in de pons gelegen is, ook via de cavernous sinus naar de orbital fissure.

  1. N. facialis: Deze zenuw bevat twee takken, namelijk de motorische en de nervus intermedius. De motorische is van belang voor de spieren van de gezichtsexpressie en de nervus intermedius is van belang voor tranen en speekselvorming, smaak van 2/3e deel van de tong en sensatie van een deel van het oor dicht bij de externe gehoorgang. De n. facialis verlaat de hersenstam via de pontomedullaire junction lateraal t.o.v. CN VI in een regio dat de cerebellopontine angle wordt genoemd. Het gaat daarna richting de interne gehoorgang. Het grootste deel van de n. facialis verlaat de schedel via het foramen stylomastoideus. Voorbij de oorspeekselklier splitst het in 5 belangrijke motorische takken voor de expressie. Op de afbeelding hieronder kan je de verschillende banen zien van de n. facialis.

  2. N. vestibulocochlearis: Tezamen met de n. facialis komt deze zenuw terecht in de interne gehoorgang.

Gehoorgangen

Via de membrana tympanica (trommelvlies) en de verschillende oorbotjes komen de geluidsgolven terecht bij het ovale venster. In het benige labyrint bevindt zich perilymfe, waarin een vliezig labyrint drijft waarin endolymfe zit. Tot het vliezige labyrint behoren ook de ductus cochlearis, de utriculus, sacculus en de drie halfcirkelvormige kanalen. De vibraties bereiken de ductus cochlearis waarin de haarcellen zullen bewegen. De haarcellen van de cochlea tezamen met hun helpende cellen worden het orgaan van Corti genoemd. Primaire sensorische neuronen in het spirale ganglion sturen hun axonen naar de dorsale en ventrale cochlear nuclei.

Vestibular Pathways

De vestibulaire kernen zijn van belang voor postuur, spiertonus en oogpositie. De semicirculaire kanalen kunnen verschillende hoekveranderingen detecteren. Rotatie van het hoofd zorgt voor veranderingen van beweging van het endolymfe. Wanneer er een versnelling plaats vindt, zal dit waargenomen worden door de crista ampullaris waarin mechanoreceptor cilia zitten. De utriculus en sacculus bevatten structuren die de maculae worden genoemd en veel lijken op de cristae ampullaris. Deze nemen lineaire acceleratie waar. Via de otolieten die zich in de macula bevinden, kunnen er trillingen opgevangen worden door de cilia, waardoor er weer signalen ontstaan naar de zenuw.

Er bestaan 4 verschillende vestibulaire kernen aan elke kant van de hersenstam:

  • Superior

  • Lateraal: Deze zorgt voor het laterale vestibulospinale traject, maar is deel van het mediale descenderende motorische systeem. Het is belangrijk voor balans en extensor tonus.

  • Mediaal: deze zorgt voor de mediale vestibulospinale traject en bevat delen van de inferiore vestibulaire nucleus. Het gaat alleen naar het cervicale deel van het ruggenmerg en is van belang voor de nek en de hoofdpositie.

  • Inferior

De fasciculus longitudinalis medialis is een belangrijke pathway die de kernen die van belang zijn voor oogbewegingen met elkaar en andere vestibulaire kernen verbindt. De FLM is van belang voor de vestibulo-oculaire reflex.

  1. N. glossopharyngeus: Via de inferior olivary en de inferior cerebellaire pedunculus gaat deze zenuw via het foramen jugularis uit de schedel. Het motorische deel zorgt voor de m. stylopharyngeus die de pharynx tijdens het praten en slikken eleveert. Hiernaast is het ook van belang bij de “gag reflex” (kotsreflex). Het visceraal sensorische deel ontvangt input uit baro- en chemoreceptoren van de carotiden. Het somatische sensorische deel bevat de sensatie van het middenoor, farynx en het posteriore 1/3e deel van de tong. Tot slot bevat de speciale visceraal sensorische categorie de smaak voor het posteriore 1/3 deel van de tong.

  2. N. vagus: Deze zenuw verlaat ook de schedelholte via het foramen jugularis. Het grootste deel zorgt voor de parasymatische innervatie van het hart, de longen en het maag-darm stelsel. De branchiale motor component van de vagus is van belang voor alle faryngeale en bovenste oesofagale spieren en de spieren van de larynx. De general somatisch sensorische vezels zijn van belang voor de farynx, larynx en een klein deel bij de externe gehoorgang. De viscerale sensatie bestaat uit de smaak van de epiglottis en farynx en ze ontvangen informatie van chemoreceptoren en baroreceptoren uit de aortaboog. De n. vagus splitst zich in de n. laryngeus recurrens die de controle bevat over alle intrinsieke spieren van de stembanden behalve de m. cricothyroideus. Deze wordt gecontroleerd door de 2e tak van de n. vagus, namelijk de n. laryngeus superior.

  3. N. accessorius:Deze zenuw ontspringt uit de bovenste 5/6 segmenten van het cervicale ruggenmerg. Vanuit daar ascendeert het naar het foramen magnum om in de intracraniale holte te komen. Via het foramen jugularis verlaat het de schedelholte weer om de m. sternocleidomastoideus en de m. trapezius te innerveren. Contractie van de linker sternocleidomastoideus zorgt voor een beweging van het hoofd naar rechts, en vice versa.

  4. N. hypoglossus: deze zenuw bevat zijn eigen foramen, namelijk het foramen hypoglossus. Het zorgt voor somatisch motorische innervatie van alle intrinsieke en extrinsieke tongspieren, behalve de palatoglossus die door CN X wordt geïnnerveerd.

CCK 12.1: Anosmia (CN I)

Mensen met unilateraal verlies van reuk hebben vaak niets door, omdat de andere kant nog werkt. Het kan veroorzaakt wordt door trauma, waardoor de n. olfactorius wordt beschadigd. Intracraniale laesies bij de olfactoire sulci in de frontale kwabben kunnen ook voor problemen van het reukvermogen zorgen.

CKK 12.2: Trigeminale zenuwstoornissen (CN V)

Het komt vaak niet voor dat er problemen zijn met deze zenuw, maar toch is er wel één probleem dat voorkomt: trigeminale neuralgia. Hierbij krijgen patiënten bepaalde periodes hevige pijn die van seconden tot minuten kunnen duren. Het krijgen van aanvallen begint na iemands 35ste. Er zijn waarschijnlijk problemen bij de distributie van V2 of V3. Laesies van de trigeminale kernen in de hersenstam zorgen voor ipsilateraal verlies van de gezichtssensatie voor pijn en temperatuur. Het kan ontstaan door demyelinisatie, infarcten, beknelling of andere laesies.

CKK 12.3: Faciale zenuwletsels (CN VII)

Het is van belang om te weten of de zwakte in het gezicht veroorzaakt wordt door upper motor neuron laesies of lower motor neuron laesies. De lower motor neuronen die het voorhoofd controleren krijgen input van upper motor neuronen van de bilaterale motorcortexen. Bij unilaterale upper motor neuron laesies is er alleen zwakte aanwezig van het gedeelte onder het voorhoofd. Lower motor neuron laesies tasten de gehele helft van het gezicht aan.

De bekendste laesie die men kent van de n. facialis is de Bell’s palsy. Hierbij zijn alle delen van de zenuw voor een bepaalde tijd aangetast, en bestaat er verlamming van zowel de oog- als de mondspieren aan één zijde (perifere zenuwlaesie).

Vaak klagen patiënten ook over retroauriculaire pijn die veroorzaakt wordt door de somatosensorische component van de zenuw. Patienten kunnen ook last krijgen van een “droog oog” door verlaagde lacrimatie. 80% van de patiënten herstelt in 3 weken, maar er kan soms ook nog wat schade achterblijven. Dit kan komen doordat er tijdens herstel regeneratieve vezels het verkeerde doelwit raken. Een voorbeeld hiervan is het ontstaan van krokodillentranen, waarbij men moet huilen i.p.v. het produceren van speeksel wanneer ze eten zien.

CKK 12.4: Corneale reflex en kaakschokreflex

Bij de corneal reflex wordt een wattenstaafje langs de cornea gebracht wat voor het sluiten van de oog moet zorgen. Wanneer dit niet gebeurt, is er waarschijnlijk een laesie in de trigeminale sensorische pathways, de n. facialis of hun connecties met elkaar. De laesie vertoont een ipsilateraal oogprobleem. Bij de jaw jerk reflex wordt er getikt op de kin. Wanneer deze naar voren springt, betekent dit dat er een motorische neuron laesie aanwezig is.

CKK 12.7: Aandoeningen van CN IX, X, XI en XII

Perifere laesies van de lagere hersenzenuwen komen niet zo vaak voor. Er is vaak sprake van centrale laesies. Glossopharyngeale neuralgie zorgt voor periodes van een zere keel en oorpijn. Schade aan de n. laryngeus recurrens zorgt voor paralyse van de stembanden of heesheid. Mensen met tumoren van de glomus jugularis, dus een tumor van de glomus lichamen die veel op de carotid bodies lijken, hebben een verminderde werking van CN IX, X en XI. De tumor kan zo groot groeien dat het in het middenoor terecht kan komen.

CKK 12.8 : Heesheid, dysartrie, dysfagie en pseudobulbare affect

Stembeschadigingen ontstaan wanneer de larynx of de stemplooien niet meer goed werken. Heesheid ontstaat door asynchronische vibratie patronen van de stemplooien. Het wordt vaak veroorzaakt door mechanische factoren als zwellen van de stemplooien. Dysartrie is een abnormale articulatie van de spraak. Het kan ontstaan bij laesies van spieren die van belang zijn voor de articulatie, neuromusculaire junctie of perifere/centrale delen van CN V, VII, IX, X of XI. Dysfagie is een verminderde slikfunctie. Het kan veroorzaakt worden door oesofagale problemen, neoplasma, of andere locale laesies. Het slikproces bestaat uit 4 stappen:

  1. orale preparatie fase

  2. orale fase

  3. faryngeale fase

  4. oesofagale fase

 

Wat zijn de kenmerken van het oppervlak van de hersenstam

 

De hersenstam bestaat uit verschillende onderdelen. Zo bevat het de middenhersenen (mesencephalon), de pons en de medulla. Het cerebellum is aan het dorsale oppervlak van de pons en het bovenste deel van de medulla gehecht. Het dak van de middenhersenen, het tectum, wordt gevormd door de colliculi superiores en colliculi inferiores. Aan de voorkant bevat het mesencephalon cerebrale pedunculi. Achter de pons bevindt zich het vierde ventrikel. De dorsolaterale kant van de pons wordt gehecht aan het cerebellum via de superiore, mediale en inferiore cerebellaire pedunculi.

Hierboven kun je zien dat er verschillende hersenzenuwen bestaan. De eerste twee hersenzenuwen ontspringen echter niet uit de hersenstam. Ze kunnen dus direct contact maken met de voorhersenen. De twee nn. olfactorius lopen via de peervormige spuit richting de tractus olfactorius die langs de ventrale kant van de frontale kwabben naar de olfactorische sulci lopen. De optische zenuwen lopen via het chiasma opticum. CN III – XII ontspringen wel uit de hersenstam. Ze ontspringen aan de ventrale kant of de ventrolaterale kant. De enige uitzondering is CN IV. Deze komt aan de dorsale kant van de hersenstam naar buiten.

 

Hoe werken de pupilcontrole en oogbewegingen? - Chapter 13

 

 

Hoe zit de anatomie van het oog in elkaar?

Het oog heeft externe (oogbewegingen) en interne oogspieren(lensaccommodatie en pupilcontrole). Oogbewegingsbanen en stoornissen worden ingedeeld op twee niveaus: supranucleair en nucleair/infranucleair. De nucleaire en infranucleaire banen, dus de banen van de hersenstamkernen van de n.oculomotoris, n.trochlearis en n.abducens (III, IV, VI). En de supranucleaire banen, dit zijn banen in de hersenstam en frontale hersenen die verbonden zijn met de eerder genoemde kernen.

Elk oog heeft 6 externe oogspieren. De m. rectus medialis zorgt voor een beweging naar nasaal. De m. rectus lateralis naar temporaal. De m. rectus superior naar boven en de m. rectus inferior naar beneden. De m. obliquus superior zorgt voor intorsie, de m. obliquus inferior voor extorsie. De obliquus superior hecht aan het oog na een bochtje via de trochlea (een fibreus loepje aan het mediale orbitabot), de obliquus inferior niet. Wat voor oogbeweging er wordt gemaakt, is afhankelijk van de oogas. De m. rectus superior kan voor zowel intorsie, als elevatie zorgen bij naar voren kijken. De m. rectus inferior kan bij naar voren kijken ook zorgen voor depressie en extorsie. Bij abductie (naar temporaal) van 23 graden van het oog kan de m. rectus superior voor pure elevatie zorgen. Bij adductie vindt er meer intorsie plaats. Is het oog geadduceerd, dan kan de m. obliquus superior zorgen voor depressie en de m. obliquus inferior voor elevatie. Bij abductie zorgt de m. obliquus superior met name voor intorsie en de inferior voor extorsie.

De nervi oculomotoris, trochlearis en abducens lopen vanaf de hersenstam, door de sinus cavernosus, naar de orbita via de fissura orbitales superior. De n. oculomotoris innerveert alle externe spieren, behalve de m. rectus lateralis (n. abducens) en de m. obliquus superior (n. trochlearis). De ramus superior van de n. oculomotoris innerveert de m. rectus superior, de m. levator palpebrae superior (ooglid elevator). De ramus inferior innerveert de m .rectus medialis, m. rectus inferior, m. obliquus inferior. Deze ramus bevat ook preganglionaire parasympatische vezels die naar de m. pupillae constrictor worden vervoerd. De hersenstam kernen van n. III, IV en VI behoren samen met de n. XII tot de somatisch motorische kernen. Ze liggen allemaal in het midden naast de ventrikels. Hun vezels treden ventraal uit, behalve de trochlearis die dorsaal uittreedt.

De oculomotoriskernen bevinden zich in de bovenste middenhersenen, ventraal van de het periaquaductale grijze stof. Tussen de a. cerebri posterior en a. cerebri superior treedt de n. oculomotoris uit deze kern via de fossa interpeduncularis. De Edinger-Westphal kern is een subkern van de oculomotoriskernen en bevat parasympatische vezels. Deze vezels zorgen voor pupilvernauwing en bevinden zich mediaal en oppervlakkig in de n. oculomotoris. De vezels zijn gevoelig voor aneurysmata, met name van de nabijgelegen a .communicans posterior. De subkernen zijn klinisch niet zo relevant op twee dingen na:

  • Eenzijdige zwakte van de pupilvernauwing of m. levator palpebrae superior kan niet komen door een eenzijdige laesie van de oculomotoriskern.

  • Een oculomotoriskern laesie zorgt voor aantasting van de contralaterale m. rectus superior en de ipsilaterale m. rectus suprior.

Klinisch zal een laesie van de oculomotoriskern dus nooit zorgen voor een eenzijdige ptosis, eenzijdige m. rectus superior verlamming of eenzijdige verwijde, niet-reagerende pupil. De trochleariskernen bevinden zich in de lage middenhersenen, ze liggen ook ventraal van het periaquaductale grijze stof. Het zijn de enige hersenzenuwen die dorsaal uittreden en elkaar kruisen. Ze lopen naar beneden, kruisen dan naar de tegenovergestelde kant en treden uit bij het mediale anterieure velum van de hersenstam (hier zijn ze gevoelig voor verdrukking door kleine hersenentumoren), dan gaan ze onder het tentorium cerebelli door, via hetzelfde eerder genoemde traject naar de orbita. De abducenskernen liggen op de vloer van het vierde ventrikel. Het treedt uit bij de kruising tussen de pons en medulla, dan via de subarachnoïdale ruimte naar het kanaal van Dorello. Dit bevindt zich tussen de dura en schedel. Via Dorello maakt het een scherpe bocht bij de petrouze punt van het temporaal bot en dan net zoals de andere zenuwen via hetzelfde traject naar de orbita. Deze zenuw is gevoelig voor druk van bovenaf, zoals bij een verhoogde intracraniële druk.

Wat is dubbelzien?

Oorzaken:

  1. Laesies van de n. oculomotoris, abducens, trochlearis en hun centrale banen

  2. Neuromusculaire transmissie stoornis

  3. Stoornissen van de externe oogspieren (schildklierstoornis, orbitale myositis)

  4. Mechanisch trauma (orbitafractuur met spierklemming)

Af en toe kan dubbelzien (diplopie) door intoxicatie (alcohol), stoornissen van de supranucleaire oculomotoris banen en het syndroom van Magendie-Hertwig (skew deviatie) veroorzaakt worden. Het zien van 3 of meer beelden of mono-oculair dubbelzien kan veroorzaakt worden door: visuele cortexstoornis, psychiatrische stoornissen of een oogziekte, maar niet door oogbewegingsproblemen. Vraag uit of de diplopie verdwijnt bij het bedekken van één oog. Zo ja, dan is de oorzaak een abnormaliteit van de oogbeweging. Is er slechts een gering verschil in oogrichting, dan kan een patiënt alleen last hebben van een wazig beeld i.p.v. diplopie. Vraag ook altijd bij welke kijkrichting het optreedt. En of het verergert bij het kijken van objecten die dichtbij of juist veraf liggen.

Als een externe oogspier niet goed werkt, kunnen de ogen niet meer in dezelfde richting kijken en ontstaat er dubbelzien. Onthoud als vuistregel: het beeld dat het meest van de mediaanlijn afligt en in de richting van de geprobeerde kijkrichting, is altijd diegene die door het aangedane oog gezien wordt. Bijv: als je naar links kijkt en één oog kan niet naar links bewegen, dan zal dat oog een tweede verplaatst beeld aan de linkerkant vormen. Een test die je kunt uitvoeren is de ‘rode glas test’, hierbij zet je over het ene oog een rood glas, dan wordt er een klein wit licht direct voor de patiënt gezet. Dan moet de patiënt het licht volgen in alle 9 oogbewegingen, hierbij moet de patiënt melden waar hij de witte en rode beelden ziet. Normaliter overlappen deze twee beelden elkaar bij alle bewegingen. De richting van het dubbelzien kan je er ook mee bepalen. Exotropie is een abnormale laterale richtingsafwijking van één oog. Een mediale afwijking heet esotropie. Hypetropie is een oog dat hoger staat dan normaal. Voor subtiele standsafwijkingen kun je de volgende testen uitvoeren: licht in beide ogen schijnen van voren (normaal: symmetrische cornea reflectie) en een oog afdekken en dan weer niet (dek je een oog af, terwijl het in de oogrichting kijkt van een zwakke spier, dan kan het iets terugdraaien naar de neutrale oogstand). Deze laatste milde verlate zwakte bij een afgedekt oog, heet (exo/eso)forie. Bij jonge kinderen kan een congenitale oogspierzwakte leiden tot strabismus en later amblyopie. Snel ingrijpen is dan dus noodzakelijk.

Wat veroorzaakt verlamming van de n. oculomotoris?

Symptomen die hierbij passen zijn: complete ptosis (m. levator palpebrae superior), wijde lichtstijve pupil (parasympatische vezels die via oculomotoris lopen) en het oog kan alleen intorsie, een beetje abductie en een beetje depressie verrichten (alle externe oogspieren, behalve m.rectus lateralis en obliquus superior doen het niet). Het oog kan naar beneden gaan staan in rust. Verschillende combinaties van deze klachten treden op bij incomplete laesies. Bij gedeeltelijke laesies kunnen bijv. de oogbewegingen best goed blijven en de pupil juist aangedaan zijn of omgekeerd. Er ontstaat diagonaal gekruist dubbelzien, met name bij mediaal of naar boven kijken. Convergentie is ook verstoord, wat ervoor kan zorgen dat de patiënt erger dubbelziet bij voorwerpen die dichtbij staan en minder erg bij voorwerpen die veraf staan. Via de n. oculomotoris lopen immers ook de postganglionaire parasympatische vezels van de m. ciliaris die de lensaccomodatie regelt. Veelvoorkomende oorzaken van een oculomotorisparese zijn: hoofdtrauma, microvasculaire neuropathie (bijv. door hoge bloeddruk, te hoog vetgehalte) en neuropathie door diabetes. Een andere belangrijke oorzaak is een aneursyma, meestal van de a. communicans posterior. Overige oorzaken: infectie, veneuze trombose of tumor in de subarachnoïdale ruimte, orbita of sinus cavernosus, temporaallob herniatie over het tentorium. Bij dat laatste ontstaat er natuurlijk ook vaak hemiplegie en coma. Bij kinderen kan een ophtalmoplegische migraine leiden tot een omkeerbare oculomotorisparese. Middenhersenen laesies (lacunaire infarcten) of infarcten in de oculomotoriskernen kan ook voor verlamming zorgen. Neuromusculaire overgang- of spierziekten kunnen voor klachten zorgen die lijken op een oculomotorisparese. Als een patiënt met een oculomotorisverlamming binnenkomt, moet je meteen denken aan een aneurysma van de a. communicans posterior tot het tegendeel bewezen is. Klassieke hierbij is een pijnlijke n. oculomotorisparese waarbij er stoornis van de pupil is. Verricht snel een spoed CT-angiogram of MRa. Een complete oculomotorisparese, die pijnloos is zonder pupilsymptomen wijst meestal op een microvasculair/diabetes neuropathie. En niet op aneurysmata (zelden wel). Een gedeeltelijke parese zonder pupilsymptomen kan, vooral met pijn, door een gedeeltelijke verdrukking door een aneurysma komen. In sommige gevallen is alléén de ramus superior of inferior aangedaan. Uitval van de ramus superior (m.rectus superior en levator palpebrae) komt meestal door een massa in of rond de orbita.

Wat veroorzaakt verlamming van de n. trochlearis?

Depressie en intorsie van het oog valt uit, er ontstaat dus verticaal dubbelzien. Als de uitval hevig is, ontstaat er hypertropie in het oog, extorsie kan ook maar is meestal niet goed te waar te nemen. De verticale diplopie verergert bij het naar beneden kijken en naar nasaal. Het dubbelzien vermindert als de patiënt het hoofd van het aangedane oog af draait en naar boven kijkt, dus de kin naar beneden houdt (respectievelijk, wordt de extorsie en hypertropie zo opgeheven). Onthoudt: de hoofdbewegingen zijn altijd in de richting van de uitgevallen oogbewegingen. Door de vier dikgedrukte termen aan te tonen kun je de diagnose stellen. Als test kun je een horizontale lijn voor de patiënt houden, de patiënt ziet dan 2 lijnen, met de lagere lijn schuin. Samen vormen deze lijnen een pijlpunt, die richting het aangedane oog wijst. Oorzaken: vaak trauma (de zenuw is dun en heeft een lang traject, dus is snel aangedaan), processen in de sinus cavernosus/orbita/subarachnoïdale ruimte (infectie, aneurysma, tumor), stoornis van de bloedvaten en neoplasmata in de middenhersenen of bij het tectum, congenitale trochlearisparese en onbekend (microvasculair: diabetes?). De congenitale parese is erg lang latent aanwezig, waardoor er pas als de patient volwassen is ineens dubbelzien ontstaan. Overige oorzaken van verticaal dubbelzien: myasthenia gravis, externe spierafwijkingen, skew deviatie en ramus superior n. oculomotoris laesie. Een verticale afwijking in de stand van de ogen door een supranucleaire afwijking heet skew deviatie. Oorzaken hiervan zijn laesies van de hersenstam, kleine hersenen of binnenoor. Bij skew deviatie is het verticaal afwijkingen meestal constant in alle oogrichtingen. De schuinstand van het hoofd kan ook komen door: meningitis, torticollis, inspiciënt tonsillaire herniatie, laesies van de kleine hersenen.

Wat veroorzaakt verlamming van de n.abducens?

Hierbij ontstaat horizontaal (ongekruiste) dubbelzien dubbelbeelden (in de verte). Het dubbelzien verergert bij het kijken in de verte en vermindert bij het kijken naar objecten dichtbij. Het verergert ook bij abductie van het oog. Normale abductie is niet mogelijk. In sommige gevallen is er esotropie van het aangedane oog. Ter compensatie kan het hoofd naar het aangedane oog toe bewogen worden. Abducensparese is een belangrijk waarschuwingssymptoom van infra- of supratentoriale tumoren, hydrocefalus, pseudohersentumor en andere intracraniale laesies. De n. abducens is immers erg gevoelig voor verdrukking van boven door verhoogde intracraniële druk, de verlamming kan hierbij een- of tweezijdig zijn. Weer kan het ook veroorzaakt worden door processen in de orbita/subarachnoïdale ruimte en sinus cavernosus. Oorzaken als aneurysmata, infectie, neoplasmata, hoofdtrauma, sinus cavernosus trombose en ontsteking komen vaak voor. De oorzaak is niet altijd bekend en dit kan door microvasculaire neuropathie (diabetes) komen. Laesies in de pons die de uittredende abducensvezels aantasten zorgen voor een verminderde abductie in het ipsilaterale oog. Dit kan lijken op een perifere n. abducens laesie, maar dit geeft een parese in bij naar voren kijken in de richting van de laesie. Bewegingen van beide ogen in één richting zijn verminderd, vaak is er ook ipsilaterale gezichtsspierzwakte, omdat de facialiskern dicht ligt. Overige oorzaken van horizontaal dubbelzien: externe oogspierziekten (schildklierziekte, tumor, orbita trauma, ontsteking).

Hoe werken pupillen en andere autonome banen?

De parasympatische vezels zorgen voor pupilconstrictie en de sympathische vezels voor pupilverwijding. Licht komt het oog binnen, dan worden de retinale ganglioncellen actief. Het signaal gaat verder naar beide tracti optica (vanwege de chiasma kruising). Dan gaan de extrageniculate vezels richting de arm (brachium) van de superior colliculus, het laterale geniculatum passerend. Dan gaan ze naar de pretectale kernen, daar synapseren ze en gaan de vezels door naar beiden Edinger-Westphalkernen. Deze bevatten preganglionaire parasympatische zenuwcellen. Een deel van de kruisende vezels gaat naar de posterior commissuur. De parasympatische vezels gaan van de Edinger-Westphalkernen, bilateraal, via de n. oculomotoris naar het ganglion cilliare. Hiervandaan gaan postganglionaire parasympatische vezels naar de pupillen, die dan vernauwen. Licht in één oog zorgt voor een directe reactie in hetzelfde oog en een indirecte reactie in het andere oog. De accommodatie reflex treedt op als een object van veraf dichterbij beweegt. Hierbij ontstaat er accommodatie van de lensspier, pupilvernauwing en convergentie van de ogen. Het wordt geactiveerd door licht signalen naar de visuele cortex, hiervandaan worden de pretectale kernen weer geactiveerd, waardoor via de eerder beschreven parasympatische banen, pupilvernauwing ontstaat. Het aanspannen van de m. ciliare (lensspier) wordt door dezelfde banen in werking gesteld.

De sympathische banen lopen vanaf een aantal hypothalamuskernen naar beneden in de laterale hersenstam en cervicale ruggenmerg naar T1 en T2. Deze banen activeren preganglionaire sympathische zenuwcellen in de intermediolaterale celkolom van de bovenste thoracale streng. Via de ventrale zenuwwortel T1 en T2 treden de axonen van de preganglionaire sympathische zenuwcellen uit. Ze lopen dan boven de longtop langs en komen samen met de truncus sympathicus via de witte rami communicantes. Ze stijgen dan omhoog om te synapseren in het superieure cervicale ganglion. Hiervandaan gaan er postganglionaire sympathische vezels omhoog door de plexus caroticus, langs de wanden van de a. carotis interna. En dan naar de sinus cavernosus, eindigend in de m. dilator pupillae. De sympathische banen zorgen ook voor de gladde spieren van de m. tarsalis superior (van Müller). Deze eleveren het bovenste ooglid. Als er teveel signalen van de sympathicus komen, staan de ogen wijd open. Sympathische vezels van de eerder genoemde baan innerveren ook de gladde spieren van de m. orbitalis (van Müller) (voorkomt dat het oog in de orbita zakt), zweetklieren van de nek en gezicht en huidarteriën. Bij het Horners syndroom zijn deze functies gestoord.

Wat voor pupilafwijkingen zijn er?

Afwijkingen in de pupil kan aan beide ogen of in één oog (met anisocorie als gevolg) ontstaan.

Fysiologische anisocorie

Een kleine anisocorie van 0,6 millimeter is bij 20% van de bevolking aanwezig. Er zijn geen andere oogafwijkingen (vertraagde verwijding bijv.)

N. oculomotoris stoornis
Een laesie in de efferente parasympatische baan van de Edinger-Westphal kern naar de pupilvernauwingspier, leidt tot een vernauwingsstoornis. Het gevolg: een eenzijdig wijde pupil. Er is een verminderde of afwezige directe pupilreflex in het aangedane oog en een verminderde of afwezige indirecte reflex als licht in de niet-aangedane oog geschenen wordt. Anisocorie valt meer op bij licht, dan in het donker.

Syndroom van Horner
Klassieke symptomen zijn: miosis, anhidrosis en ptosis. Anisocorie valt hierbij meer op in het donker dan bij licht. Er bestaat nog een directe en indirecte pupilreflex van de aangedane pupil, maar er is een vertraagde verwijding t.o.v. de normale pupil wanneer het licht er niet meer op geschenen wordt. Een ciliospinale reflextest kan helpen bij een vermoeden: knijp in de nek, dit zorgt voor sympathische signalen, leidend tot pupilverwijding in de niet-aangedane kant. In de kant met Horner, gebeurt er niets. Anhidrosis kun je ontdekken door met de vinger over het gezicht te gaan. De kant zonder zweet voelt gladder aan. Horner ontstaat door een laesie in het eerder beschreven sympathische traject. Mogelijke locaties van de stoornis, zijn dus: orbita, sinus cavernosus, plexus caroticus, truncus sympathicus, T1 en T2 zenuwwortels, ruggenmerg en laterale hypothalamus of hersenstam. Preganglionaire laesies kun je onderscheiden van postganglionaire laesies m.b.v. hydroxyamfetamine oogdruppels. Deze stimuleren de afscheiding van noradrenaline en zo pupilverwijding bij preganglionaire laesies, maar niet bij postganglionaire laesies. En bij postganglionaire laesies ontstaat meestal geen anhidrosis (aftakking vóór het ganglion). “Pons pupillen” ontstaan soms bij grote, tweezijdige laesies van de pons, hierbij zijn beide pupillen klein (vermoedelijk doordat de afdalende sympathische banen verstoord worden), maar reageren op licht.

Marcus Gunn pupil

Bij het afferente pupil reflex defect is de directe pupilreflex in het aangedane oog afwezig of verminderd, maar het indirecte pupilreflex normaal. Ook wel Marcus Gunn pupil genoemd. Met de ‘swinging flashlight test’ kun je dit bevestigen. Wanneer je het licht van de normale naar het aangedane oog beweegt, kan je het duidelijk zien: de aangedane pupil verwijd door het licht. Je moet deze abnormale verwijding niet verwarren met de normale, kortdurende heen en weer verandering in pupilgrootte die soms voorkomt als er licht in het oog geschenen wordt. Dit heet hippus. Marcus Gunn pupil ontstaat door een n. opticus laesie (niet in of na het chiasma). Er ontstaat bij een retina afwijking of n. opticus laesie geen anisocorie.

Licht-dichtbij dissociatie

Er ontstaat hierbij minder vernauwing bij licht dan bij accommodatie. Het Argyll Robertson pupil is hier een typisch voorbeeld van. Hierbij is er ook sprake van kleine en niet-regulaire pupillen. Het is geassocieerd met neurolues. Bij het syndroom van Adie, syndroom van Parinaud en diabetes zie je de dissociatie ook.

Syndroom van Adie

Bij een myotonische pupil van Adie is er sprake van een verwijde pupil, die niet goed reageert op licht. Karakteristiek voor deze stoornis is een degeneratie van de postganglionaire sympathische zenuwcellen of het ganglion ciliare. Met de accomodatiereflex kan er wat vernauwing uitgelokt worden, maar de pupil blijft vernauwd en verwijdt heel langzaam. Dit wordt een tonische of myotonische pupil genoemd. De oorzaak is onbekend.

Corectopie van de middenhersenen

Laesies van de middenhersenen kunnen in sommige gevallen een ongewone abnormaliteit van de pupillen veroorzaken, waardoor de pupil een irregulaire vorm, weg van het midden aanneemt. Het is een relatief zeldzame afwijking.

Wat is pupilvernauwing en verwijding door medicatie?

Opiaten kunnen voor tweezijdig sterk vernauwde pupillen zorgen, een barbituraten overdosis ook. Anticholinergica die op muscarine receptoren inwerken (zoals atropine en scopolamine) zorgen voor verwijde pupillen. Het kan eenzijdig zijn bij lokale medicatie in het oog. Je kunt pilocarpine 1% gebruiken om een onderscheid tussen anticholinergica of een laesie te maken. Zoek vermoedelijke hersenstamlaesies altijd uit, maar houd rekening met de invloed van medicatie wat dus ook de oorzaak kan zijn van pupilafwijkingen, doe altijd een toxische screening.

Hangend bovenooglid

Het sluiten van het oog wordt gedaan door de m. orbicularis oculi, geïnnerveerd door de n. facialis. Het openen wordt gedaan door de gladde spieren van Müller (sympathische vezels) en de dwarsgestreepte spieren van de m. levator palpebrae superior (n. oculomotoris). De m. frontalis, door de n. facialis heeft een accessoir aandeel. Oorzaken van ptosis (een- of tweezijdig) zijn: een orbitale massa, myasthenia gravis, overbodige huidflappen door ouderdom, syndroom van Horner en een n. oculomotorisparese. Ptosis bij myasthenia gravis neemt toe bij vermoeidheid en verergert bij een behouden blik naar boven. Tweezijdige gesloten ogen of ptosis, zonder bewusteloosheid, wordt veroorzaakt door: ernstige neuromusculaire aandoeningen, oculomotoriskern laesies, bewuste oogsluiting bij fotofobie (meningitis, migraine) en pariëtale lob infarct. Bij m. orbicularis oculi zwakte (facialislaesie) kan een verwijde palpebraal fissuur ontstaan, dat in het andere oog aangezien kan worden voor een ptosis. Let er hierbij op dat bij ptosis het bovenste ooglid iets over de iris zakt in het aangedane oog.

Sinus carvernosus en orbita

De sinus cavernosus bestaat uit een verzameling veneuze sinusoïden die veneus bloed ontvangen van het oog en oppervlakkige cortex. Van de sinus gaat het naar de v. jugularis. De sinussen bevinden zich aan beide kanten van de hypofyse en liggen tussen de durale en periosteale lagen van de dura mater. De sinus cavernosus omringt de a. carotis interna, n. oculomotoris, n. abducens, n. trochlearis, n. opthalmicus en deels de n. maxillaris. De sympathische vezels die deels meelopen met de plexus carotis lopen ook voor een deel door de sinus cavernosus. Boven de sinus cavernosus loopt de n. opticus die via het optische kanaal de orbitale apex binnentreedt. Het gebied waar alle arteriën, venen en zenuwen van de orbita samenkomen, voordat ze via het optische kanaal met de intracraniale ruimte communiceren, heet de orbitale apex.

Orbitale apex laesie en sinus cavernosus laesie

Een laesie in de sinus cavernosus kan alle zenuwen die erdoor lopen aantasten of enkele. Een algehele laesie verstoort de n. abducens, trochlearis en oculomotoris. Je ziet: totale verlamming van de oogspieren, meestal met een verwijde stijve pupil. Als de n. opthalmicus en deels de maxillaris meedoen treedt er gevoelsuitval op van hun gebieden. Er kan ook sprake zijn van het syndroom van Horner, maar dat is moeilijk te zien als de oculomotoris ook is uitgevallen. Het sinus cavernosus syndroom zorgt dus voor uitval van de 3e, 4e, de eerste aftakking van de 5e en de 6e hersenzenuw. Oorzaken kunnen zijn: hypofyse apoplexie (bloeding in de hypofyse), meningioom, hypofysetumor, caverneuze sinus trombose door een bacteriële infectie, uitzaaiingen, nasopharyngeale tumoren, een arterioveneuze fistel tussen cavernosus en carotis en schimmelinfecties. Bij deze fistels of aneurysmata is de n. abducens vaak betrokken (ligt het dichtste bij de carotis).

Bij een laesie van de orbitale apex is de n. opticus, naast de eerder genoemde zenuwen, vaak ook aangedaan. Je ziet dan: visusverlies meestal met proptosis en de n. maxillaris blijft intact (is al via foramen rotundum vertrokken). Het orbitale apex syndroom zorgt dus voor uitval van de 2e, 3e, 4e, eerste aftakking van de 5e en de 6e hersenzenuw. Oorzaken kunnen zijn: cellulitis orbitae, idiopathische granulomateuze ziekte (orbitale myositis/pseudotumor), uitzaaiingen en schimmelinfecties. Deze twee benoemde syndromen zijn spoedgevallen, doe een spoed orbitomie en biopsie als er niets blijkt uit de liquorpunctie en contrast MRI.

Supranucleaire banen

De supranucleaire banen lopen van de hersenstam en kleine hersenen naar de voorhersenen. Ze beïnvloeden de kernen van abducens, oculomotoris en trochlearis. Hiermee worden verticale, horizontale en vergente oogbewegingen. Er zijn 4 typen oogbewegingen te onderscheiden: de volg oogbewegingen (hierdoor kun je bewegende objecten stabiel bekijken, ze gaan onbewust, snelheid tot 100°/s), de reflex oogbewegingen (nystagmus en het vestibulo-oculair reflex), vergentie oogbewegingen (snelheid 20°/s, fixatie van beide ogen mogelijk bij het dichtbij komen en weg gaan van een object) en de oogsprongen oftewel saccaden (snelle oogbewegingen tot 700°/s, waarmee je een bepaald gebied in het gezichtsveld kan brengen).

Wat zijn horizontale oogbewegingen?

Horizontale oogbewegingen worden gedaan door de rectus lateralis en medialis. De kernen van de abducens, vestibulocochlearis en trochlearis zijn met elkaar verbonden door de mediale longitudinale fasciculus (MLF). Door deze verbindingen kunnen de oogbewegingen op elkaar afgestemd worden. De abducens kern controleert ook de horizontale oogbewegingen beiderzijds in de richting, die aan dezelfde kant van de kern zit. Dus sommige zenuwcellen in de abducenskern lopen naar de ipsilaterale m.rectus lateralis en anderen via de eerder genoemde fasciculus naar de contralaterale oculomotoris kern en dus contralaterale m. rectus medialis. De paramediane pontine reticulaire formatie is een extra horizontale blik centrum. Het zorgt voor signalen van de cortex en andere banen naar de abducenskern. Dit resulteert in een laterale horizontale blik.

Hersenstamlaesies met horizontale afwijkingen

Een n.abducens laesie resulteert in een abductiebeperking in het ipsilaterale oog. Een abducenskern laesie in een ipsilaterale laterale blik parese beiderzijds. Laesies van het paramediane pontine reticulaire formatie geven dat ook. Een internucleaire ophtalmoplegie ontstaat als bij een laesie van de mediale longitudinale fasciculus. Hierbij kan het oog ipsilateraal van de laesie niet volledig adducteren bij een horizontale blikrichting, er is ook nystagmus van het tegengestelde oog. Veelvoorkomende oorzaken van deze laesie zijn: ponsinfarcten, neoplasmata en MS. Is de mediale fasciculus én de PPRF of abducenskern is aangedaan, dan ontstaat het 1-en ½ syndroom. Er is sprake van een ipsilaterale laterale blik parese en een ipsilaterale INO. Het ipsilaterale oog kan helemaal niet horizontaal bewegen. Het contralaterale oog verlies de helft van zijn bewegingen en kan alleen maar abduceren.

Wat zijn verticale en vergente oogbewegingen?

Verticale oogbewegingen worden door de m. rectus superior en inferior en de m. obliquus superior en inferior verzorgd. De verticale oogbewegingen worden door de pretectale regio en reticulaire formatie van de rostrale middenhersenen gecoördineerd. Het dorsale gedeelte hiervan doet de blik naar boven en het ventrale gedeelte de blik naar beneden. Laesies (tumor bijv.) dorsaal zorgen voor een verstoorde blik naar boven. Laesies ventraal een verstoorde blik naar beneden.

Van de rostrale intersitiele kern van de MLF wordt gedacht dat hij een rol speelt bij het naar beneden kijken. Andere kernen die mogelijk ook een aandeel hebben zijn: de interstitiële kern van Cajal en de Darkschewitsch kern. Bij progressieve supranucleaire parese ontstaat er middenhersenenatrofie en een beperking van de verticale oogbewegingen. Bij het locked-in-syndroom kunnen alle lichaamsbewegingen en horizontale oogbewegingen aangedaan zijn. Soms zijn hierbij de verticale oogbewegingen intact, doordat die centra in de hersenstam gespaard zijn gebleven. Beweging van de bovenste oogleden is gelinked aan de verticale oogbewegingen, de hypothese is dat de M groep zenuwcellen vlakbij de verticale oogbeweging kernen hiervoor zorgen. De ooglidafwijkingen bij Parinaud ontstaan waarschijnlijk door schade aan deze zenuwcellen.

Vergentie bewegingen krijgen input vanuit de visuele banen in de occipitale en pariëtale cortex, de hersenstamkernen die ze coördineren zijn nog niet onbekend. De mediale recti zorgt voor convergentie. De laterale recti voor divergentie.

Het syndroom van Parinaud

Klachten zijn: grote, irregulaire pupillen met licht-dichtbij dissociatie, ooglid afwijkingen (tweezijdige ptosis, tweezijdig lid retractie; teken van Collier), een verstoorde convergentie soms met convergentie-retractie nystagmus en verstoorde verticale oogbewegingen, met name bij naar boven kijken. Vaak wordt dit syndroom veroorzaakt door: MS, hydrocephalus, vasculaire afwijkingen van de middenhersenen en tumoren rond de epifyse. Bij hydrocephalus kunnen de ogen naar binnen en beneden gaan staan: het ondergaande zon teken. Vooral bij kinderen zie je dit. Oorzaak: Parinaud en abducensparese. Bij thalamus bloeding kun je een vergelijkbare binnen en beneden afwijking van de ogen zien.

Wat zijn de frontale hersenen?

Vanuit de voorhersenen lopen er een aantal banen naar de hersenstam die oogbewegingen coördineren. Deze banen verlopen via de superior colliculus of direct naar de hersenstam. Het best bekende gebied dat de oogbewegingen coördineert zijn de frontale oogvelden, zij zorgen voor saccaden in contralaterale richting via de contralaterale PPRF. Meer naar achter, vind je de parieto-occipito-temporale cortex gebieden, die verzorgen de ipsilaterale volgbewegingen via de kleine hersenen, PPRF en vestibulaire kernen. Ze verzorgen ook gedeeltelijk de contralaterale oogbewegingen. De visuele cortex en visuele associatieve cortex spelen ook een belangrijke rol in oogbewegingen. De basale ganglia spelen een controlerende rol.

Als een hemisfeer beschadigd raakt, ontstaat er vaak een blikrichting voorkeur naar de kant van de laesie. De contralaterale bewegingen worden immers beschadigd. Vaak is er daarbij ook sprake van spierzwakte contralateraal van de laesie, waardoor de ogen van de kant met spierzwakte afkijken. Er is paralyse aan bijv. de rechterkant van het lichaam en de ogen kijken naar links. Bij bepaalde klinische situaties kijken de ogen naar de kant van de zwakte toe, dit heet verkeerde-richting-ogen. Er is dus paralyse aan bijv. de rechterkant van het lichaam en de ogen kijken naar rechts. Oorzaken: laesies in de ponsbasis en tegmentum, thalamus bloeding en insultactiviteit in de cortex.

Willekeurige en reflex oogbewegingen

De cervicale spinale proprioceptoren, de kleine hersenen en vestibulaire kernen beïnvloeden continue willekeurige oogbewegingen en oogreflexen, zoals de optokinetische nystagmus en het vestibulo-oculaire reflex. De langzame fase van de nystagmus wordt door de ipsilaterale posterieure cortex beïnvloedt, met verbindingen naar de flocculonodulaire lob van de kleine hersenen en de vestibulaire kern. De snelle fase door de frontale oogvelden, die naar de contralaterale PPRF signaleren. De langzame fase wordt dus door laesies in de oogvolgbanen verstoord. De snelle fase door laesies in de saccadenbanen of frontale cortex. Het testen van de nystagmus kan je dus helpen om subtiele afwijkingen in de oogbewegingsbanen op te sporen. Het vestibulo-oculaire reflex zorgt ervoor dat de ogen zich kunnen stabiliseren op een object, tijdens het bewegen van het hoofd en lichaam. Van de vestibulaire kernen worden komt er input, die via de MLF, de extra-oculaire kernen controleren. Je kunt bij comapatiënten de intactheid van de hersenstambanen bepalen door de oculocephalische manoeuvres of met calorische koud watertest. Bij gezonde mensen zorgen de banen naar de kleine hersenen (nodulus en flocculus) ervoor dat visuele fixatie de vestibulo-cochleaire reflex kan overrulen. Visuele fixatie kan ook nystagmus onderdrukken op eenzelfde manier. Bij de vestibulo-oculaire reflex onderdrukkingstest, moet een patiënt op een voorwerp fixeren dat meebeweegt, terwijl het hoofd rond gedraaid wordt. Ontstaat er nystagmus, dan is er een stoornis van de kleine hersenen. Ten slotte zijn proprioceptieve signalen ook nodig om de ogen op een object te kunnen fixeren. Vooral tijdens nek en hoofd bewegingen.

 

Hoe zit de anatomie van het oog in elkaar?

 

Het oog heeft externe (oogbewegingen) en interne oogspieren(lensaccommodatie en pupilcontrole). Oogbewegingsbanen en stoornissen worden ingedeeld op twee niveaus: supranucleair en nucleair/infranucleair. De nucleaire en infranucleaire banen, dus de banen van de hersenstamkernen van de n.oculomotoris, n.trochlearis en n.abducens (III, IV, VI). En de supranucleaire banen, dit zijn banen in de hersenstam en frontale hersenen die verbonden zijn met de eerder genoemde kernen.

Elk oog heeft 6 externe oogspieren. De m. rectus medialis zorgt voor een beweging naar nasaal. De m. rectus lateralis naar temporaal. De m. rectus superior naar boven en de m. rectus inferior naar beneden. De m. obliquus superior zorgt voor intorsie, de m. obliquus inferior voor extorsie. De obliquus superior hecht aan het oog na een bochtje via de trochlea (een fibreus loepje aan het mediale orbitabot), de obliquus inferior niet. Wat voor oogbeweging er wordt gemaakt, is afhankelijk van de oogas. De m. rectus superior kan voor zowel intorsie, als elevatie zorgen bij naar voren kijken. De m. rectus inferior kan bij naar voren kijken ook zorgen voor depressie en extorsie. Bij abductie (naar temporaal) van 23 graden van het oog kan de m. rectus superior voor pure elevatie zorgen. Bij adductie vindt er meer intorsie plaats. Is het oog geadduceerd, dan kan de m. obliquus superior zorgen voor depressie en de m. obliquus inferior voor elevatie. Bij abductie zorgt de m. obliquus superior met name voor intorsie en de inferior voor extorsie.

De nervi oculomotoris, trochlearis en abducens lopen vanaf de hersenstam, door de sinus cavernosus, naar de orbita via de fissura orbitales superior. De n. oculomotoris innerveert alle externe spieren, behalve de m. rectus lateralis (n. abducens) en de m. obliquus superior (n. trochlearis). De ramus superior van de n. oculomotoris innerveert de m. rectus superior, de m. levator palpebrae superior (ooglid elevator). De ramus inferior innerveert de m .rectus medialis, m. rectus inferior, m. obliquus inferior. Deze ramus bevat ook preganglionaire parasympatische vezels die naar de m. pupillae constrictor worden vervoerd. De hersenstam kernen van n. III, IV en VI behoren samen met de n. XII tot de somatisch motorische kernen. Ze liggen allemaal in het midden naast de ventrikels. Hun vezels treden ventraal uit, behalve de trochlearis die dorsaal uittreedt.

De oculomotoriskernen bevinden zich in de bovenste middenhersenen, ventraal van de het periaquaductale grijze stof. Tussen de a. cerebri posterior en a. cerebri superior treedt de n. oculomotoris uit deze kern via de fossa interpeduncularis. De Edinger-Westphal kern is een subkern van de oculomotoriskernen en bevat parasympatische vezels. Deze vezels zorgen voor pupilvernauwing en bevinden zich mediaal en oppervlakkig in de n. oculomotoris. De vezels zijn gevoelig voor aneurysmata, met name van de nabijgelegen a .communicans posterior. De subkernen zijn klinisch niet zo relevant op twee dingen na:

  • Eenzijdige zwakte van de pupilvernauwing of m. levator palpebrae superior kan niet komen door een eenzijdige laesie van de oculomotoriskern.

  • Een oculomotoriskern laesie zorgt voor aantasting van de contralaterale m. rectus superior en de ipsilaterale m. rectus suprior.

Klinisch zal een laesie van de oculomotoriskern dus nooit zorgen voor een eenzijdige ptosis, eenzijdige m. rectus superior verlamming of eenzijdige verwijde, niet-reagerende pupil. De trochleariskernen bevinden zich in de lage middenhersenen, ze liggen ook ventraal van het periaquaductale grijze stof. Het zijn de enige hersenzenuwen die dorsaal uittreden en elkaar kruisen. Ze lopen naar beneden, kruisen dan naar de tegenovergestelde kant en treden uit bij het mediale anterieure velum van de hersenstam (hier zijn ze gevoelig voor verdrukking door kleine hersenentumoren), dan gaan ze onder het tentorium cerebelli door, via hetzelfde eerder genoemde traject naar de orbita. De abducenskernen liggen op de vloer van het vierde ventrikel. Het treedt uit bij de kruising tussen de pons en medulla, dan via de subarachnoïdale ruimte naar het kanaal van Dorello. Dit bevindt zich tussen de dura en schedel. Via Dorello maakt het een scherpe bocht bij de petrouze punt van het temporaal bot en dan net zoals de andere zenuwen via hetzelfde traject naar de orbita. Deze zenuw is gevoelig voor druk van bovenaf, zoals bij een verhoogde intracraniële druk.

 

Hoe zien de structuren en vascularisatie van de hersenstam eruit? - Chapter 14

 

 

Hoe is de hersenstam opgebouwd?

De hersenstam kan functioneel worden verdeeld in 4 structuren: de nuclei van de hersenzenuwen en gerelateerde structuren, de lange banen, het cerebellum en de reticulaire formatie met gerelateerde structuren.

De hersenstam bestaat uit drie verschillende anatomische structuren: de middenhersenen, pons en medulla oblongata (van boven naar beneden). De hersenstam kan ook van voor naar achter worden verdeeld in verschillende structuren: de basis, het tegmentum en het tectum. Het tectum ligt dorsaal van de aquaductus cerebri, het tegmentum ligt voor de aquaductus cerebri en vierde ventrikel. Hierin zitten de meeste nuclei en de reticulaire formatie. De basis bevat vooral veel vezels van de lange banen.

Net als in het ruggenmerg liggen ook in de hersenstam de motorische nuclei ventraal en de sensorische nuclei dorsaal. De scheiding wordt gevormd door de sulcus limitans.

Wat zijn de nuclei van de hersenzenuwen?

Er zijn drie longitudinale kolommen van motorische nuclei en drie longitudinale kolommen van sensorische nuclei.

Motorische nuclei

De hersenzenuwen met somatische motorische nuclei zijn de n. oculomotorius (n. III), de n. trochlearis (n. IV), de n. abducens (n. VI) en de n. hypoglossus (n. XII).

Naast de somatische motorische nuclei zijn er ook branchiale (uit de kieuwbogen) motorische nuclei. Dit zijn de motorisch nuclei van de n. trigeminus (n. V), de n. facialis (n. VII), de nuclei ambiguus en de spinale nuclei accessorius (n. XI). De nuclei ambiguus zijn nuclei van twee hersenzenuwen gecombineerd: de n. glosspharyngeus (n. IX) en de n. vagus (n. X). de spinale nuclei accessorius liggen niet meer in de hersenstam, maar in het cervicale ruggenmerg.

Er zijn vier hersenzenuwen die parasympathische vezels hebben. Deze vezels lopen vaak met de motorische vezels van die hersenzenuw mee. De parasympathische hersenzenuwen zijn de n. oculomotorius, de n. facialis, de n. glossopharyngeus en de n. vagus. De bijbehorende parasympathische nuclei zijn de Edinger-Westphal nuclei (n. III), de superior speekselklier nuclei (n. VII), de inferior speekselklier nuclei (n. IX) en de dorsale motor nucleus van de n. X.

Sensorische nuclei

De n. trigeminus (n. V), de n. facialis (n. VII), de n. glossopharyngeus (n. IX) en de n. vagus (n. X) hebben sensorische vezels. Al deze vezels komen samen in het complex van de nuclei trigeminus. Dit complex bestaat uit drie kernen: de mesencefale nucleus, de hoofdnucleus en de spinale trigeminus nucleus (in de medulla). Alle drie de kernen spelen een rol bij verschillende vormen van gevoel. De mesencefale nucleus is belangrijk voor de proprioceptie, de hoofd nucleus is voor de fijne tast en de spinale trigeminus nucleus is belangrijk voor de vitale sensibiliteit (pijn en temperatuur).

De n. vestibulocochlearis (n. VIII) is verantwoordelijk voor de innervatie van het vestibulum en de cochlea. Deze zenuwen geven speciale somatische sensorische input aan de nuclei cochlearis en de nuclei vestibularis. De nucleus cochlearis bestaat uit een dorsale en ventrale kern. De nucleus vestibularis bestaat uit 4 nuclei: superior, inferior, mediaal en lateraal. Deze nuclei krijgen informatie uit het vestibulum over de positie van het hoofd en de versnelling in de beweging. Tussen de nuclei vestibularis en de nuclei die betrokken zijn bij oogbewegingen loopt de mediale longitudinale fasciculus. Deze fasciculus speelt een rol bij de vestibulo-oculaire reflexen (denk aan nystagmus bij draaiduizeligheid).

De viscerale sensorische vezels gaan naar de nuclei solitarius. Rostraal in deze nucleus komen de speciale viscerale afferente banen voor de smaak, waarbij hersenzenuw VII, IX en X een rol spelen. Caudaal komen de algemene viscerale afferente vezels.

Wat zijn de lange banen?

De tractus corticospinalis loopt ventraal in de pons. In de ventrale medulla loopt deze baan verder als de pyramide baan. Cervicomedullair kruist de pyramide baan, waarna het lateraal door het ruggenmerg loopt.

De gnostische sensibiliteit loopt in de achterstreng. De perifere neuronen vormen een synaps wanneer ze het ruggenmerg inkomen. Dit gebeurt in de nucleus gracilis voor de benen en de nucleus cuneatus voor de armen. In de nek kruisen de vezels naar de contralaterale zijde en lopen ze verder als de mediale lemniscus naar de thalamus. Daar vormen ze opnieuw een synaps en lopen de vezels naar de sensorische cortex.

De vitale sensibiliteit (tractus spinothalamicus) loopt in de voorzijstreng. Deze perifere vezels kruisen direct naar contralateraal wanneer ze het ruggenmerg inkomen.

Wat is locked-in syndroom?

Wanneer alle motorische lange banen zijn uitgevallen, maar de sensorische banen en de cognitie intact zijn, spreekt men van het locked-in syndroom. De oorzaak is vaan een infarct ventraal in de pons, waarbij de tracti corticospinalis en corticobulbaris worden aangedaan. Vaak zijn er nog wel verticale oogbewegingen mogelijk, waarvan gebruik kan worden gemaakt in de communicatie met de patiënt.

Wat is het cerebellum?

Het cerebellum speelt een rol in het coördineren van bewegingen. Wanneer er een laesie zit in het cerebellum, kan dit leiden tot ataxie: ongecoördineerde bewegingen. Deze ataxie vindt ipsilateraal aan de laesie plaats, omdat de vezels twee keer kruisen.

Het cerebellum is via drie pedunculi verbonden aan de hersenstam. De superior pedunculus bevat vooral output vanuit het cerebellum. De middelste is de grootste pedunculus. Deze bevat vooral input uit de verschillende nuclei in de hersenstam. De inferior peduculus bevat ook input voor het cerebellum, uit het ruggenmerg.

Wat is reticulaire formatie?

De reticulaire formatie loopt in de gehele lengte van de hersenstam door het tegmentum. Het bovenste deel is vooral belangrijk in het reguleren van het bewustzijn. Het onderste deel is belangrijk in het reguleren van motorische, reflexmatige en autonome functies.

Wat is rostrale reticulaire formatie?

Het bewustzijn

Het bewustzijn is een moeilijk te omschrijven iets. De mate van bewustzijn wordt bepaald door de alertheid, aandacht en bewustzijn van de omgeving/oriëntatie. Systemen die hierbij betrokken zijn, zijn de mediale en laterale frontopariëtale associatie cortex en arousal circuits.

Zoals eerder omschreven is het rostrale deel van de reticulaire formatie betrokken bij het bewustzijn. Laesies in dit deel van de reticulaire formatie en laesies van het mediale diencephalon kunnen dan ook coma veroorzaken. Ook zeer grote bilaterale laesies van de cerebrale cortex of thalamus kunnen coma veroorzaken. In de arousal circuits worden verschillende systemen onderscheiden, met elk andere neurotransmitters.

Acetylcholine (ACh)

ACh is de belangrijkste neurotransmitter van het perifere zenuwstelsel. In het centrale zenuwstelsel is er geen grote plaats voor acetylcholine als neurotransmitter, maar wel als neuromodulator. Vooral in de pontomesencefale regio worden nog cholinerge neuronen aangetroffen. Vanuit dit systeem zijn er ook projecties naar de thalamus en naar de cortex. Het effect van ACh op de thalamus is vaak stimulerend. Ook in de basale voorhersenen en de hippocampus heeft ACh een directe rol. In de hippocampus ontstaat onder invloed van ACh een ritmische oscillatie: het hippocampale thèta ritme. Dit is belangrijk voor geheugenfuncties.

ACh speelt een rol bij aandacht, geheugen en leren. Degeneratie van cholinerge neuronen of farmacologische blokkade veroorzaakt klachten van delier en geheugenstoornissen, maar kunnen ook bijdragen aan het ontstaan van Alzheimer.

Dopamine

Ventraal in de middenhersenen zit de substantia nigra pars compacta. Hier speelt dopamine een grote rol. Ook in het ventrale tegmentale gebied speelt dopamine een rol. Vanuit het mesencephalon zijn er drie projectiesystemen waar dopamine een belangrijke rol in speelt:

  • Het mesostriatale of nigrostriatale projectiesysteem: van de substantia nigra naar de nucleus caudatus en putamen. Laesies in dit gebied worden gezien bij de rol van Parkinson.

  • Het mesolimbische projectiesysteem: van het ventrale tegmentale gebied naar het limbische systeem. Dit systeem is betrokken bij gevoelens van beloning en het bestaan van verslaving. Overactiviteit van dit systeem speelt een rol bij het ontstaan van positieve symptomen (wanen, hallucinaties en onsamenhangende spraak) bij schizofrenie. Dit wordt behandeld met een dopamine antagonist.

  • Het mesocorticale projectiesysteem: van het ventrale tegmentale gebied naar de prefrontale cortex. Dit systeem is van belang voor het werkgeheugen en het starten van een beweging. Laesies hier spelen een rol bij de klachten van de ziekte van Parkinson (hypokinesie, cognitief verlies) en de negatieve symptomen (ontbreken van psychische uitingen die normaal wel aanwezig zijn) bij schizofrenie.

Noradrenaline

Noradrenaline speelt vooral een rol in de locus coeruleus en het laterale tegmentale gebied. Vanuit deze gebieden gaan er projecties naar de voorhersenen (incl. thalamus). Op de thalamus heeft noradrenaline een excitatoir effect. Noradrenaline speelt een rol bij aandacht, het slaap-waak ritme en het humeur. De activiteit van neuronen in de locus coeruleus vertoont een circadiaan ritme: ’s nachts is de activiteit lager dan overdag.

Serotonine

Serotonine speelt vooral een rol in de raphe nuclei in de hersenstam. Laesies in projectiesystemen met serotonerge neuronen (vooral naar de voorhersenen) spelen een rol bij het ontstaan van depressie, angststoornissen, obsessief compulsief gedrag, agressie en eetstoornissen. Ook deze neuronen zijn ’s nachts iets minder actief dan overdag.

Caudaal in de pons en medulla zitten raphe nuclei die projecteren op het cerebellum, de medulla en het ruggenmerg. Deze neuronen zijn betrokken bij pijnmodulatie, ademhaling, temperatuurregulatie en motorische controle.

Histamine

Histminerge neuronen zitten vooral in de posterior hypothalamus. Projecties vanuit hier zijn waarschijnlijk betrokken bij de alertheid.

Andere neurotransmitters

Orexine komt vooral voor in de posterior laterale hypothalamus. Deze neuronen maken deel uit van belangrijke arousal systemen. Adenosine speelt een rol bij de alertheid. Het inhibeert de thalamus en cortex. De activiteit van deze stof is het hoogst vlak voordat men in slaap valt. GABA remt serotonine, noradrenaline, histamine en acetylcholine en stimuleert hierdoor de diepe slaap.

Slaap-waak ritme

Er worden 5 stadia onderscheiden in de slaap: 4 stadia van nonREM slaap en de REM slaap. Tijdens de REM slaap komen dromen voor. Het EEG is in dit stadium als dat van een wakker persoon. In stadia 3 en 4 van de nonREM slaap lijkt het EEG op dat van iemand in coma.

Gebieden in de medulla, anterior hypothalamus en basale voorhersenen zijn belangrijk voor de nonREM slaap. In de reticulaire formatie van de pons liggen verschillende REM-on cellen. dit zijn GABA neuronen. Cholinerge neuronen in de herstam activeren glutamaat REM-on cellen, zodat de spiertonus verlaagt. Wanneer deze neuronen niet goed werken ziet met complex gedrag tijdens de REM slaap.

Bij narcolepsie is er een deficiëntie van orexine. Hierdoor hoeft de nonREM slaap niet te worden doorlopen voor men in de REMslaap komt. Deze patiënten kunnen vanuit een wakker stadium direct in de REM slaap komen, of ze verliezen plotseling de spiertonus (cataplexie).

Coma en soortgelijke aandoeningen

Een coma is te onderscheiden van slaap omdat de patiënt in coma niet te wekken is en niet reageert op prikkels. De ogen zijn gesloten. Er is een verminderde activiteit van de cerebrale cortex en de diencephalon-bovenste hersenstam arousal systemen. Hersenstam reflexen zijn nog wel aanwezig. Het cerebrale metabolisme is verlaagd met tenminste 50%.

Een coma ontstaat vaak door een reversibele oorzaak. Snelle behandeling is dan ook gewenst. Na het waarborgen van de ABC stabiliteit wordt er thiamine, dextrose en naloxon gegeven, omdat thiamine deficiëntie, hypoglycaemie en opiaat overdosis goed behandelbare oorzaken van coma zijn.

Onderzoek van de pupillen kan veel informatie geven over de oorzaak van de coma. Een toxische of metabole oorzaak geeft meestal een normale pupil reactie. Een transtentoriale herniatie of een laesie van de middenhersenen geeft een gedilateerde pupil die niet op licht reageert. Een laesie van de pons geeft een kleine pupil die wel op licht reageert. Bij een opiaat overdosis wordt een pinpoint pupil gezien.

Wanneer een patiënt hersendood is, vertonen de voorhersenen en de hersenstam helemaal geen activiteit meer. Ook de hersenstamreflexen zijn afwezig. Er is geen activiteit op het EEG en het metabolisme is verlaagd tot bijna 0.

Bij een vegetatieve staat is er wel een slaap-waak ritme, er zijn primitieve responsen en reflexen door de herstam en diencephalon. Het metabolisme is met 50% verlaagd, maar de patiënt opent wel de ogen en kan reageren op stimuli.

Wat is caudale reticulaire formatie?

Motorische, reflex en autonome systemen

Het onderste deel van de reticulaire formatie is van belang voor motorische functies, reflexen en autonome functies, waaronder de ademhaling en de hartslag.

In het reguleren van de ademhaling speelt input in de nucleus solitarius een rol. Het pre-Botzinger complex is de pacemaker voor de ademhaling. Beide kernen liggen in de medulla. De nucleus solitarius speelt ook een rol in het reguleren van de hartslag en de bloeddruk, door input van de n. IX en n. X.

Enkele motorische systemen hebben hun oorsprong in de hersenstam. Dit zijn de reticulospinale, vestibulospinale, rectospinale en rubrospinale banen.

Fysiologische verschijnselen als kuchen, hikken, niezen, gapen, rillen, hokhalzen, overgeven, slikken, lachen en huilen, worden bepaald door de caudale reticulaire formatie.

Misselijkheid en braken kunnen worden geïnduceerd door toxische stoffen uit het bloed, die door de bloed-hersenbarrière gaan in de area postrema, of door serotonine afgifte in de maag en dunne darm, waarna er serotonerge vezels naar de nucleus solitarius en de area postrema gaan. Activatie van deze twee gebieden speelt ook een rol bij het ontstaan van misselijkheid en braken door afwijkingen aan het vestibulaire systeem en cerebellum.

Wat is de vascularisatie van de hersenstam?

Aan beide kanten treedt er een a. vertebralis uit de a. subclavia. Deze a. vertebralis loopt door de processi transverus van de wervels omhoog. Op de pontomedullaire overgang komen de twee a. vertebralis samen en vormen de a. basilaris. Deze loopt ventraal langs de pons. In de cirkel van Willis splitst de a. basilaris in twee a. cerebri posterior. Dit is om de pontomesencephale overgang. De a. cerebri posterior vasculariseert de thalamus en een deel van de occipitale en temporale kwabben.

Het vertebrobasillaire syteem kent een viertal grote takken: de a. cerebelli posterior inferior (PICA), a. cerebelli anterior inferior (AICA), a. cerebelli superior (SCA) en e a. cerebri posterior (PCA). De PICA, AICA en SCA vasculariseren het cerebellum. De PICA voorziet de laterale medulla en het inferior cerebellum. Hij ontspringt uit de a. vertebralis. De AICA vasculariseert de laterale caudale pons en een klein deel van het cerebellum. De AICA ontspringt uit de proximale a. basilaris. De SCA ontspringt uit de a basilaris en vasculariseert het superior cerebellum en een klein deel van de rostrale laterodorsale pons. Vlak boven de SCA ontspringt de PCA uit de a. basilaris. Tussen deze twee vaten door loopt de n. oculomotorius. De PCA vasculariseert de middenhersenen, het grootste deel van de thalamus, de mediale occipitale kwabben en e inferior-mediale temporale kwabben.

Het paramediale gebied van de hersenstam wordt gevasculariseerd door paramediane takken.

Aandoeningen van het vertebrobasilaire systeem

Alarmsignalen voor ischemie in het vertebrobasilaire systeem zijn: vertigo, misselijkheid, diplopie, disconjugatie van de oogstand, verminderd zicht, ataxie, instabiele gang, dysarthrie, dysfagie, sensibiliteitsstoornis, tintelingen, hemiparese, quadriparese, slaperigheid en hoofdpijn.

Wanneer het gezicht aan de ene kant is aangedaan en de rest van het lichaam aan de contralaterale kant, is er een laesie van de hersenstam. Vaak zijn er ook afwijkingen in de functies van hersenzenuwen. Afasie, hemineglect, hemianopie en insulten pleiten tegen betrokkenheid van de hersenstam en doen meer denken aan laesies in de hemisferen.

Afhankelijk van de klinische symptomen kan onderscheid worden gemaakt tussen laesies in de middenhersenen, pons en medulla.

Typische afwijkingen voor laesies in de middenhersenen zijn: oculomotorius parese, unilaterale of bilaterale pupil dilatatie, ataxie, flexie van het postuur en verminderd bewustzijn.

Typische afwijkingen voor laesies in de pons zijn: bilateraal teken van Babinski, gegeneraliseerde verminderde spierkracht, perioraal doof gevoel, tintelingen in het gezicht, bilateraal gezichtsveldverlies van bovenste of onderste helft of verminderde scherpte, irregulaire ademhaling, ogen die naar beneden kijken, rillen, myoclonus van het palatum, abducens parese, bilaterale kleine maar reactieve pupillen, extensie van het postuur en verminderd bewustzijn.

Typische afwijkingen van een laesie in de medulla zijn: vertigo, ataxie, nystagmus, misselijkheid, braken, ademstops, autonome verschijnselen en de hik.

In alle gevallen bestaat de behandeling uit tPA, binnen 4,5 uur na het beginnen van de klachten.

Wat voor specifieke syndromen zijn er?

Twee belangrijke syndromen die vaak voorkomen zijn het laterale medullaire syndroom (syndroom van Wallenberg) en het mediale basis pons infarct. Het syndroom van Wallenberg is een infarct van de hersenstam, waarbij het laterale tegmentum is betrokken. De motoriek is hierbij minimaal aangedaan. Meestal is er sprake van een trombose van de a. vertebralis. De symptomen: ipsilaterale ataxie, vertigo, instabiele gang, horizontale of rotatoire nystamus, misselijkheid en braken, verminderde/afwezige vitale sensibiliteit van de ipsilaterale gezichtshelft en de contralaterale lichaamshelft, het syndroom van Horner (ptosis, miosis en soms anhidrosis), paralyse van ipsilaterale stemband, heesheid en dysfagie.

De oorzaak van mediale vasculaire problemen in de pons is meestal een lacunair infarct van de perforerende paramediane pons vaten. Deze vaten kruisen de middenlijn niet, waardoor het infarct ook tot één kan beperkt blijft. Vaak is de oorzaak een chronische hypertensie. Wanneer het infarct ook de lange banen betrekt, ontstaat het lacunaire syndroom met zwakte van de contralaterale gezichtshelft, arm en been en dysarthrie.

Bij grote infarcten kan ook het tegmentum betrokken raken. Hierbij kan de faciale colliculus zijn aangedaan, waardoor er sprak is van zwakte van de ipsilaterale gezichtshelft en een ipsilaterale horizontale blik verlamming, maar een contralaterale hemiparese. Bij de horizontale blikverlamming kijkt het oog weg van de aangedane lichaamshelft.

 

Hoe is de hersenstam opgebouwd?

 

De hersenstam kan functioneel worden verdeeld in 4 structuren: de nuclei van de hersenzenuwen en gerelateerde structuren, de lange banen, het cerebellum en de reticulaire formatie met gerelateerde structuren.

De hersenstam bestaat uit drie verschillende anatomische structuren: de middenhersenen, pons en medulla oblongata (van boven naar beneden). De hersenstam kan ook van voor naar achter worden verdeeld in verschillende structuren: de basis, het tegmentum en het tectum. Het tectum ligt dorsaal van de aquaductus cerebri, het tegmentum ligt voor de aquaductus cerebri en vierde ventrikel. Hierin zitten de meeste nuclei en de reticulaire formatie. De basis bevat vooral veel vezels van de lange banen.

Net als in het ruggenmerg liggen ook in de hersenstam de motorische nuclei ventraal en de sensorische nuclei dorsaal. De scheiding wordt gevormd door de sulcus limitans.

 

Hoe is het cerebellum opgebouwd? - Chapter 15

 

 

Wat zijn cerebellaire kwabben, pedunculi en diepe kernen?

Het cerebellum is aan de dorsale kant van de pons en de rostrale kant van de medulla gehecht met behulp van 3 pedunculi. Het bestaat uit een vermis waaromheen twee hemisferen zitten. De diepste spleet in het cerebellum is de primaire fissure die de anteriore en posteriore kwab van elkaar scheidt.

De fissura posterolateralis, die op het ventrale inferiore gedeelte zit, scheidt de posteriore kwab van de flocculonodulaire kwab. Deze laatste kwab is een gebied met belangrijke verbindingen naar de vestibulaire kernen. Via pedunculi zijn de twee flocculi gehecht aan de nodulus op de middenlijn. De nodulus is het meest inferiore gedeelte van de vermis. De drie cerebellaire pedunculi waar net over gesproken werd, zijn:

  1. Superior cerebellaire pedunculus: output van het cerebellum af. Een andere naam is het brachium conjunctivum.

  2. Middle cerebellaire pedunculus: input naar het cerebellum toe. Wordt soms ook de brachium pontis genoemd door de grote connecties met de pons.

  3. Inferior cerebellaire pedunculus: input naar het cerebellum toe. Een andere naam voor de inferior cerebellaire pedunculus is het restiform body.

Het cerebellum kan verdeeld worden in 3 functionele gebieden die van mediaal naar lateraal lopen:

  1. Vermis en flocculonodulaire kwabben: De vermis is van belang bij de coördinatie van de proximale ledematen en truncus, terwijl de flocculonodulaire kwabben van belang zijn bij de balans en de vestibulo-oculaire reflexen.

  2. Intermediaire deel van de cerebellaire hemisfeer: Van belang voor de coördinatie van de distale ledematen.

  3. Laterale deel van de cerebellaire hemisfeer: van belang bij de planning van het motorprogramma van de extremiteiten. Een groot deel van het laterale deel kan unilateraal verwijderd worden zonder dat er ernstige problemen zullen ontstaan.

Alle output van het cerebellum gaat via de diepgelegen cerebellaire kernen en de vestibulaire kernen. De diepgelegen cerebellaire kernen zijn van lateraal naar mediaal:

  • Dentate nucleus: Dit zijn de grootste en ontvangen informatie van de laterale cerebellaire hemisferen.

  • Emboliform nucleus: tezamen met de globose nucleus vormen zij de interposed nuclei.

  • Globose nucleus: tezamen met de emboliform nucleus vormen zij de interposed nuclei.

De interposed nuclei ontvangen input uit het intermediaire deel van de cerebellaire hemisferen.

  • Fastigial nucleus: Deze ontvangen input uit de vermis en een klein deel uit de flocculonodulaire kwab.

De vestibulaire kernen ontvang informatie uit de inferior vermis en de flocculi.

Hoe werkt het microscopisch circuit van het cerebellum?

De cerebellaire cortex bevat 3 lagen van binnen naar buiten:

  1. Granule cellaag: Deze laag bevat een groot aantal granulen.

  2. Purkinje cellaag: Deze laag bevat de cellichamen van de grote Purkinje cellen.

  3. Moleculaire laag: Deze laag bestaat uit ongemyeliniseerde axonen van granulen, dendrieten van Purkinje cellen en verschillende typen interneuronen.

Er zijn twee soorten van synaptische input naar het cerebellum:

  1. Mossy vezels: deze ascenderen door de cerebellaire witte stof om excitatoire synapsen op de dendrieten van de granulaire cellen te vormen. Granulaire cellen sturen vervolgens axonen naar de moleculaire laag, die bifurceren en parallelle vezels vormen die naar de folia gaan. De folia zijn de bladeren van het cerebellum. De parallelle vezels gaan vervolgens naar de dendrieten van de Purkinje cellen om excitatoire synapsen te vormen. De Purkinje cellen vormen daarna inhibitoire synapsen op de diepe cerebellaire kernen en vestibulaire kernen die vervolgens voor output zorgen van het cerebellum naar andere gebieden via excitatoire synapsen.

Alle output van de cerebellaire cortex worden door de axonen van de Purkinje cellen naar de cerebellaire witte stof gestuurd.

  1. Climbing vezels: Deze ontspringen uit de neuronen in de contralaterale inferior olivary nucleus. Ze omringen het cellichaam van de Purkinje cellen om krachtige excitatoire synapsen te vormen.

De cerebellaire cortex bevat verschillen inhibitoire interneuronen. Basket cellen en stellate cellen zitten in de moleculaire laag. Deze cellen ontvangen synaptische input van de parallelle vezels en zorgen dan voor een laterale inhibitie van de Purkinje cellen. De stellate cellen komen uit op de dendrieten, terwijl de basket cellen een sterke invloed op de cellichamen hebben.

Golgi cellen bevinden zich rondom de granulaire cellaag en ontvangen excitatoire input van parallelle vezels in de moleculaire laag. Ze zorgen voor een inhibitie feedback op de dendrieten van de granulaire cellen, waardoor de duur van exciterende input op de granulaire cellen wordt verkort.

In een gespecialiseerd gebied in de granulaire cellaag vinden complexe synaptische interacties plaats. Dit gebied wordt de cerebellaire glomerulus genoemd.

Wat zijn cerebellaire output pathways?

Cerebellaire pathways zijn georganiseerd rond de 3 functionele gebieden. Bij cerebellaire laesies treden verstoringen in de coördinatie op in het ipsilaterale deel ten opzichte van de laesie. Dit komt doordat er twee keer gekruist wordt. De eerste keer vindt plaats bij de superior cerebellaire pedunculus en de tweede keer bij de overgang naar het ruggenmerg.

De laterale cerebellaire hemisfeer stuurt informatie naar de nucleus dentatus die vervolgens via de superior cerebellaire pedunculus informatie stuurt naar de contralaterale ventral lateral nucleus van de thalamus (VL). De vezels die deze kern ingaan worden de fasciculus thalamicus genoemd. De anteriore delen van de fasciculus bevat output van de basale ganglia die informatie naar de anterior ventrale laterale nucleus brengt. De output van het cerebellum gaat naar het posteriore gedeelte. Het VL zorgt voor het overbrengen van informatie naar de motorische cortex en de bijbehorende andere cortexen. Hierdoor wordt het corticospinale systeem beïnvloed.

Het kan ook zo zijn dat de output vezels via de nucleus dentatus naar de rode kern in de middenhersenen gaan.

De intermediaire hemisfeer stuurt zijn informatie naar de emboliforme en globose kernen. Deze interposed kernen gaan via de superior cerebellaire pedunculus naar de contralaterale thalamus VL om naar de primaire motor cortex te gaan en het laterale corticospinale traject te beïnvloeden.

De cerebellaire vermis stuurt informatie naar de fastigiale nucleus. Vanuit daar wordt het via de superior cerebellaire pedunculus naar een gekruiste pathway gebracht. Deze bereikt de VL van de thalamus en is van belang voor de cortex die het anterior corticospinale traject beïnvloedt.

De flocculonodulaire kwabben en inferior vermis worden soms gerefereerd als het vestibulocerebellum. Verbindingen tussen het cerebellum en de vestibulaire kernen zijn van belang voor het evenwicht en balans.

Wat zijn cerebellaire input pathways?

De input die het cerebellum bereiken komen van:

  1. Alle gebieden van de cerebrale cortex

  2. Verschillende sensorische modaliteiten

  3. Hersenstam kernen

  4. Ruggenmerg

Een grote bron voor de input bestaat uit de corticopontine vezels uit de frontale, temporale, pariëtale en occipitale kwabben die naar de capsula interna en cerebrale pedunculi lopen. De primaire sensorische en motorische cortexen en een deel van de visuele cortex maken verzorgen het grootste deel van de corticopontine vezels. De corticopontine vezels gaan naar de ipsilaterale pons en synapsen in de pontine nuclei. Pontocerebellaire vezels kruisen de middenlijn en zorgen voor de mossy vezels die bijna de gehele cerebellaire cortex bereiken.

Een andere bron voor de input zijn de spinocerebellaire vezels. Deze reizen in vier trajecten:

  1. Dorsale en ventrale spinocerebellaire traject: voor onderste extremiteiten

  2. Cuneocerebellaire en rostral spinocerebellaire traject: voor bovenste extremiteiten

De spinocerebellaire pathways zorgen voor twee vormen van feedback informatie naar het cerebellum:

  1. Afferente informatie over de ledemaatbewegingen: verkregen door dorsale spinocerebellaire traject en cuneocerebellaire traject

  2. Informatie over de activiteit van ruggenmerg Interneuronen: verkregen door ventrale spinocerebellaire traject en rostraal spinocerebellaire traject.

Hoe werkt de vasculaire toevoer naar het cerebellum?

Er zijn 3 takken van de vertebrale en basilaire arteriën die het cerebellum voorzien van bloed:

  1. Posterior inferior cerebellaire arterie: ontspringt uit de a. vertebralis. Het voorziet de laterale medulla, grootste deel van de inferiore helft van het cerebellum en de inferior vermis.

  2. Anterior inferior cerebellaire arterie: ontspringt uit de lower basilaris. Voorziet de inferior laterale pons, de middle cerebellaire pedunculus en de flocculus.

  3. Superior cerebellaire arterie; ontspringt uit de top van de basilaris. Voorziet de upper laterale pons, superior cerebellaire pedunculus en grootste deel van het superiore deel van de cerebellaire hemisfeer.

KCC 15.2: Klinische bevindingen en lokalisatie van cerebellaire laesies

Ataxie refereert naar de verwarde contracties van agonist en antagonist spieren en het gebrek van normale coördinatie. Ataxische bewegingen hebben abnormale timing (dysrythmia) en abnormale trajecten door de lucht(dysmetria)

Truncal Ataxia versus appendicular ataxia

Laesies aan de cerebellaire vermis zorgen voor een breedbasisch looppatroon. Wanneer er geen andere significante abnormaliteiten worden ontdekt wordt het truncal ataxie (rompataxie) genoemd. Laesies aan de intermediaire en laterale delen zorgen voor veranderingen in de laterale motorsystemen. Ze hebben ataxie bij bewegen van de extremiteiten. Dit wordt ook wel appendiculaire ataxie genoemd.

Foute localisatie van ataxie

Ataxie wordt vaak veroorzaakt door laesies buiten het cerebellum. Het gaat dan om de input en output pathways. Ataxie-hemiparesis is een syndroom dat vaak door lacunaire infarcering wordt veroorzaakt. Men heeft dan door uitval van unilaterale upper motor neuronen symptomen en ataxie. Het wordt vaak veroorzaakt door laesies aan de capsula interna of pons. Sensorische ataxie zorgt voor verlies van propriocepsis.

Wat voor symptomen en signalen van cerebellaire stoornissen zijn er?

Symptomen van cerebellaire stoornissen

Patienten met cerebellaire laesies hebben vaak last van overgeven, ongecoördineerde bewegingen, vermoeidheid en verkoudheid.

Other abnormalities can confound the cerebellar exam

Bij het examineren van een patiënt moet men eerst kijken of er veranderingen bij upper of lower motor neuronen zijn, sensorisch verlies of basale ganglia dysfunctie. Verlies van propriocepsis kan voor sensorische ataxie zorgen. Bewegingsverstoringen als bij morbus Parkinson worden geassocieerd met basale ganglia dysfunctie waardoor men langzamere bewegingen gaat maken.

Testing for appendicular ataxia

De meeste abnormaliteiten kunnen beschreven worden aan de hand van een combinatie van dysmetrie en dyrshythmia. Dysmetrie is het abnormale doorschieten van een beweging richting een doelwit. Dysrhythmia is het abnormale ritme en timing van bewegingen. Er zijn verschillende testen te onderscheiden:

  1. Finger-nose-finger: bij deze test moet de patiënt zijn vinger naar zijn neus brengen en vervolgens naar de vinger van de behandelende arts.

  2. Heel-shin: Bij deze test moet de ene hiel langs de andere hiel worden gehaald. Soms worden er ook testen gebruikt waarbij de hand een ritme moet aangeven op de heup. Hierdoor kan je ontdekken of er sprake is van dysrhythmia.

Om te kunnen ontdekken of de armen doorschieten of juist helemaal niet doorschieten kan de arts ervoor kiezen om de patiënt zijn armen op een bepaald moment om hoog te laten brengen.

Testen voor truncale ataxie (rompataxie)

Deze mensen hebben een breedbasisch looppatroon. Het ziet eruit als een looppatroon van iemand die dronken is. Je kunt dit onderzoeken door de patiënt steeds de ene voet voor de andere voet te plaatsen door de hiel tegen de tenen te zetten. Hiernaast kan ook de test van Romberg worden uitgevoerd.

Eyemovement abnormalities

Mensen met cerebellaire laesies kunnen ook verschijnselen van oculaire dysmetria krijgen. Vaak is er nystagmus aanwezig.

Spraakabnormaliteiten

Er kunnen irregulaire fluctuaties aanwezig zijn in de snelheid en het volume. Het wordt soms “explosieve spraak”genoemd.

 

Het cerebellum is aan de dorsale kant van de pons en de rostrale kant van de medulla gehecht met behulp van 3 pedunculi. Het bestaat uit een vermis waaromheen twee hemisferen zitten. De diepste spleet in het cerebellum is de primaire fissure die de anteriore en posteriore kwab van elkaar scheidt.

 

De fissura posterolateralis, die op het ventrale inferiore gedeelte zit, scheidt de posteriore kwab van de flocculonodulaire kwab. Deze laatste kwab is een gebied met belangrijke verbindingen naar de vestibulaire kernen. Via pedunculi zijn de twee flocculi gehecht aan de nodulus op de middenlijn. De nodulus is het meest inferiore gedeelte van de vermis. De drie cerebellaire pedunculi waar net over gesproken werd, zijn:

  1. Superior cerebellaire pedunculus: output van het cerebellum af. Een andere naam is het brachium conjunctivum.

  2. Middle cerebellaire pedunculus: input naar het cerebellum toe. Wordt soms ook de brachium pontis genoemd door de grote connecties met de pons.

  3. Inferior cerebellaire pedunculus: input naar het cerebellum toe. Een andere naam voor de inferior cerebellaire pedunculus is het restiform body.

Het cerebellum kan verdeeld worden in 3 functionele gebieden die van mediaal naar lateraal lopen:

  1. Vermis en flocculonodulaire kwabben: De vermis is van belang bij de coördinatie van de proximale ledematen en truncus, terwijl de flocculonodulaire kwabben van belang zijn bij de balans en de vestibulo-oculaire reflexen.

  2. Intermediaire deel van de cerebellaire hemisfeer: Van belang voor de coördinatie van de distale ledematen.

  3. Laterale deel van de cerebellaire hemisfeer: van belang bij de planning van het motorprogramma van de extremiteiten. Een groot deel van het laterale deel kan unilateraal verwijderd worden zonder dat er ernstige problemen zullen ontstaan.

Alle output van het cerebellum gaat via de diepgelegen cerebellaire kernen en de vestibulaire kernen. De diepgelegen cerebellaire kernen zijn van lateraal naar mediaal:

  • Dentate nucleus: Dit zijn de grootste en ontvangen informatie van de laterale cerebellaire hemisferen.

  • Emboliform nucleus: tezamen met de globose nucleus vormen zij de interposed nuclei.

  • Globose nucleus: tezamen met de emboliform nucleus vormen zij de interposed nuclei.

De interposed nuclei ontvangen input uit het intermediaire deel van de cerebellaire hemisferen.

  • Fastigial nucleus: Deze ontvangen input uit de vermis en een klein deel uit de flocculonodulaire kwab.

De vestibulaire kernen ontvang informatie uit de inferior vermis en de flocculi.

 

Wat is de functie van basale ganglia? - Chapter 16

 

 

Hoe zit de driedimensionale anatomie van de basale ganglia in elkaar?

De basale ganglia nemen net als het cerebellum ook deel aan de complexe netwerken die de descenderende motorsystemen beïnvloeden. Patienten met laesies van de basale ganglia kunnen hyperkinetische of hypokinetische bewegingsverstoringen hebben.

De basale ganglia zijn een collectie van grijze stof kernen die diep in de witte stof in de cerebrale hemisferen liggen. De belangrijkste componenten zijn de nucleus caudatus, putamen, globus pallidus, nucleus subthalamicus en substantia nigra. De caudatus en het putamen worden vaak gezien als een grote kern, het (neo)striatum. Het striatum ontvangt alle input voor de basale ganglia. De caudatus is opgedeeld in het hoofd (caput), het lichaam (corpus) en de staart (cauda). Het putamen is een grote kern die het laterale deel van de basale ganglia vormt. Anterior en ventraal fuseert het putamen met het hoofd van de nucleus caudatus. Deze regio wordt het ventrale striatum genoemd en is van belang voor het limbische circuit. Mediaal ten opzichte van het putamen ligt de globus pallidus. Deze bevat een ‘internal segment’ en een ‘external segment’. Het putamen en de globus pallidus vormen tezamen de nucleus lentiformis, oftewel de lensvormige kern.

De substantia nigra bevat een ventraal deel dat de substantia nigra pars reticulata wordt genoemd. Het bevat cellen die veel op de cellen lijken uit het interne segment van de globus pallidus. De meer dorsaalgelegen substantia nigra pars compacta bevat de dopaminerge neuronen. Degeneratie van deze neuronen komt vaak voor bij de ziekte van Parkinson.

Hoe werkt de input, output en intrinsieke connections van de basale ganglia?

Alle input voor het basale ganglia komt via het striatum. Output gaat via het interne segment van de globus pallidus en de substantia nigra pars reticulata. Er zijn verschillende parallelle pathways in de basale ganglia voor verschillende functies:

  • General motor control

  • Oogbewegingen

  • Cognitieve functies

  • Emotionele functies

Inputs to the basale ganglia

Het putamen is de belangrijkste inputkern van het striatum voor de motor control pathways. De meeste corticale inputs zijn excitatoir en gebruiken glutamaat als neurotransmitter. De substantia nigra pars compacta is een dopaminerge nigrostriatale pathway en is excitatoir voor sommige cellen en inhibitoir voor andere in het striatum. Het striatum ontvangt ook input van intralaminaire kernen.

Output uit de basale ganglia

De output komt uit het interne segment van de globus pallidus en de substantia nigra pars reticularis. De substantia bevat informatie voor het hoofd en de nek terwijl de globus de rest van het lichaam verzorgt. Het zijn inhibitoire pathways en bevatten GABA als neurotransmitter. De belangrijkste pathways gaan naar de ventral laterale en ventral anterior kernen in de thalamus via de fasciculus thalamicus. De thalamic neuronen zenden informatie van de basale ganglia naar de gehele frontale kwab. Output kan ook via andere thalamic kernen. Dit zijn de intralaminaire kernen die weer voor een feedback zorgen naar het striatum, en de mediodorsale kern, die primair betrokken is bij de limbische circuits.

Intrinsieke basale ganglia connecties

Er zijn twee predominante pathways van input naar output nuclei door de basale ganglia. Dit zijn de directe pathway die van het striatum direct naar het interne segment van de globus pallidus of naar de substantia nigra pars reticulata loopt en de indirecte pathway. Deze gaat van het striatum eerst naar het externe segment en dan naar de subthalamische nucleus voordat het bij het interne segment terecht komt. In de afbeelding hiernaast kan je zien hoe de verschillende pathways lopen en welke neurotransmitters er aanwezig zijn.

Parallele basale ganglia pathways voor algemene beweging, oogbeweging, cognitieve en emotie

Er bestaan 4 verschillende kanalen die door de basale ganglia gaan. De eerste 3 worden de dorsale striatal pathways genoemd en het limbische kanaal wordt de ventral striatal pathway genoemd.

Bron voor corticale input

Basale ganglia input kernen

Basale ganglia output kernen

Corticale targets of output

Motorische kanaal: somatosensorisch, primaire motorisch en premotorische cortex

Putamen

Globus pallidus interne segment en substantia nigra pars reticulata

Supplementaire motor gebieden, premotorische cortex, primaire motorische cortex

Oculomotorische kanaal: posterior pariëtale cortex en premotorische cortex

Lichaam caudate

Globus pallidus interne segment, substantia nigra pars reticulata

Frontale oogvelden, supplementaire oogvelden

Prefontrale kanaal: posterior pariëtale cortex, premotorische cortex

Hoofd van caudate

Globus pallidus interne segment en substantia nigra pars reticulata

Prefrontale cortex

Limbische kanaal: temporal cortex, hippocampus, amygdala

Nucleus accumbens, ventral caudate, ventral putamen

Ventral pallidum, globus pallidus interne segment en substantia nigra pars reticulata

Anterior cingulated, orbital frontal cortex

Ansa lenticularis, lenticular fasciculus en forelvelden

Het interne segment van de globus pallidus stuurt outputberichten naar de thalamus via twee verschillende pathways. De eerste is de ansa lenticularis en de tweede is de lenticular fasciculus. Deze vezels gaan recht door het capsula interna terwijl die van de ansa juist met een boog eromheen gaan. De lenticulaire fasciculus en de ansa lenticularis vormen samen de thalmic fasiculus die de thalamus ingaat. Het H1 veld van Forel is de thalamic fasciculus en de H2 veld is de lenticular fasciculus.

 

Hoe zit de driedimensionale anatomie van de basale ganglia in elkaar?

 

De basale ganglia nemen net als het cerebellum ook deel aan de complexe netwerken die de descenderende motorsystemen beïnvloeden. Patienten met laesies van de basale ganglia kunnen hyperkinetische of hypokinetische bewegingsverstoringen hebben.

De basale ganglia zijn een collectie van grijze stof kernen die diep in de witte stof in de cerebrale hemisferen liggen. De belangrijkste componenten zijn de nucleus caudatus, putamen, globus pallidus, nucleus subthalamicus en substantia nigra. De caudatus en het putamen worden vaak gezien als een grote kern, het (neo)striatum. Het striatum ontvangt alle input voor de basale ganglia. De caudatus is opgedeeld in het hoofd (caput), het lichaam (corpus) en de staart (cauda). Het putamen is een grote kern die het laterale deel van de basale ganglia vormt. Anterior en ventraal fuseert het putamen met het hoofd van de nucleus caudatus. Deze regio wordt het ventrale striatum genoemd en is van belang voor het limbische circuit. Mediaal ten opzichte van het putamen ligt de globus pallidus. Deze bevat een ‘internal segment’ en een ‘external segment’. Het putamen en de globus pallidus vormen tezamen de nucleus lentiformis, oftewel de lensvormige kern.

De substantia nigra bevat een ventraal deel dat de substantia nigra pars reticulata wordt genoemd. Het bevat cellen die veel op de cellen lijken uit het interne segment van de globus pallidus. De meer dorsaalgelegen substantia nigra pars compacta bevat de dopaminerge neuronen. Degeneratie van deze neuronen komt vaak voor bij de ziekte van Parkinson.

 

Hoe is het limbisch systeem opgebouwd? - Chapter 18

 

 

Hoe is het limbische systeem opgebouwd?

Het limbische systeem heeft verschillende corticale en subcorticale structuren die zich bevinden in de mediale en ventrale regio’s van de cerebrale hemisferen. Deze structuren vormen het grootste gedeelte van het voorbrein in veel verschillende diersoorten. De functies van het limbische systeem spelen een belangrijke rol bij het overleven in het dierenrijk. De functies kunnen worden verdeeld in 4 categorieën:

  1. Olfaction (reukzin)
  2. Memory (geheugen)
  3. Emotions and drives (emoties en driften)
  4. Homeostatic functions including autonomic and neuroendocrine control (homeostatische functie).

Deze functies kunnen onthouden worden door het ezelsbruggetje HOME. Veel verschillende structuren van het limbische systeem hebben een bijdragen in deze processen. Het olfactoire complex is essentieel voor reukzin, de hippocampal formation voor geheugen, de amygdala voor emoties en driften en de hypothalamus voor homeostase. Niet alleen deze gebieden zijn verantwoordelijk voor de functies, er is samenwerking tussen verschillende gebieden.

De limbische cortex vormt een ring achtige limbische lobe om de rand van de corticale mantel heen, die het corpus callosum en bovenste-hersenstam-diencephalische junction omringt. Deze ‘grand lobe limbique’ werd voor het eerst omschreven door Broca. De belangrijkste delen van de limbische cortex zijn de cingulate gyrus en de parahippocampal gyrus. De parahippocampal gyrus wordt gescheiden van het overblijfsel van de temporale kwab door de collaterale sulcus, die anterieur doorgaat als de rhinal sulcus. De uncus is een bobbel die zichtbaar is anterieur mediaal op de parahippocampal gyrus. De cingulate gyrus komt samen met de parahippocampal gyrus posterieur van de isthmus. Andere regio’s in het limbische systeem zijn de mediale orbitofrontale gyri, de temporale polen en de anterieure insulaire cortex.

De hippocampal formation is de mediale en dorsale voortgang van de parahippocampale gyrus. Het heeft de vorm van een C en het speelt een belangrijke rol in het geheugen. De hippocampus heeft 3 lagen en heet de archicortex. Neocortex (ook wel isocotex) is een 6-lagige cortex waaruit de cerebrale cortex voor het meeste bestaat. De mesocortex (limbic cortex) is een overgang tussen 6-lagige en 3-lagige cortex, die gevonden wordt in de parahippocampale gyrus en de orbitofrontale cortex. De allocortex is een cortex met minder dan 6 lagen, die onder verdeeld kan worden in archicortex en paleocotex. Archicortex is een 3-lagige hippocampale cortex, die dus vooral gevonden wordt in de hippocampal formation. De paleocortex is een 3-lagige olfactoire cortex, die gevonden wordt in de piriforme cortex. Corticoid gebieden zijn simpele cortexen die samengaan met subcorticale nuclei, en deze worden gevonden in de amygdala.

De amygdala overlapt het anterieure deel van de hippocampus en ligt dorsaal van de temporale hoorn. De posterieure amygdala liggen onder de uncus. De amygdala bestaat uit 3 belangrijke nuclei: corticomediale nuclei, basolaterale nuclei en centrale nuclei. Ook de C-vormige bed nucleus van de stria terminalis wordt als deel van de amygdala gezien.

Diencephale structuren werken mee met alle functies van het limbische systeem. Deze structuren zijn de hypothalamus, de mediodorsale nucleus van de thalamus, de antereieure nucleus van de thalamus en de habenula. Limbische inputs voor de basala ganglia komen aan in de ventrale striatum en de nucleus accumbens en worden voortgebracht via de ventrale pallidum naar de mediodorsale nucleus van de thalamus.

Het basale voorbrein heeft ook corticoïde structuren, De term substantia innominata wordt gebruikt voor het basale voorbrein of voor een nucleus die erin ligt, die de nucleus basalis of Meynert heet. Deze nucleus basalis heeft cholinerge neuronen die de meeste cholinerge innervatie verzorgen voor de cerebrale cortex. Andere nuclei die in het basale voorbrein liggen zijn olfactory tubercle, ventrale pallidum, nucleus of the diagnoal band of Broca en de preoptic area.

De septale regio ligt dorsaal van het basale voorbrein en helpt ook mee met limbische pathways. De belangrijkste nuclei hiervan zijn de mediale septale nucleus en de laterale septale nucleus. De mediale septale nucleus heeft cholinerge neuronen die een belangrijke rol spelen bij de functie van het geheugen. Inputs van de hippocampale formatie komen voornamelijk naar de laterale septale nucleus. De nucleus accumbens is betrokken bij het basale ganglia-limbisch systeem circuit.

Sommige hersenstam kernen hebben verbindingen met limbische pathways waardoor ze soms tot het limbische systeem gerekend worden. Voorbeelden hiervan zijn: interpeduncular nucleus, superior central nucleus, dorsal tegmental nucleus, ventral tegmental nucleus, parabrachial nucleus, periaqueductual gray, reticular formation, nucleus solitarius en dorsale motor nucleus van de nervus vagus.

Wat is het olfactoire systeem?

Rhinencephalon betekent letterlijk ‘neus brein’ en wordt gebruikt voor hersenstructuren die alleen betrokken zijn bij de reukzin. Reuk wordt tot stand gebracht door zowel geur van het externe milieu maar ook door smaak (retronasal smell). Bipolaire olfactoire receptor neuronen in de olfactoire mucosa zorgen voor genexpressie van olfactoire receptor genen. Een geur kan soms meerdere olfactoire receptors activeren, daardoor kunnen er oneindig veel verschillende geuren tot stand komen.

Olfactoire receptor neuronen sturen ongemyeliniseerde axonen in de olfactoire zenuwen door de cribiforme plate om uiteindelijk de olfactoire bulb te bereiken. De olfactoire bulb is een deel van het centrale zenuw stelsel en het ligt in een groeve die de olfactoire sulcus heet, tussen de gyrus rectus en de orbitofrontale gyri. In de glomeruli van de olfactoire bulb synapteren de olfactoire receptor neuronen op mitral cellen en tufted cellen. Beiden hebben lange axonen die de olfactoire tractus ingaan en uiteindelijk de olfactoire cortex bereiken. Net zoals de mitrale cellen en tufted cellen, heeft de olfactoire bulb interneuronen die periglomerulaire cellen en granule cellen heten.

De primaire olfactoire cortex is uniek in de sensorische systemen, omdat het direct input ontvangt van de secondary sensory neurons zonder dat er nog modulatie plaatsvindt door de thalamus. De primaire olfactoire cortex bestaat uit een piriforme cortex en de periamygdaloid cortex.

Mitrale cellen en tufted cellen projecteren direct via de laterale olfactoire stria van de olfactoire tractus op de primaire olfactoire cortex. Ook projecteren ze op de corticomediale nucleus van de amygdala en sommige vezels op de olfactoire tubercule (die ligt in de anterieure perforated substance). Deze projecties zijn belangrijk voor emotionele en motivationele aspecten van reukzin.

De anterieure entorhinale cortex ontvangt projecties van de piriforme cortex. Deze entorhinale cortex heeft ook een rol in het geheugen, wat uit kan leggen waarom bepaalde geuren bepaalde herinneringen oproepen. De piriforme cortex projecteert op de orbitofrontale olfactoire area, in de mediodorsale nucleus van de thalamus, op de basolaterale amygdala, de laterale preoptische area en de nucleus van de diagonale band.
Er is geen directe projectie van de piriforme cortex naar de hippocampal formation en de hippocampal formation heeft ook geen significante rol bij het verwerken van geur.

Atrofie van de anterieure olfactoire nucleus zorgt bij Alzheimer patiënten voor een verminderd reukvermogen.

Wat is hippocampale vorming?

Er zijn 2 regio’s in de hersenen die belangrijk zijn in het vormen van herinneringen: de mediale temporale lobe memory areas, waarin de hippocampal formation en de adjacent cortex van de parahippocampal gyrus zich bevinden en de mediale diencephalische memory areas, waaronder de thalamische medio-dorsale nucleus, anterieure nucleus van de thalamus, mammillary bodies en andere diencephalische nuclei uit het 3e ventrikel. Deze twee gebieden zijn verbonden met elkaar door verschillende pathways. Deze witte materie netwerk connecties zijn dus nodig voor een goede functie van het geheugen. Het basale voorbrein speelt ook een rol in het geheugen, vooral door de cholinerge projecties op de cerebrale cortex.

De belangrijkste structuren van het mediale temporale kwab geheugen systeem zijn de hippocampal formation en de parahippocampal gyrus. De drie bestanddelen van de hippocampal formation zijn de dentate gyrus, de hippocampus en het subiculum. De belangrijke neuronen van de dentate gyrus zijn de granule cellen. Er zijn 3 lagen in de dentate gyrus, namelijk de moleculaire laag, de granulaire cel laag en daarna de polymorfische laag.

De belangrijkste neuronen van de hippocampus en het subiculum zijn de pyramidale cellen. Er zijn ook 3 lagen hierin, namelijk de moleculaire laag, de pyramidale cel laag en de polymorfische laag. De moleculaire lagen van de dentate gyrus en de subiculum vormen de hippocampal sulcus. De groeve in de mediale temporale kwab dorsaal van de hippocampal formation heet de choroid fissure. Anterieur vormt de hippocampal formation het pes hippocampi, ook wel het hoofd van de hippocampus. De hippocampal formation gaat naar achter en vormt daar de hippocampale staart en verdwijnt daarna wanneer het onder de posterieure hoek van het splenium van het corpus callosum gaat.

De parahippocampal gyrus heeft bepaalde corticale gebieden die verbindingen hebben met de hippocampal formation, waarvan de belangrijkste de entorhinal cortex (Brodmann’s area) is. De functie van de entorhinale cortex is dat het een relay vormt tussen input en output tussen de cortex en de hippocampal formation. Het posterieure deel van van de parahippocampale gyrus heet de parahippocampale cotex. Aan de laterale kant wordt de parahippocampale gyrus begrensd door de collaterale sulcus, die anterieur doorgaat als de rhinale sulcus.

De perforant pathway gaat door het subiculum, door de hippocampale sulcus en daarna naar de granulaire cel laag van de dentate gyrus. De hippocampus heeft verschillende pyramidale cellen die CA genoemd worden, van CA1 tot en met CA4. Granulaire cellen van de dentate gyrus vormen axonen die mossy vezels heten, die synapteren op de dendrieten van de CA3 pyramidale cellen. De axonen van CA3 verlaten de hippocampal formation via de fornix. Deze axonen vormen de Schaffer collateralen, die synapteren op de dendrieten van CA1. CA1 cellen projecteren op het volgende cellulaire relay, die in het subiculum ligt. De pyramidale cellen van het subiculum projecteren op de fornix en weer op de neuronen van de entorhinale cortex, waarna het proces compleet is.

Ook is er nog de alvear pathway, waarbij de neuronen van entorhinale cortex direct projecteren op CA1 en CA3. Dit is het directe pathway, de perforant pathway is het tri-synaptische pathway. LTP is long term potentiation een proces dat gevonden wordt in de perforant pathway-granule cel, mossy vezels van CA3 en Schaffer collateralen CA1 verbindingen. Hoge frequentie activiteit zorgen voor een langdurige stijging van de synaptische kracht tussen de neuronen. Door dit principe kunnen de synapsen een associatieve functie hebben.

De belangrijkste input van de hippocampale formatie komt binnen bij de entorhinale cortex. Veel van deze informatie gaat eerst nog langs de adjacent perirhinal cortex en de parahippocampale cortex voordat het bij de entorhinale cortex terecht komt. Een belangrijke output van de hippocampale formatie is de projectie van de subiculum naar de entorhinale cortex en vanaf daar weer terug naar de multimodale association cortex. Een andere grote output pathway is de fornix die outputs overdraagt naar het diencephalon en de septale nuclei. Het subiculum is de belangrijkste bron van output vezels van de hippocampale formatie naar het fornix net als naar de entorhinale cortex. Ook stuurt het subiculum monosynaptische verbindingen naar de amygdala, de orbitofrontale cortex en het ventrale striatum. Het subiculum is daarom een belangrijke structuur voor hippocampale outputs.

De hippocampal formation krijgt modulatoire inputs via de fornix door cholinerge neuronen in de mediale septale nucleus en de nucleus van de diagonale band.

Fornix is een naam voor de witte stof die door het ventriculaire systeem van de hippocampal formatie heen kronkelt. Output vezels van de hippocampal formatie vormen een witte stof laag in het ventriculaire oppervlak van de hippocampus genaamd de alveus. Axonen die uit de fornix komen hebben 3 doelen: de meeste vezels komen op uit het subiculum en gaan naar de mediale en laterale mammillary nuclei van de hypothalamus. Een aantal vezels die opkomen uit het subiculum en de hippocampus gaan naar de laterale septale nucleus. De laatste groep vezels gaan naar de anterieure thalamische nucleus.

Wat is de amygdala?

De amygdala of amygdaloid nucleair complex is een groep van nuclei die 3 grote nuclei heeft: corticomediale nuclei, basolaterale nuclei en centrale nuclei. De corticomediale nuclei zijn verbonden met de olfactoire gebieden en zijn betrokken in reukzin en interactie met de hypothalamus. De centrale nucleus heeft ook verbindingen met de hypothalamus en hersenstem en zijn belangrijk bij autonome controle.

De amygdala is belangrijk in de functie van het geheugen en blijkt ook een belangrijke rol te spelen bij de reukzin, en dan vooral de emotionele en motivationele kant daarvan.

Subcorticale connecties van de amygdala zijn belangrijk in de motivationele, autonome en neuroendocriene aspecten van de limbische functie. Deze verbindingen zijn meestal efferent van de amygdala maar hebben ook afferente componenten. Dit gebeurt door 2 belangrijke pathways, de stria terminalis en de ventral amygdalofugal pathway.

  • Stria terminalis: “the long way around”: C-vormige structuur die van de amygdala loopt naar de wand van het laterale ventrikel en uiteindelijk de hypothalamus bereikt. De stria terminalis kan dus gezien worden als de fornix van de amygdala. Een verbinding die door deze stria terminalis wordt geleid is olfactoire informatie, die bijvoorbeeld kan zorgen voor meer of minder honger.

  • Ventral amygdalofugal pathway: “the shortcut”: deze pathway gaat anterieur van de amygdala en ontvangt verschillende structuren van het voorbrein en de hersenstam. Projecties van het voorbrein die betrokken zijn bij bijvoorbeeld emotie, motivatie en cognitieve functies.

Een ander pathway is de stria medullaris. Dit zijn vezels die langs de wand van het derde ventrikel op de mediale oppervlakte van de thalamus lopen. Het neemt vezels mee van de mediale septale nuclei naar de habenula, een kleine structuur lateraal van de pineale klier. De habenula projecteert via de hebenulointerpedunculaire tractus op de interpedunculaire nuclei in het middenbrein. De interpedunculaire nuclei projecteren op serotonergische raphe nuclei en ook op dopaminerge nuclei, die op hun beurt weer een diffuse projectie hebben over het gehele brein.

 

Hoe is het limbische systeem opgebouwd?

 

Het limbische systeem heeft verschillende corticale en subcorticale structuren die zich bevinden in de mediale en ventrale regio’s van de cerebrale hemisferen. Deze structuren vormen het grootste gedeelte van het voorbrein in veel verschillende diersoorten. De functies van het limbische systeem spelen een belangrijke rol bij het overleven in het dierenrijk. De functies kunnen worden verdeeld in 4 categorieën:

  1. Olfaction (reukzin)
  2. Memory (geheugen)
  3. Emotions and drives (emoties en driften)
  4. Homeostatic functions including autonomic and neuroendocrine control (homeostatische functie).

Deze functies kunnen onthouden worden door het ezelsbruggetje HOME. Veel verschillende structuren van het limbische systeem hebben een bijdragen in deze processen. Het olfactoire complex is essentieel voor reukzin, de hippocampal formation voor geheugen, de amygdala voor emoties en driften en de hypothalamus voor homeostase. Niet alleen deze gebieden zijn verantwoordelijk voor de functies, er is samenwerking tussen verschillende gebieden.

 

Wat zijn higher-order cerebrale functies? - Chapter 19

 

 

Wat is zijn het unimodaal en het heteromodaal associatie cortex?

Unimodale associatie cortex houdt zich primair bezig met maar één soort informatie, dus motorisch of sensorisch voor reuk, sensorisch voor zicht enzovoort. De unimodale sensorische associatie cortex krijgt input van de bijbehorende primaire sensorische cortex. De unimodale motorische cortex stuurt output naar de primaire motorische cortex en is belangrijk voor planning van moeilijke bewegingen.

Heteromodale associatie cortex integreert verschillende soorten informatie, zowel motorisch als sensorisch én met het limbische systeem. Dit zorgt voor de hoogste cerebrale functies. Deze cortex ligt voornamelijk in de frontale kwabben en de parieto-occipitotemporal junctions.

Wat is cerebrale localisatie en lateralisatie?

De hersenen bestaan uit gespecialiseerde regio’s die onderling verbonden zijn door netwerken. Afwijkingen kunnen gesteld worden aan de hand van de anatomie van de hersenen. Deze diagnosen kunnen er echter ook naast zitten, doordat meerdere regio’s samenwerken. Disconnection syndromes ontstaan als twee verbonden regio’s afgesneden worden en dus niet meer samen kunnen werken.

De hemisferen vertonen ook specialisatie, dat wil zeggen dat één hemisfeer een bepaalde taak op zich neemt. Dit is voornamelijk zo bij taal. Dit zit bij het overgrote deel van de in de dominante hemisfeer (in 90% van de gevallen is dit de linkerhemisfeer). De niet-dominantie hemisfeer is gespecialiseerd in aandacht aan beide kanten van de omgeving en visuele-ruimtelijke oriëntatie.

Als er een laesie van de dominante hemisfeer op jonge leeftijd ontstaat, verhuizen de functies naar de niet-dominante hemisfeer. Hierdoor is er nauwelijks verlies van functie.

De anterior frontal association cortex heeft te maken met de planning en uitvoering van gedrag. De posterior association cortex heeft te maken met het interpreteren en betekenis toekennen aan sensorische inforatie.

Wat is de dominante hemisfeer?

In de dominante hemisfeer zitten taalverwerking en functies die daarbij horen. De sensorische informatie die van het oor binnenkomt gaat eerst naar de primaire auditieve cortex. Vandaar gaat het naar Wernicke’s area, waar het taalbegrip zit. Hier wordt begrepen dat de geluiden woorden zijn. In Broca’s area worden woorden gevormd en uitgesproken.

Er zijn meerdere verbindingen tussen Wernicke’s en Broca’s area, de meest bekende arcuate fasciculus. Verder staat Broca’s area in verbinding met de frontale kwabben, voor goede uitspraak en grammatica. Wernicke’s area staat in verbinding met de supramarginal en angular gyri. Dit is voor het taalbegrip en het omzetten van geluid in woorden. Tijdens lezen wordt ook informatie van de visuele cortex naar de taalplaatsen gestuurd. Als er laesies aanwezig zijn in de niet-dominante hemisfeer, veroorzaakt dit het onvermogen om emoties in woorden te leggen of te begrijpen.

KCC 19.4 Broca’s aphasia

Een Broca’s afasie ontstaat meestal door een infarct van de a. cerebri media (nogmaals, in 90% van de gevallen is dit de linker a. cerebri media). De patiënt kan niet meer vloeiend spontaan praten. Ook heeft hij last van prosody: het onvermogen om melodie in de woorden te leggen. De spraak bij een Broca’s afasie is met moeite, telegrafisch, zonder grammatica en monotoon. Dit ontstaat ook deels door problemen met een naam geven aan dingen. Ook kunnen patiënten woorden niet herhalen door de isolatie van Wernicke’s area. Deze mensen worden vaak gefrustreerd en depressief. Ook kunnen ze deels verlamd zijn.

KCC 19.5 Wernicke’s aphasia

Bij Wernicke’s aphasia is er sprake van een verminderd taalbegrip. De patiënt kan goed vloeiend en grammaticaal correct praten, maar dit is betekeningloos gepraat met paraphasic errors. De patiënt is zich vaak niet bewust van zijn conditie.

Paraphasic errors zijn het vervangen van woorden door een woord met min of meer dezelfde betekenis (semantic paraphasia) of vervangen door een woord dat bijna hetzelfde klinkt (phonemic paraphasia). Andere klachten kunnen zijn contralaterale gezichtsveld uitval, verlamming, en boos of paranoïde gedrag.

Wat is de niet-dominante hemisfeer?

De niet-dominante hemisfeer is voor ruimtelijk inzicht en gelateraliseerde aandacht. Het bewustzijssysteem zorgt voor alertheid, aandacht en bewust worden van iets. Dit wordt gedaan door vele hersenstucturen. De niet-dominante hemisfeer is meestal de rechter hemisfeer. Deze geeft aandacht aan zowel de linker als de rechter kant, de linker kant iets meer. De linker hemisfeer geeft ook aandacht, maar minder en alleen naar de rechter kant.

Bij laesies van de rechter hemisfeer, is er zware verwaarlozing van de linker kant. Bij laesies van de linker hemisfeer is er een beetje of geen verwaarlozing van de rechter kant. Bij bilaterale laesies is er een klein beetje aandacht naar de linker kant.

Wat is ruimtelijke oriëntatie en integratie?

De ruimtelijke oriëntatie ontstaat door de integratie van vele sensorische informatie, zoals visuele, proprioceptieve, vestibulaire en auditieve informatie. De visuele informatie is het belangrijkste. Visuele informatie wordt verwerkt in twee stromen, namelijk in de ventrale occipitale, temporale en prefrontale cortex (wat zie ik?) en de dorsale (waar zie ik het?).

De parietal association cortex ligt vlakbij de occipital, temporal en prefrontal kwabben in de dorsale stroom. Het zorgt zo voor de analyse van locatie en beweging van visuele objecten in de ruimte.

Frontale kwabben

De frontale kwabben zorgen dat wij mensen effectief en op een sociaal geaccepteerde manier handelen. Patiënten met een laesie hebben echter vaak geen afwijkingen in de spreekkamer, maar kunnen onmogelijk functioneren in de buitenwereld.

De frontale kwab beslaat 1/3e deel van de cerebrale hemisferen. De primaire motorcortex ligt voor de centrale sulcus. Lateraal voor de primaire motorcortex ligt de premotorische cortex en in de dominante hemisfeer Broca’s area. Mediaal ligt voor de primaire motorcortex de supplementary motor area. De frontale oog velden liggen in de premotorcortex lateraal. De limbische systemen liggen deels in de frontale kwabben in de anterior cingulate gyrus en de posteromedial orbitofrontal cortex. De ventrale kwabben bevatten de orbitofrontal olfactory area.

De prefrontale cortex ligt voor de motor en premotor cortex en voor het limbische systeem. Hierin ligt de heteromodel association cortex.

Prefrontale cortex

De prefrontale cortex heeft vele connecties met zowel corticale als subcorticale regio’s, die voornamelijk bidirectionaal zijn. Corticale connecties zijn er met de associatie cortex van de parietale, occipitale en temporale kwab, de unimodal sensory association cortex en de heteromodal association cortex. Verder ook met de motor association cortex en het limbische systeem.

Subcorticale connecties zijn er met de amygdala en de anteromedial temporal cortex door de uncinate fasciculus. Ook zijn er connecties met de hippocampus en de mediodorsal nucleus van de thalamus. Verder is er een connectie met de basale ganglia en ontvangt het projecties van subcorticale en hersenstam modulatory neurotransmitter systemen.

De functies van de prefrontale cortex zijn erg veel. Het zorgt in ieder geval voor ons “mens-zijn”. Dit doordat we ons kunnen inhouden (restraint), begerenswaardige zaken najagen (initiative) en het correct uitvoeren van verschillende stappen achter elkaar (order).

Verder bevat de prefrontale cortex ons werkgeheugen. Dit is onze mogelijkheid tot het vasthouden van informatie voor een korte tijd, terwijl we ook andere cognitieve functies uitvoeren. Dit wordt gedaan door de dorsolaterale prefrontale cortex. De dorsolaterale cortex zorgt ook voor selectieve aandacht en het integreren van informatie van het limbische systeem en de heteromodale associatieve cortex. De dorsolaterale prefrontale cortex en de medial temporal lobes werken samen in het leren van nieuwe stof.

KCC 19.11 Frontale lobstoornissen

Omdat de frontal lobe zoveel verschillende functies heeft, zijn laesies lastig te begrijpen. Symptomen kunnen elkaar tegenspreken, zelfs binnen één patiënt. Dit komt doordat laesies verschillende functionele delen kunnen bevatten. Bilaterale laesies veroorzaken vaak complexe gedragsproblemen.

Er is geprobeerd om een indeling te maken in de laesies, maar hier bestaan nog vele uitzonderingen op. Dorsolaterale convexity laesies veroorzaken een apathische, levenloze staat, terwijl orbitofrontale leasies juist een inpulsieve, ongeremde staat veroorzaken. Verder kan het ook ingedeeld worden in linker frontal laesies, die leiden tot depressie-achtige symptomen en rechter frontal laesies, die leiden tot gedragsstoornissen als manie.

Belangrijke informatie wordt gehaald uit de anamnese en de heteroanamnese, waarbij aan bekenden wordt gevraagd of de patiënt slecht functioneert in de buitenwereld. Er moet goed gelet worden op gedrag abnormaliteiten.

Patiënten kunnen passief en apathisch zijn, zonder spontane activiteit, vertraagde reacties en kort en zacht praten. Patiënten kunnen ook ongeremd zijn, met vreemd gedrag, ongepaste grapjes en agressief gedrag. Patiënten hebben nauwelijks inzicht in hun stoornis en kunnen vertrouwelijke informatie vertellen. Patiënten kunnen ook omgevingsafhankelijkheid vertonen, waardoor ze reageren op elke stimulus. De patiënten hebben een verstoorde concentratie en soms incontinentie, maar daar geven ze niet om.

Neurologisch onderzoek wordt ook gedaan naar de mentale status van de patiënt. De concentratie en het geheugen worden getest, de omgevingsafhankelijkheid wordt getest. Bij een Go-No-Go test moet de patiënt een vinger opsteken als er één keer geklapt wordt en geen vinger opsteken als er twee keer geklapt wordt. Bij de Stroop test moet de patiënt de kleuren benoemen van woorden als rood en groen, die in een andere kleur afgedrukt zijn.

Patiënten hebben ook moeite met het vormen van lijsten van verwante woorden. Bij testen waarbij de patiënt woorden moet verzinnen, zijn een sensitieve test van een dominante frontale laesie. Een voorbeeld van zo een test in de FAS test, waarbij de patiënt achtereenvolgens binnen 60 seconden zoveel mogelijk woorden moet verzinnen met een F, vervolgens een A en vervolgens de S.

De abstracte redenering wordt getest door middel van het benoemen van gelijkenissen in zelfstandige naamwoorden. Dit moet van makkelijk naar moeilijk gedaan worden, om zo inzicht te krijgen in de ernst van de aandoening. De mate van goed oordelen van de patiënt kan het best getest worden door het te vragen aan bekenden van de patiënt.

Een niet-vloeiende afasie kan duiden op frontaalkwab dysfunctie. Soms kan er ook verwaarlozing van de linker kant gezien worden bij rechter frontale laesies.

De schedelvorm moet ook onderzocht worden, omdat deze door bepaalde ziektes afwijkingen kan geven. De reuk moet onderzocht worden, omdat tumoren in de orbitofrontale frontal kwab anosmie veroorzaken. Saccades moeten onderzocht worden aan de hand van het testen van de optokinetische nystagmus.

Er kunnen ook motorische afwijkingen zijn. Bij een primaire motorcortex of precentrale gyrus laesie, ontstaat hemiparese. Bij een prefrontale cortex laesie ontstaan reflex afwijkingen of loopafwijkingen. Ook kan er sprake zijn van paratonie, over verhoogde spierspanning.

Patiënten kunnen ook primitieve reflexen die aangeboren zijn, maar later verdwijnen, weer opnieuw vertonen. De belangrijkste hiervan is het grasp-reflex. De arts aait de handpalm van de patiënt, waarna de patiënt de hand van de arts grijpt. Als de arts vraagt waarom doe je dat, reageert de patiënt met weet ik niet.

Afwijkingen in het looppatroon zijn meestal een schuifelende, onvaste magnetische gang (voeten blijven aan de vloer geplakt). In de differentiële diagnose moeten ook laesies zitten van andere hersenstructuren, omdat deze dezelfde symptomen kunnen veroorzaken als een frontale kwab laesie.

Visuele associatie cortex

Als er sensorische input is naar de primaire visuele cortex, gaat dit vervolgens twee kanten op.

De dorsale weg is naar de parieto-occipital assoctiation cortex, waar de vraag waar? beantwoord wordt. Dit wordt gedaan door de analyse van beweging en ruimtelijke relaties.

De ventrale weg is naar de occipitotemporale association cortex, waar de vraag wat? bantwoord wordt. Dit wordt gedaan door de analyse van vorm, kleur, gezicht, letters en andere stimuli.

Er zijn nog vele dingen onduidelijk over de visuele associatie cortex, waar veel onderzoek naar gedaan wordt.

Één van deze problemen is het bindingsprobleem. Hoe kan het dat we verschillende sensorische input (zicht, gehoor) kunnen combineren tot één concept?

Een tweede probleem is de visual mental imagery, hoe kunnen we beelden vormen in ons hoofd, zonder dat we echt iets zien?

Een derde probleem is het fenomeen blindsight. Hoe kan een blind persoon toch taken uitvoeren die zicht nodig hebben, zoals een envelop door de gleuf in een brievenbus gooien?

Hoe werkt het bewustzijnssysteem?

Bewustzijn kan verdeeld worden in de inhoud van bewustzijn en de mate van bewustzijn. De inhoud van het bewustzijn is waar men bewust van wordt, hierbij zijn de sensorische, motorische en emotionele systemen betrokken. De mate van het bewustzijn wordt geregeld door netwerken die werken op de voorgaande systemen, zoals de hersenstam, de thalamus en hypothalamus en meer. Deze zorgen dan voor alertheid, aandacht en bewust wording van jezelf en omgeving.

Bij aandacht zijn er twee componenten: selectieve aandacht en het volhouden van aandacht (concentratie).

Selectieve aandacht is aandacht voor bepaalde objecten of stimuli en het negeren van andere. Hierbij worden andere hersenstructuren betrokken, bijvoorbeeld bij aandacht voor het zien van een lamp, worden de betrokken delen van de visuele cortex geactiveerd. Bij selectieve aandacht zijn ook specifieke delen van het limbische systeem en de associatie cortex betrokken.

Concentratie ontstaat door ten eerste selectieve aandacht, en het negeren van andere stimuli. Vervolgens wordt aandacht behouden door je niet te laten afleiden.

Belangrijk bij beide processen is het vergroten van het gewenste signaal en de onderdrukking van ongewenste stimuli. Om de aandacht te verschuiven van het ene onderwerp naar het andere moet het ene onderwerp losgelaten worden en het andere vastgehouden worden (geestelijk).

De hersenstructuren die betrokken zijn bij aandacht zijn de widespread projection systems, de frontal en parietal association cortex, de anterior cingulate cortex en limbische wegen, het tectum, de pretectal area en pulvinar en cerebellum en basale ganglia. De hersenstructuren die betrokken zijn bij selectieve aandacht zijn vaak ook betrokken bij concentratie, en andersom. Voor aandacht is er een wakkere, alerte staat nodig. Dit ontstaat na “opwinding”.

Widespread projection systems: de opwindingsmechanismen voor de widespread projection systems zijn de bovenste hersenstam, thalamus, hypothalamus en basale voorbrein.

De bovenste hersenstam stuurt cholinerge en non-cholinerge projecties naar de thalamus, hypothalamus en basale voorbrein. Deze structuren hebben vele projecties naar verschillende cortexen. Nonadrenerge en serotonische systemen hebben ook projecties naar verschillende cortexen. Dopaminerge systemen hebben projecties zijn het striatum, de limbische cortex en de prefrontale cortex.

De opwindingsmechanismen van de thalamus zijn intralaminar, midline, ventral medial nuclei die inputs van de bovenste hersenstam en cholinerge nuclei naar de cerebrale cortex versturen.

Bij de hypothalamus zijn de betrokken structuren de posterior lateral hypothalamic histaminergic neurons en orexinergic neuron. Deze ontvangen input van de anterior hypothalamus en hersenstam. Ze hebben projecties naar de cortex en thalamus.

De basal forebrain systems zijn de nucleus basalis, diagonale band en mediaal septal cholinergic en GABAergic neuronen. Deze ontvangen input van de hersenstam. Ze projecteren naar de hele cortex en de thalamus.

Frontal en parietal association cortex: de parietal cortex ligt vlakbij de auditieve, visuele en somatosensorische unimodale associatie cortex. Hierdoor is het erg geschikt voor heteromodale ruimtelijke oriëntatie.

De frontal heteromodal association cortex is belangrijk voor aandacht. Het verzorgt de motorisch-intentionele aspecten, de reorientatie naar stimuli en het behouden van aandacht en verminderen van afleidingsmogelijkheden.

Anterior cingulate cortex en limbische wegen: de anterior cingulate cortex is verbinden met verschillende limbische structuren. Samen zorgt het voor de motivatie voor aandacht voor een bepaalde stimulus.

Tectum, pretectal area en pulvinar: deze structuren werken samen met de parietotemporo-occipital cortex om de visuele aandacht naar een relevante stimulus te sturen, door middel van saccades.

Bewustzijn van zichzelf en omgeving

Bewustzijn (of awareness) is één van de moderne mysteries. Voor bewustzijn is er een inhoud en een mate van alertheid, aandacht en bewust wording nodig. Belangrijk is onze subjectieve persoonlijke gewaarwording. Bewuste gewaarwording is ons vermogen om verschillende vormen van informatie om te zetten in een efficiënte samenvatting van mentale activiteit.

Geheugen heeft sterk te maken met awareness. Het geheugen kan worden ingedeeld in declaratief (bewuste gewaarwording) en nondeclaratief geheugen. Er zijn aanwijzingen dat dezelfde mechanismen werken bij zowel aandacht als bewustwording. Dit betekent dat er misschien wel geen onderscheid gemaakt kan worden.

Een aspect van awareness is de binding van sensorische, motorische, emotionele en geheugen informatie, waardoor we alles samen als één ervaring beschouwen. Theorieën voor deze binding zijn horizontale connecties tussen corticale lagen en specifieke regio’s voor dit fenomeen.

Ons werkgeheugen, begrip van chronologische volgordes en zelfcontrole spelen een belangrijke rol in awareness. Als een mens wakker is, maar niet met een bepaalde taak bezig is, is er een resting state activity in de hersenen. Dit zijn spontane fluctuaties van activiteit. Als een persoon tegelijk aan het dagdromen is, is er een default mode network activity.

De medial parietal region speelt een potentieel belangrijke rol in zelfreflectie, introspectie en zelfbewustzijn. Dus, het is nog onduidelijk hoe bewuste gewaarwording werkt, maar het kan te maken hebben met lokale gespecialiseerde processen en distributienetwerken.

KCC 19.15 Delirium en andere acute mentale status aandoeningen

Mental status disorders zijn vermindering van aandacht, verwardheid en geheugenverlies, soms samen met verminderde alertheid. Dit wordt encephalopathie genoemd (diffuse brain dysfunction). Het ontstaat vaak bilateraal in de hersenen.

Encephalopathie kan acuut (uren tot weken) en chronisch (maanden tot jaren) zijn. Vaak is het nog moeilijk om te bepalen of het acuut ontstaan is of chronisch is, omdat familieleden vaak hun ogen sluiten voor de verslechterde mentale status.

Acute encephalopathie: ontstaat vaak na toxische of metabole vergiftiging, infectie of trauma en is vaak omkeerbaar. Verminderde aandacht is vooral bij encephalopathie van acute aard. Er is sprake van verwardheid, een fluctuerend niveau van alertheid en moeite met het produceren van nieuwe herinneringen. Een delirium is een acute, verwarde staat met agitatie en hallucinatie. Dit ontstaat vaak bij alcoholisten als ze stoppen met drinken (delirium tremens).

Het is belangrijk om eerst de patiënt te evalueren: bloedonderzoek, vitale functies, urineonderzoek. Vervolgens moet de medicatie van de patiënt bekeken worden voor toxische effecten. Als er niks gevonden wordt, moet er achtereenvolgens neuronale beeldvormende techniek, een lumbaal punctie en een EEG (voor subtiele toeval of vertraagde signaaloverdracht) gemaakt worden.

Bij oudere patiënten ontstaat het nogal eens na een schijnbaar kleine oorzaak, zoals een infectie. Op de intensive care kan het ook nogal eens voorkomen door verdovende middelen, immobilisatie en slaap.

Chronische encephalopathie: er is een progressieve verslechtering en heeft dus een slechte prognose. Maar er kan ook een statische verhouding zijn, dus niet slechter en niet beter worden. De term dementie wordt gebruikt voor een afname in mentale functies.

 

Wat is zijn het unimodaal en het heteromodaal associatie cortex?

 

Unimodale associatie cortex houdt zich primair bezig met maar één soort informatie, dus motorisch of sensorisch voor reuk, sensorisch voor zicht enzovoort. De unimodale sensorische associatie cortex krijgt input van de bijbehorende primaire sensorische cortex. De unimodale motorische cortex stuurt output naar de primaire motorische cortex en is belangrijk voor planning van moeilijke bewegingen.

Heteromodale associatie cortex integreert verschillende soorten informatie, zowel motorisch als sensorisch én met het limbische systeem. Dit zorgt voor de hoogste cerebrale functies. Deze cortex ligt voornamelijk in de frontale kwabben en de parieto-occipitotemporal junctions.

Join World Supporter
Join World Supporter
Log in or create your free account

Why create an account?

  • Your WorldSupporter account gives you access to all functionalities of the platform
  • Once you are logged in, you can:
    • Save pages to your favorites
    • Give feedback or share contributions
    • participate in discussions
    • share your own contributions through the 7 WorldSupporter tools
Follow the author: Vintage Supporter
Promotions
Image

Op zoek naar een uitdagende job die past bij je studie? Word studentmanager bij JoHo !

Werkzaamheden: o.a.

  • Het werven, aansturen en contact onderhouden met auteurs, studie-assistenten en het lokale studentennetwerk.
  • Het helpen bij samenstellen van de studiematerialen
  • PR & communicatie werkzaamheden

Interesse? Reageer of informeer

verzekering studeren in het buitenland

Ga jij binnenkort studeren in het buitenland?
Regel je zorg- en reisverzekering via JoHo!

Supporting content
Medicine & Healthcare Worldwide: learn, study or share - Starting page
Access level of this page
  • Public
  • WorldSupporters only
  • JoHo members
  • Private
Statistics
[totalcount] 1
Comments, Compliments & Kudos

Add new contribution

CAPTCHA
This question is for testing whether or not you are a human visitor and to prevent automated spam submissions.
Image CAPTCHA
Enter the characters shown in the image.